LPN NCLEX Case Study Questions + Ch. 31 Care Of The Child With A Physical/Mental/Cognitive Disorder

अब Quizwiz के साथ अपने होमवर्क और परीक्षाओं को एस करें!

abortion (AB)

1. Induced - on purpose or THERAPEUTIC (to save mom's life). 2. Spontaneous - < 20 wks or it happens out of no where. • bleeding + cramping = threatened 3. Complete - all products of conception are out. 4. Miscarriage - unoticed.

What is the largest fluid compartment in the body? 1. Intracellular 2. Extracellular 3. Interstitial 4. Intravascular

1. Intracellular

Rogan

1. Mom blood type O or Rh - * antibodies AB 2. Check babies blood type * potential for blood exchange * problematic if baby is type A or B * leads to jaundice 3. If mom is Rh- * candidate for Rogan (blood product)

Preclampsia

1. Protein spilling in urine as a result of HTN. 2. Kidneys are now compromised. 3. S/S: Headache, visual disturbances, epigastric pain. 4. Severe preeclampsia S/S: amasada (general edema), ascites, output < 30 mL, Clonus which lead to seizure activity, hypereflexive. 5. Tx: prevent eclampsia (occurs when the mother begins to have seizure activity

Contraction of the aorta

1. Restriction of the aorta = high pressure on the left side of the heart * VSD may be involved

Hypertrophic pyloric stenosis

1. S/S: PROJECTILE VOMITING, olive-shape mass in the epigastrium to the right of the umbilicus, visible peristalsis. 2. Cause: Unknown, males are affected more than girls. 3. Treatment/Interventions: Surgical relief of the pyloric obstruction. * Metabolic alkalosis

Idiopathic Thrombocytopenic Purpura

1. S/S: Petechiae 2. Risk factors: viral infections, females 3. Treatment/ Interventions: Prednisone, splenectomy, in children it resolves without any treatment

renin angiotensin system

1. Start with the kidneys 2. Angiotensin is a potent vasoconstrictor

Blood circulation pattern

1. Superior or Inferior vena cava 2. Right atrium 3. Tricuspid valve 4. Right ventricle 5. Pulmonary semilunar valve 6. Pulmonary artery 7. Capillaries in the lungs 8. Pulmonary veins 9. LA 10. Bicuspid valve 11. LV 12. Aortic semilunar valve 13. Aorta

HTN

1. Systolic pressure * Left ventricle pushing blood all through out the body. * Left ventricle is stronger. * Right ventricle gets blood to the lungs. 2. Diastole * Ventricles rest * Atria take over 3. At times serves as a defense mechanism * Elevated HR & RR - sympathetic nervous system ("fight or flight") 4. Renin angio tension system *Start with the kidneys * Make urine but require good blood flow 5. When there is a decrease in B/P the kidneys release the enzyme called "Renin" * Renin turns into angiotensin when it hits the liver

Pyrazinamide

1. TB medication 2. Either bacteriostatic or bactericidal 3. Available only in oral route 4. Side effects: hepatotoxicity, hyperuricemia

Ethambutol (Myambutol)

1. TB medication 2. Supresses RNA synthesis 3. Do not give to children under 13 years old 4. Side effects: blindness, retrobulbar neuritis

Intussuception

1. Telescoping of one portion of the intestine into another. 2. S/S: JELLY STOOL (feces is mixed with blood and mucus from the intestinal mucosa) 3. Treatment/Med. management: assessment of bowel sounds and v/s, hydrostatic reduction.

The fetal position is ROA. Where is the fetal presenting part in relation to the maternal pelvis? 1. The occiput is facing the right side and the front of the maternal pelvis. 2. The mentum is facing the right side and the front of the maternal pelvis. 3. The occiput is facing the left side and the back of the maternal pelvis. 4. The sacrum is facing the right side and the front of the maternal pelvis.

1. The occiput is facing the right side and the front of the maternal pelvis.

Rifampin (Rifadin)

1. Used for treatment of TB, meningococcus, influenza B, and leprosy 2. Bactericidal agent 3. Causes URINE, FECES, SALIVA, SWEAT, SKIN, SPUTUM, TEARS, a red orange color. 4. Oral birth control become ineffective 5. Side effects: Hepatitis, hematologic disorders

Magnesium Sulfate

1. Decreases B/P (vasodilation) - CNS depressant. 2. Side effects: flushed, hot, disoriented, heaviness feeling on chest, RR (MONITOR), HR (MONITOR). 3. Assess - deep tendon reflexes, hyperreflexia = Tx not working, HYPOREFLEXIA = magnesium sulfate toxicity. Calcium gluconate is the antidote.

Depolarization and repolarization

1. Depolarization: electrical activity when the heart contracts 2. Repolarization: relaxation phase of the heart

ectopic pregnancy

1. Dialation & curatage (D&C) • cells must be taken out completely, may turn cancerous. 2. HCG levels are drawn to make sure no cells are left. 3. Refrain from pregnancy for at least 1 year.

Sodium (Na+)

1. Extracellular electrolyte 2. Regulates muscle contractility 3. Maintains acid - base balance 4. Normal: 135 - 145

Gestational HTN

1. HTN = change > 30 systolic/ change > 15 diastolic. 2. Progressive disease. 3. Gestational HTN can progress to preclampsia

Veins

1. Have valves 2. Slow blood flow * easy for clots to form 3. Bilateral issues * Ex: bilateral edema 4. Pigmentation - red/darker hue. 5. O2 accumulated at the site. * Warmth at the site * Most patients are not walkers, bed bound

Postpartum complications

1. Hemorrhage 2. Uterine atony • overstretched uterus • causes: Multiple pregnancies, hydromnios, Full bladder.

Atrial septal defects

1. Hole in the wall between the RA and LA 2. Blood goes from LA to RA 3. S/S: asymptomatic, paradoxical embolus, potentially leads to a stroke 4. Split sound of "Lub" & "Dub" when auscultating.

Bronchopulmonary dysplasia

1. Impaired gas exchange 2. S/S: retraction, nasal flaring, cyanosis when stressed. * Develops in premature infants * Risk factor: PDA

A test that may be done in late pregnancy to determine fetal well-being is the nonstress test. This test is based on which phenomenon? 1. Fetal heart rate increases in connection with fetal movement. 2. Braxton Hicks contractions cause an increase in fetal heart rate. 3. Fetal heart rate slows in response to contractions. 4. Fetal movement causes an increase in maternal heart rate.

3. Fetal heart rate slows in response to contractions.

A patient with sickle cell anemia asks the nurse why the sickling crisis does not stop when oxygen therapy is started. Which statement by the nurse is the most accurate answer? A) "sickling occurs in response to decreased blood viscosity, which is not affected by oxygen therapy." B) "When red cells sickle, they occlude small vessels, which causes more local hypoxia and more sickling." C) "The primary problem during sickling cell crisis is destruction of abnormal cells, resulting in fewer RBCs to carry oxygen." D) "Oxygen therapy does not alter the shape of the abnormal erythrocytes but only allows for increased oxygen concentration in Hgb."

B) "When red cells sickle, they occlude small vessels, which causes more local hypoxia and more sickling."

The health care provider has talked to a patient and his wife about the treatment plan for his bladder cancer. Later, the patient tells the nurse he does not understand what the health care provider is going to do. What is the most appropriate initial response by the nurse? A) "Okay. I'll explain it to you again." B) "Make a list of questions for the doctor." C) "Try not think about the treatment." D) "Tell me what you know about the treatment."

D) "Tell me what you know about the treatment."

An infant has passed currant jelly stools. Which diagnostic test is most likely to be performed? A) Endoscopy B) Ultrasonography C) Barium esophagography D) Barium enema

D) Barium enema Currant jelly stool is characteristically observed in intussusceptions: the most common cause of intestinal obstruction. BE test demonstrates obstruction in intestine and helps in diagnosis of intussusceptions. Endoscopy examination is performed for the assessment of gastroesophageal reflux. Ultrasonography is performed for assessment of hypertrophic pyloric stenosis. Barium esophagography is a diagnostic test for gastroesophageal reflux.

A newborn male infant is being assessed by the pediatrician in the nursery. The infant is showing signs of heart failure, and an audible machine-like murmur is heard at the upper left sternal border. Further assessment finds a widened pulse pressure and bounding pulses. Which type of congenital heart disease does this assessment data identify? A) Tetralogy of Fallot B) Atrial septal defect C) Patent ductus arteriosus D) Ventricular septal defect

D) Ventricular septal defect Failure of the ductus arteriosus to close within the first weeks of life leads to these assessment data. Ventricular septal defect causes signs of heart failure with a loud, harsh systolic murmur and palpable thrill. Arterial septal defect causes signs of heart failure with a harsh systolic murmur auscultated over the third intercostal space. Tetralogy of Fallot results in severe cyanosis and hypoxia, systolic ejection murmur, clubbing of the nail beds, dyspnea, squatting, poor growth, mental slowness, syncope, and cerebrovascular disease.

The priority short term goal for disorders of the urinary system is: A) patient confidentiality B) privacy C) education for patient and family D) normal patterns of urinary elimination

D) normal patterns of urinary elimination

What is diffusion, osmosis, and filtration?

Diffusion - movement of particles from an area of high concentration to low concentration. * Ex: CO2 out of cells Osmosis - H2O into and out of cells to correct imbalances in H2O concentration. Filtration - movement of H2O and small solute particles, but not larger particles through a filtration

Give an example of a traditional antihistamine: (Drugs used for URIs)

Diphenhydramine (Benadryl) - used for prevention/relief of allergies, motion sickness, sleep-aid.

Exercise stress EKG

Dipyridamole (Persantine) or Adenosine (Adenocard) may be used if the patient can't tolerate activity. Makes the patient's heart respond as if they were stress/activity.

The collection of objective data for a patient with acute glomerulonephritits could include which symptoms? (SELECT ALL THAT APPLY). A) Periorbital edema B) Anorexia C) Hypotension D) Frankly sanguineous urine E) Headaches

A) Periorbital edema B) Anorexia C) Hypotension

The nurse is reviewing the health history of a patient suspected of having renal calculus. What factors in the patient's history increases the patient's risk factor for developing the condition? (SELECT ALL THAT APPLY). A) Stasis of urine caused by obstruction B) Infections of urinary tract C) Hypoparathyroidism D)Diabetes mellitus E) Immobility

A) Stasis of urine caused by obstruction B) Infections of urinary tract

A client complains of left-sided chest pain after the client finished playing racquetball. The client is hospitalized and diagnosed with left pneumothorax. When assessing the client's left chest area, the nurse expects to identify: a) Dull sound on percussion b) Vocal fremitus on palpation c) Rales with rhonchi on auscultation d) Absence of breath sounds on auscultation

d) Absence of breath sounds on auscultation The left lung is collapsed; therefore, there are no breath sounds. A tympanic, not a dull, sound will be heard with a pneumothorax. There is no vocal fremitus because there is no airflow into the left lung as a result of the pneumothorax. Rales with rhonchi will not be heard because there is no airflow into the left lung as a result of the pneumothorax.

What age group is most likely to be diagnosed with scoliosis? a) Adults b) Infants c) Children d) Adolescents

d) Adolescents Scoliosis can occur at any age but is most often seen in adolescent girls.

You are assuming care of a toddler who has lead poisoning. You know that children are at higher risk form lead poisoning because: a) The bodies of young children absorb at a higher rate b) The brains of young children are developing rapidly c) Young children often chew or swallow non-food items d) All of these

d) All of these

What should the nurse do while preparing a patient suspected of having a brain tumor for a computer tomography (CT) scan of the brain? a) Immobilize the patient's neck. b) Place a cap over the patient's head. c) Administer a low-dose sedative to the patient. d) Determine whether the patient is allergic to iodine.

d) Determine whether the patient is allergic to iodine. A CT scan involves the use of iodine dye in the contrast medium. Before performing the CT scan, the nurse should determine whether the patient is allergic to iodine. Immobilize the patient only if the patient has a

While collecting data of an infant who is fed with bovine's milk, the nurse observes that the infant is fussy and has a pale tongue. What instruction does the nurse expect from the primary health care provider (PHP) for this condition? a) Provide the infant with kangaroo care. b) Feed the infant with only breast milk. c) Avoid placing the infant on the pillow. d) Give iron-fortified formula to the infant

d) Give iron-fortified formula to the infant Bovine's milk, or cow's milk, may cause bloody stools in infants. Due to elimination of blood through the stools, the infant is likely to develop anemia. Anemic infants look pale and are fussy. The PHP would instruct the nurse to feed iron-fortified milk to the infant to prevent anemia. Kangaroo care is given to ease the child from being fussy, but this is not related to blood iron levels in the infant. Feeding the child with only breast milk will not be sufficient, as breast milk is deficient of iron. Parents should avoid placing the infants on pillows, as this may lead to suffocation.

An adult patient comes to the clinic with white patches in the mouth that are diagnosed as oral thrush (candidiasis). The nurse knows this patient should be tested for which disease? a) Hepatitis C virus (HCV) b) Herpes simplex virus (HSV) c) Haemophilus influenzae type B (HIB) d) Human immunodeficiency virus (HIV)

d) Human immunodeficiency virus (HIV) Oral thrush is not an infection that occurs in patients with healthy immune systems. The patient should be tested for HIV. Testing for HSV, HIB, and HCV is not necessary.

What should the nurse discuss with new parents to help them prepare for infant care? a) Allowing crying time to help the lungs develop b) Establishing a set feeding schedule to promote steady weight gain c) Counting the number of stool diapers daily to confirm adequate hydration d) Learning specific behaviors involving states of wakefulness to promote positive interactions

d) Learning specific behaviors involving states of wakefulness to promote positive interactions Discussing behaviors during the baby's waking times that will promote positive interaction helps parents understand the unique features of their newborn and promotes interaction and care during periods of wakefulness. A healthy infant's lungs are developed at birth. It is best that infants be on a demand feeding schedule, not a routine schedule. Demand feeding provides for individuality; healthy infants gain weight steadily. Counting the number of stool diapers daily is not a reliable method of determining adequate hydration.

A client with a history of hypertension and left ventricular failure arrives for a scheduled clinic appointment and tells the nurse, "My feet are killing me. These shoes got so tight." The nurse's best initial action is to: a) Weigh the client b) Notify the primary health care provider c) Take the client's pulse rate d) Listen to the client's breath sounds

d) Listen to the client's breath sounds After the assessment protocol of airway, breathing, and circulation (ABCs), the nurse should assess the client's breath sounds for crackles that may indicate the development of heart failure. Although the client should be weighed, it is not the priority. Shoes that become too tight indicate pedal edema, which is a sign of fluid retention; 1 L of fluid weighs 2.2 pounds. Eventually the health care provider will be notified, but the nurse should have more data for the practitioner. Taking the client's pulse rate will happen, but not as a priority.

After an amputation, a client's residual limb is bandaged snugly throughout the postoperative period. The nurse teaches the client that the primary purpose of the rigid bandaging of the residual limb is to: a) Prevent suture line infection b) Promote drainage of secretions c) Prevent injury to the residual limb d) Promote shrinkage of the distal end of the residual limb

d) Promote shrinkage of the distal end of the residual limb The wrapping of the residual limb is done to reduce swelling and shape the limb for fitting a prosthesis in the future. Infection is not prevented in this manner; surgical asepsis should be maintained. Secretion drainage is not promoted by wrapping the limb; portable drainage systems are used for this purpose. A special sock is used to protect the residual limb from irritation and injury.

Which is the correct impulse pattern of the cardiac conduction system? a) Pacemaker → Bundle of His → Sinoatrial (SA) node → Bundle branches b) Purkinje fibers → Sinoatrial (SA) node → Right and left bundle branches → AV node c) Pacemaker → Sinoatrial (SA) node → Bundle of His → Atrioventricular (AV) node → Purkinje fibers d) Sinoatrial (SA) node → Atrioventricular (AV) node → Bundle of His → Right and left bundle branches → Purkinje fibers

d) Sinoatrial (SA) node → Atrioventricular (AV) node → Bundle of His → Right and left bundle branches → Purkinje fibers The impulse pattern of the cardiac conduction system is as follows: SA node → AV node → Bundle of His → Right and left bundle branches → Purkinje fibers

A nurse is providing education to a patient with severe obstructive sleep apnea (OSA) who has not been compliant with nasal continuous positive airway pressure (CPAP) therapy. Which suggestion would be most helpful? a) Lose at least 15 pounds b) Drink one glass of alcohol before bed c) Use a mouthpiece to bring jaw and tongue forward while sleeping d) Use a bilevel positive airway pressure (BiPAP) system instead of a CPAP system

d) Use a bilevel positive airway pressure (BiPAP) system instead of a CPAP system BiPAP systems are more comfortable for patients, and the patients are more likely to be compliant with treatment. Because of the patient's severe OSA, a 15-pound weight loss and mouthpiece are unlikely to be helpful, because these are treatments for mild OSA. Alcohol worsens OSA.

Your patient, who is recovering from heart surgery, suddenly becomes unresponsive. You determine that the patient is apneic and pulseless and begin CPR, calling for help as you do so. The cardiac monitor shows a wide, regular and rapid rhythm. You know that this rhythm is probably: a) Ventricular fibrillation (V-Fib) b) Sinus bradycardia c) Normal rhythm the patient is recovering d) Ventricular tachycardia

d) Ventricular tachycardia Ventricular tachycardia appears as a wide, rapid and regular rhythm on the cardiac monitor or ECG. A recent history of angina, CHF or MI makes it more likely that the rhythm is ventricular tachycardia, rather than a more benign rhythm with aberrancy.

The nurse concludes that a client with glaucoma needs education when the client states "It is dangerous for me to: Multiple choice question Use sedatives." Lift heavy objects." Become constipated." Take atropine in any form."

Use sedatives." Sedatives have no effect on intraocular pressure. Additional teaching is not necessary; lifting heavy objects, becoming constipated, and taking atropine in any form should be avoided because they will increase intraocular pressure.

A nurse is caring for a patient with myasthenia gravis. The nurse knows weakness of which muscles is the most dangerous complication of myasthenia gravis? a) Vocal cord b) Diaphragm c) Anal sphincter d) Bladder sphincter

b) Diaphragm Although the patient may experience weakness of the diaphragm, vocal cord, anal and bladder sphincters, the most dangerous is the diaphragm, because the diaphragm is required for spontaneous respiration. Patients with weakness of the diaphragm may need to be mechanically ventilated to sustain life.

The nurse is reviewing a teaching plan for a client that has been prescribed a 2-gram sodium diet. The plan should include which foods that are low in sodium? a) Meat and fish b) Fruits and juices c) Milk and cheese d) Dry cereals and grains

b) Fruits and juices

While a client with an abdominal aortic aneurysm is being prepared for surgery, the client complains of feeling lightheaded. The client is pale and has a rapid pulse. The nurse concludes that the client is: a) Hyperventilating b) Going into shock c) Experiencing anxiety d) Developing an infection

b) Going into shock The clinical findings are early signs of shock. Shock ensues rapidly after a ruptured aortic aneurysm because of profound hemorrhage. The nurse can observe hyperventilation by watching the client's breathing patterns; rapid respirations are expected with hyperventilation. There are no data that indicate that the client is hyperventilating. Anxiety usually is not associated with lightheadedness unless there is accompanying hyperventilation. The signs and symptoms are not inclusive enough to indicate infection; there is no indication of fever.

The nurse is collecting the data of a patient who has symptoms of weakness, weight loss, and shortened breath. The nurse measures the patient's inflamed tissue as 8 mm after a Mantoux test. What immediate care would the primary health care provider prescribe to ensure the patient's safety? a) Intranasal fluticasone (Flovent) b) Isoniazid (INH) and rifampin (Rifadin) c) 100 mg of zafirlukast (Accolate) intravenous d) Theophylline (Accurbron) and para-aminosalicylate sodium (PAS)

b) Isoniazid (INH) and rifampin (Rifadin) Weakness, weight loss, and shortened breath are the characteristic symptoms of tuberculosis. The Mantoux test, or tuberculin skin test, is performed to identify the presence of mycobacterium (TB) in the blood. The tissue on the skin gets inflamed and hardened; if the tissue measures more than 5 mm, it confirms that the patient is infected with tuberculosis. The primary health care provider will immediately prescribe first line antituberculosis drugs such as isoniazid (INH) and rifampin (Rifadin) to the patient. Fluticasone (Flovent) is a corticosteroid used in treating various respiratory tract infections and also used in treating TB as second line agent. Zafirlukast (Accolate) is an anti-inflammatory agent used in the treatment of asthma. Theophylline (Accurbron) is a bronchodilator used to treat conditions like asthma. Para-aminosalicylate sodium (PAS) is a second line anti-tuberculosis agent used after treating with the first line drugs.

Your client is receiving Tobramycin 80 mg TID IV. The physician has ordered a peak and trough with the 3rd dose. When should these levels be drawn? a) Peak immediately before the 3rd dose; trough immediately after the 3rd dose. b) Peak one hour before the 3rd dose; trough just prior to the 4th dose. c) Peak one hour before the third dose; trough 30 minutes after the 3rd dose has infused. d) Trough one hour before the 3rd dose; peak one hour after the 3rd dose has infused.

b) Peak one hour before the 3rd dose; trough just prior to the 4th dose. Peak and trough are methods used to establish the effectiveness of a drug. Peak is drawing the serum blood levels after the drug is administered as it distributes and reaches its peak therapeutic range. Trough is drawing the serum blood levels right before the next dose. Trough is the lowest drug level that is needed to reach therapeutic range.

A nurse is providing discharge instructions for a client with angina who has a prescription for sublingual nitroglycerin tablets. The nurse should teach the client that the nitroglycerin sublingual tablets have lost their potency when: a) Sublingual tingling is experienced. b) The tablets are more than three months old. c) The pain is unrelieved, but facial flushing is increased. d) Onset of relief is delayed, but the duration of relief is unchanged.

b) The tablets are more than three months old. Nitroglycerin tablets are affected by light, heat, and moisture. Loss of potency can occur after three months, reducing the drug's effectiveness in relieving pain. A new supply should be obtained routinely. Experiencing sublingual tingling indicates the tablets have retained their potency. Unrelieved pain with an increase of facial flushing and delayed relief with the duration of relief remaining the same do not necessarily indicate loss of potency.

A nurse is attempting to interview a patient in severe pain. In an effort to promote communication, what is the best action the nurse can take? a) postpone the interview until the pain is manageable b) address the pain and then proceed with the interview c) ask the patient to ignore the pain during the interview d) promise the patient pain medication after the interview

b) address the pain and then proceed with the interview

A nurse is preparing a patient with left-sided heart failure for diagnostic imaging. The nurse anticipates the diagnostic imaging will show which anomaly? a) Splenomegaly b) Hepatomegaly c) Pleural effusion d) Mediastinal shift

c) Pleural effusion Left-sided heart failure leads to pleural effusion. Right-sided heart failure leads to splenomegaly and hepatomegaly. Left-sided heart failure does not lead to mediastinal shift.

To maintain homeostasis it is necessary for the __________and the __________to balance each other in the body fluids.

"cations (positive ions); anions (negative ions)

In an EKG/ECG what does a T wave reflect?

1. Ventricular repolarization 2. Upright in most leads 3. Last longer than ventricular depolarization

Potential causes of hypocalcemia may include which conditions? (Select all that apply.) 1. Vitamin D deficiency 2. Diuretic use 3. Injury of the thyroid gland 4. Severe burns 5. Renal failure

1. Vitamin D deficiency 2. Diuretic use 4. Severe burns 5. Renal failure

Normal PT (prothrombin time, Pro-time) lab value:

12.0-14.0

Normal platelet lab value:

150,000-400,000

As the woman's labor progresses, which assessment finding indicates that the second stage of labor has begun? 1. Passage of a mucous plug 2. Bearing-down reflex 3. Dilation of the cervix to 7 cm 4. Change in shape of the uterus

2. Bearing-down reflex

Normal PTT (partial thromboplastin time) lab value:

20-25 Sec

In acute respiratory acidosis, the renal compensatory mechanisms begin to operate within how many hours? 1. 6 2. 12 3. 24 4. 48

3. 24

The nurse has just reviewed the fetal heart rate on an assigned laboring patient. What finding indicates the need to notify the charge nurse? 1. Accelerations 2. Early decelerations 3. Average FHR of 126 bpm 4. Late decelerations

4. Late decelerations

Normal fasting glucose lab value:

70-115

Normal Chl (chloride) lab value:

96-106

Cancer occurs when:

A cell's genetic mutations are defective and cannot induce cell death. Cancer occurs when a cell's genetic mutations are defective and therefore cannot induce cell death, also known as apoptosis. In the absence of apoptosis, cell growth becomes uncontrolled and invades various systems of the body, leading to various levels of dysfunction. Uncontrolled cell growth masses are also known as tumors.

What causes metabolic acidosis/alkalosis?

A failure of the kidneys to regulate the bicarbonate concentration in the blood.

What causes respiratory acidosis/alkalosis?

A failure of the lungs to regulate the carbonic acid concentration in the blood.

Which problem constitutes a medical emergency? A) Anura B) Polyuria C) Dysuria D) Dyspnea

A) Anuria

Which goal would have priority in planning care of the aging patient with urinary incontinence? A) Recognizes the urge to void B) Mobility necessary for toilet ink independently C) Episodes of incontinence decrease D) Drinks a minimum of 2000 mL of fluids per day

A) Recognizes the urge to void

Aortagram

ABD aorta and major leg arteries are examined by X-ray using a contrast medium. Aneurysms, aortic arch, and branches can be visuallized.

Normal phosphate lab value:

Adult: 3.0-4.5 Child: 4.0-7.0

Give an example of a beta-adrenergic bronchodilator. How do they work?

Albuterol, Epinephrine - used during the acute phase of an asthmatic attack. Reverses airway constriction. 1. Side effects: insomnia, tremors, restlessness, increased HR 2. Patients who have HTN, DM, & DYSRHYTHMIAS need to be monitored closely

Hypotonic solution

All particles from the extra cellular space goes into the cell. * causes edema (ex: in lower extremities)

Prednisone (Meticorten) is prescribed for a client with an exacerbation of colitis. Before administering the first dose, the nurse teaches the client that: Multiple choice question Symptoms associated with the colitis will decrease slowly over time The client will be protected from getting an infection Although the medication causes anorexia, weight loss may not occur Although the medication decreases intestinal inflammation, it will not cure the colitis

Although the medication decreases intestinal inflammation, it will not cure the colitis Prednisone inhibits phagocytosis and suppresses other clinical phenomena of inflammation; this is a symptomatic treatment that is not curative. The response usually is rapid. The drug suppresses the immune response and increases the potential for infection. Appetite is increased; weight gain may result from this or from fluid retention.

Successful communication includes which of the following components?

Appropriateness, efficiency, flexibility, feedback

Atrial flutter

Cardiac dysrhythmia with rapid contractions of 200 to 300 beats/min

The walls of the thoracic cavity are lined with a serous membrane composed of tough endothelial cells; what is this membrane called? A) visceral pleura B) apneustic serosa C) pneumotaxic serosa D) parietal pleura

D) parietal pleura

What does a p-wave represent?

Depolarization of the atria

What does the QRS complex represent?

Depolarization of the ventricles; covers atrial repolarization as well, but can not be seen on the EKG tracing.

Normal cholesterol lab value:

Desirable: <200

When assessing a client with Graves disease (hyperthyroidism) the nurse expects to identify a history of: Multiple choice question Diaphoresis Menorrhagia Dry, brittle hair Sensitivity to cold

Diaphoresis Increased basal metabolic rate, increased circulation, and vasodilation result in warm, moist skin. Menorrhagia, dry, brittle hair, and sensitivity to cold are associated with hypothyroidism.

A nurse expects that a client with right-sided heart failure will exhibit:

Distended neck veins

The LPN is caring for a patient who is unconscious. The LPN provides oral care to the unconscious patient: a) every four hours b) daily c)every two hours d) once a shift

Every two hours For the patient who is unconscious, mouth breathing is common. Therefore, in order to maintain the integrity of the oral cavity and mucous membranes, oral care should be given every two hours and as needed in between.

An active adolescent is admitted to the hospital for surgery for an ileostomy. When planning a teaching session about self-care, the nurse includes sports that should be avoided by a client with an ileostomy. Which should be included on the list of sports to be avoided? Multiple selection question Football Swimming Ice hockey Track events Cross-country skiing

Football Ice hockey Trauma to the abdominal wall and to the stoma should be avoided; contact sports, such as football and ice hockey, are contraindicated . Trauma to the abdominal wall is a minimal risk when swimming. Track events are not associated with trauma to the abdominal wall. Cross-country skiing is not associated with trauma to the abdominal wall.

Give an example of an expectorant: (Drugs used for URIs)

Guafenesin (Robitussin)

If the lungs are unable to correct respiratory acidosis, the __________ will respond in an attempt to correct the imbalance.

Kidneys

An elevated bilirubin may be a sign of:

Liver disease Bilirubin is a yellowish substance found in bile. It is produced by the body when red blood cells are broken down by the liver. Low levels of bilirubin are not typically a concern. High levels of bilirubin, however, may be a sign of disease and require further evaluation.

You need to perform an admission on a patient who does not speak English. What is the best course of action?

Locate a translator and ask them to translate for you while you perform your admission assessment.

A client has an excision of a thrombosed external hemorrhoid. What should the nurse teach the client to use when cleaning the anus after a bowel movement? Multiple choice question Betadine pads Soft facial tissue Medicated pads (Tucks) Sterile 4×4-inch gauze pads

Medicated pads (Tucks) Witch hazel-moistened pads (Tucks) are not irritating and are soothing to the anal mucosa. Betadine may cause excessive drying and irritation. The rectum always is contaminated; external cleansing with Betadine will not affect appreciably the bacteria present. Dry facial tissue is irritating and can cause trauma. Sterile gauze pads are unnecessary; the rectal area is considered contaminated.

insensible water loss

Not measureable * perspiration * expiration

Normal sinus rhythm

Originates in the SA node. Rate: 60-100bpm P waves: precede each QRS complex (atrial depolarization). PR interval: interval between atrial and ventricular depolarization. QRS complex: ventricular depolarization T wave: ventricular repolarization

A nurse assists in the vaginal delivery of a newborn infant. After the delivery, the nurse observes the umbilical cord lengthen and a spurt of blood from the vagina. The nurse documents these observations as signs of what? a) placenta prebia b) abruptio placenta c) placental separation

Placental separation As the placenta separates, it settles into the lower uterine segment. The umbilical cord lengthens, and a sudden trickle or spurt of blood appears.

For what client response must the nurse monitor to determine the effectiveness of amiodarone (Cordarone)? Multiple choice question Results of fasting lipid profile Presence of cardiac dysrhythmias Degree of blood pressure control Incidence of ischemic chest pain

Presence of cardiac dysrhythmias Amiodarone is a class III antidysrhythmic used to treat ventricular and supraventricular tachycardia, and conversion of atrial fibrillation. Results of fasting lipid profile are expected with antilipidemics. Degree of blood pressure control is expected with antihypertensives. Incidence of ischemic chest pain is expected with antianginal agents, such as nitrates.

Give an example of a decongestant: (Drugs used for URIs)

Pseudoephedrine (Sudafed) - Used for treatment of nasal congestion by URIs and allergies. Cause vasoconstriction which reduces blood flow to the affected area of the nose. Use cautiously with patients whom are taking HYPERTENSIVES, patients with DM, BPH, CD, HYPERTHYROIDISM.

Within each state or jurisdiction, the board of nursing defines:

Scope of practice

Prior to swabbing a wound to obtain a wound culture, you should cleanse the wound with:

Sterile normal saline

How do antitussives work? (Drugs used for URIs)

They suppress the non-productive cough. ex: Benzonatate (Tessalon), CODEINE (ROBITUSSIN A-C) is potentially addictive, dextromethorphan (Vicks formula 44)

What is one reason an erythrocyte sedimentation rate (ESR) would be ordered?

To determine and monitor or rule out inflammatory infective conditions. Ex: in rheumatic fever

Graves disease

Treatment: radioactive iodine & surgically removed. Endocrine disorder of increased thyroid hormone production.

Any process that interferes with normal ventilation and causes a decrease or increase in excretions of acids in the body poses the risk of causing (SELECT ALL THAT APPLY): a) respiratory acidosis b) metabolic alkalosis c) respiratory alkalosis d) metabolic acidosis

a) respiratory acidosis c) respiratory alkalosis

Fluroscopy

allows observation of movement of internal structures such as the heart producing a sort of X-ray based motion picture.

The nurse is assessing a client 12 hours after the client sustained a deep partial-thickness burn on the forearm. What characteristics should the nurse expect to identify when assessing the injured tissue? a) Red and swollen b) Blistered and wet c) Charred and white d) Leathery and black

b) Blistered and wet Deep partial-thickness burns involve some injury to the epidermis and dermis, characterized by fluid-filled vesicles that are red, shiny, and wet. Red and swollen describes a superficial partial-thickness burn. The characteristics charred and white, and leathery and black describe a full-thickness burn.

A toddler with a puncture wound to the sole is brought to the emergency department. Because of a language barrier the caregiver cannot provide a clear history of previous tetanus immunizations. Tetanus immunoglobulin (TIG) is prescribed by the practitioner. The nurse explains to the caregiver that this medication is given because it: a) Produces lifelong passive immunity to tetanus b) Confers short-term passive defense against tetanus c) Induces long-lasting active protection from tetanus d) Stimulates the production of antibodies to fight tetanus

b) Confers short-term passive defense against tetanus TIG contains antibodies, not the live or attenuated virus; it confers short-term passive immunity that is temporary. Tetanus toxoid, not TIG, stimulates the production of antibodies.

Which of the following assessment findings could the nurse see in a patient with parkinsonism? (Select all that apply.) a. An abrupt onset of symptoms b. Muscle rigidity c. Involuntary tremors d. Bradykinesia e. Bilateral muscle weakness

b. Muscle rigidity c. Involuntary tremors d. Bradykinesia

Concepts formed as a result of culture, family, friends, education, and work are known as: a. ideas b. values c. religion d. principles

b. values

A client with heart failure is on a drug regimen of digoxin (Lanoxin) and furosemide (Lasix). The client dislikes oranges and bananas. Which fruit should the nurse encourage the client to eat? a) Apples b) Grapes c) Apricots d) Cranberries

c) Apricots Lasix is potassium depleting; apricots have more than 440 mg of potassium per 100 g. Apples have about 80 to 110 mg of potassium per 100 g. Grapes have about 80 to 160 mg of potassium per 100 g, depending on the variety. Cranberries have about 65 mg of potassium per 100 g.

After a complete blood cell count (CBC) is performed for a patient receiving chemotherapy, the nurse learns the patient has a platelet count of 15,000/mm3. What should the nurse's response be? a) Document the findings b) Remove fresh flowers from the room c) Assess the patient's intravenous (IV) entry sites d) Change the patient's diet order to eliminate fresh fruit and vegetables

c) Assess the patient's intravenous (IV) entry sites The patient's platelet count is very low, so the patient is at an increased risk for bleeding. The nurse should assess the patient's skin and IV entry sites for signs of bleeding or bruising. Then the nurse should document the findings. It is not necessary to place the patient on neutropenic precautions (limiting patient's exposure to the microbes on fresh fruit, vegetables, and flowers) for thrombocytopenia.

A nurse caring for a patient on neutropenic precautions who also has stomatitis is asked by the family of the patient what an appropriate gift for the patient might be. Which gift would be appropriate for this patient? a) Fresh fruit basket b) Prepackaged snacks c) Chocolate milkshake d) Bouquet of fresh flowers

c) Chocolate milkshake The patient on neutropenic precautions should not be given fresh fruit or flowers because of the risk for infection. A milkshake may be more tolerable to the patient with stomatitis than prepackaged snacks.

A nurse is providing education to a 50-year-old woman on decreasing her risk of developing Alzheimer disease (AD). Which instruction would be most helpful for this patient? a) Limit foods high in beta carotene b) Engage in minimal weight-bearing exercise c) Complete crossword and other puzzles daily d) Keep a detailed calendar and write in a diary often

c) Complete crossword and other puzzles daily Studies have shown that the patient should be encouraged to engage in activities that require the processing of new information such as puzzles and learning a new language. Exercise and healthy diet are important to AD prevention, as well. The patient should consume foods high in beta carotene and antioxidants. A patient who is beginning to experience symptoms of AD should be instructed to keep a detailed calendar and write in a diary often.

A nurse is preparing to care for a patient with polycystic kidney disease (PKD). The nurse knows this patient is at risk for which fatal disease? a) Liver failure b) Bladder cancer c) End-stage renal disease d) Recurrent urinary tract infections

c) End-stage renal disease PKD can progress to end-stage renal disease, which is fatal. Although PKD may cause recurrent urinary tract infections, these are not necessarily fatal. Liver failure and bladder cancer can be fatal, but these are not caused by PKD.

Which vaccination prevents human papillomavirus (HPV) infections? a) Pertussis b) Tetanus c) Gardasil d) Hepatitis B

c) Gardasil Gardasil is a three-dose vaccine administered to prevent infections caused by human papillomavirus (HPV). This virus is the main cause for genital warts. Girls and young women up to 26 years of age should be vaccinated before becoming sexually active. This vaccine is not effective after exposure to HPV. The pertussis vaccine is used to fight against Bordetella pertusis in preventing typical whooping cough. The tetanus vaccine is used to fight against Clostridium tetani, which harms the nervous system. The hepatitis B vaccine is used to fight against the hepatitis B virus, which causes loss of appetite, vomiting, and nausea.

The nurse studies the arterial blood gas levels of a patient with bronchitis and finds that the patient has hypercapnia. What value would be consistent with the nurse's findings? a) pH is 7.40 b) Sa02 is 96% c) PaCO2 is 65mm Hg d) PaO2 is 50mm Hg

c) PaCO2 is 65mm Hg Hypercapnia is the condition in which, the carbon dioxide levels are increased more than the normal levels. The normal carbon dioxide levels in the body ranges from 35-45 mm Hg. The pH of 7.40 indicates normal finding in arterial blood gas report. Saturated oxygen levels are normal when they are above 95%. Partial oxygen levels as 50 mm Hg indicates hypoxemia, but not hypercapnia.

Your patient has just expired as the result of pneumocystis jirovecci pneumonia, an opportunistic infection, secondary to the patient's known positive HIV status. As you are performing post mortem care, you should do which of the following? a) Do not necessarily use standard precautions because the client is dead and no longer infectious b) place the patient in a double shroud to prevent the spread of infection during transport c) care for the client in the same manner that you did while he was alive d) use protective eye wear to prevent respiratory droplet infection from pneumocystis jirovecii pneumonia spray

c) care for the client in the same manner that you did while he was alive Standard precautions are used on all clients, including those diagnosed with HIV, after they have expired. The virus remains infectious in all bodily fluids.

Of the following positions, which is most appropriate for pelvic examination? a. Prone b. Trendelenburg c. Lithotomy d. Semi-Fowlers

c. Lithotomy The lithotomy position is used during pelvic examinations, to provide adequate visualization of the internal female reproductive organs.

Working on another's behalf to represent his concerns as a moral agent is referred to as which of the following? a. beneficence b. nursing practice c. advocacy d. ethical practice

c. advocacy

Prior to delegating a task, the nurse should first: a. be certain the delegates can legally carry out the task b. attempt to complete the task on his or her own c. be certain the task can be delegated d. call the physician for an order to permit delegation

c. be certain the task can be delegated

A patient with a history of alcohol abuse is arrested for driving under the influence. His wife bails him out of jail for the third time. His wife's response is an example of: a. attachement disorder b. reactivity c. codependency d. addiction

c. codependency codependency is a type of dysfunction in which one individual supports the addiction, substance abuse, immaturity, or other poor behavior of another in a relationship.

Asepsis is a term that refers to which of the following? a. clean b. dirty c. sterile d. infectious

c. sterile

You are educating a newly diagnosed 55 year old male with type 2 diabetes. Which of the following will you tell your patient is most effective in attaining normal or near normal blood glucose levels? a. quitting smoking b. controlling hypertension c. weight loss d. exercising

c. weight loss Weight loss has been shown to help lower and maintain blood glucose levels. In addition to improving general health, all these measure will decrease the risk of developing cardiovascular disease

Which significant risk factors for coronary heart disease carry a greater risk for women than for men? a) Obesity b) Smoking c) Hypertension d) Diabetes mellitus e) Low levels of high-density lipoprotein (HDL) cholesterol

d) Diabetes mellitus e) Low levels of high-density lipoprotein (HDL) cholesterol Diabetes is twice as strong a predictor of coronary heart disease in women as in men; diabetes cancels the cardiac protection that estrogen provides to premenopausal women. A low level of HDL-C (less than 35 mg/dL) has a greater bearing on coronary heart disease in women than in men and is the most important lipid factor in women; the significance of this is unclear. Obesity, smoking, and hypertension are risk factors common to both women and men.

You listen to report and learn that your patient requires a full lift with 2-person assist to get out of bed. Your patient rings and states that she urgently needs to go to the bathroom. No one is available to help you. Choose the best course of action. a) ignore the patient until someone is available to assist you b) use the lift by yourself because the patient states he cannot wait c) pull the patient on a bedpan d) explain to the patient that no one is available to assist, and ask for their patience

d) explain to the patient that no one is available to assist, and ask for their patience

The nurse is reviewing the assessment finding for a patient hospitalized with a stress disorder. What findings support the diagnoses? a. A vague feeling of depression b. An assumed role to protect the ego c. A main reason for all mental illnesses d. A response to any demand made upon the individual

d. A response to any demand made upon the individual

A set code of ethics for LPNs has been published by: a. American Medical Association b. American Nurses Association c. National Federation of LPNs d. National Association for Practical Nurse Education and Service

d. National Association for Practical Nurse Education and Service Within the code are principles that serve as a guide for LPNs when providing care to patients. NAPNEs also publishes a journal called the "Journal of Practical Nursing"

A nurse has performed a non stress test on a pregnant client and is reviewing the fetal monitoring strip. The nurse interprets the test as reactive and understands that this indicates which of the following? a. the need for further evaluation b. abnormal findings c. that the findings on the monitor were difficult to interpret d. normal findings

d. normal findings A reactive non stress test is a normal test. To be considered reactive, the baseline fetal heart rate must be within normal range (120-160 BPM) with good long term viability.

What does a t-wave represent?

repolarization of the ventricles

incompetent cervix

spontaneous, premature dilation of the cervix during the second trimester of pregnancy

A 36 year old woman has just discovered she is pregnant. The pregnancy is unplanned and she feels ambivalent about it. She expresses to you that she feels guilty about not feeling more excited about the baby. What would be your best response?

"It's normal to feel ambivalent at the beginning of a pregnancy"

The nurse is caring for a patient with severe hyperkalemia. What actions should be included in the plan of care for the shift? (Select all that apply.) 1. Monitor serum potassium levels. 2. Administer Kayexalate as prescribed by the health care provider. 3. Report a urinary output less than 30 mL/hr. 4. Monitor vital signs every 12 hours. 5. Restrict fluid intake.

1. Monitor serum potassium levels. 2. Administer Kayexalate as prescribed by the health care provider. 3. Report a urinary output less than 30 mL/hr.

molar pregnancy

1. Nucleus of the egg is impaired • it tries to produce a baby, but there is not enough genetic material. 2. Risk factors: infertility treatment

The patient's potassium level is 5.2. When the nurse discusses dietary selections, the patient should be instructed to limit intake of what food items? (Select all that apply.) 1. Orange juice 2. Bananas 3. Apples 4. Tomatoes 5. Red meat

1. Orange juice 2. Bananas 4. Tomatoes

Hyponatremia

1. Cells swell 2. Types of hyponatremia: evolemic (increase in H2O, Na+ stays the same). Sodium becomes diluted. * causes SIADH syndrome * hypovolemoic hyponatremia: dehydration "NO Na+" mnemonic causes: Na+ - excretion increase with renal problems O - overload of fluids (CHF, hypotonic solution) Na+ - low Na+ diet A - ADH

Which abbreviation is used to indicate hydrogen ion concentration in the body? 1. mEq 2. ATP 3. pH 4. mL

3. pH

Normal albumin lab value:

3.3-5.2

The nurse is caring for a 6-month-old infant with Down syndrome. Which instructions does the nurse include while repeating the feeding techniques and diet details of the infant to the parents? Select all that apply. A) "Provide the child with a fiber-rich diet." B) "Use a small spoon with a straight handle." C) "Stop feeding if the child spits the food out." D) "Never feed the child if the tongue protrudes out." E) "Provide the child with an adequate amount of fluids."

A) "Provide the child with a fiber-rich diet." B) "Use a small spoon with a straight handle." E) "Provide the child with an adequate amount of fluids." The nurse should instruct the parent to provide the infant with a fiber-rich diet, because constipation is a symptom seen in children with Down syndrome. The protruding tongue of the infant may obstruct swallowing. Therefore, the nurse should instruct the parent to use a small spoon with a straight handle to feed the infant. An adequate amount of fluid is given to the infant to decrease the risk of constipation. Thrusting the food out of the mouth is a common observation in children with Down syndrome. The parents should be instructed to feed the infant even if the infant spits the food out of the mouth, in effort to meet the nutritional requirements. The parents are instructed to feed the infant despite the tongue protrusion.

The most common complication of hemodialysis is: a) hypertension b) hypotension c) nausea and vomiting d) general weakness

Hypotension Hypotension occurs most frequently, affecting 20-30% of patients at some point. It may be caused by removing too much weight (i.e. due to inaccurate pre-dialysis weight), heart disease, septicemia, or taking blood pressure medications prior to dialysis.

The purpose of inserting a chest tube is to:

Restore negatie pressure int he intrapleural space. Insertion of a chest tube is an invasive procedure designed to restore negative pressure in the intrapleural space. When the normally negative pressure of the intrapleural space is disrupted, it causes the lung to collapse and a patient to develop respiratory symptoms. Therefore, the tube is placed to restore negative pressure until the underlying condition can heal. Conditions that commonly necessitate a chest tube include a pneumothorax, blunt chest trauma, empyema, or hemothorax

Sinus bradycardia

Slow rhythm originating in the SA node. Rate: < 60bpm Managed by insertion of a pacemaker. Causes: OSP, vomiting, intracranial tumors, MI, meds (overuse of digitalis, beta-adrenergic blockers, calcium channel blockers), ICP, hypothermia, vagal stimulation, carotid sinus massage, endocrine disturbances. S/S: fatigue, lightheadedness, syncope. Med. management: atropine to increase HR.

A client with cancer of the thyroid is scheduled for a thyroidectomy. Postoperatively the nurse plans to have a: Multiple choice question Quiet, dimly lit room for the client Tracheostomy set at the client's bedside Large soft pillow for use under the client's head Suction machine set on intermittent suction at the client's bedside

Tracheostomy set at the client's bedside A tracheostomy set should be available in the event there is excessive edema at the surgical site, which can cause tracheal compression. A quiet, dimly lit room for the client is not necessary after a thyroidectomy. The head should be kept in anatomical alignment, the neck not flexed or hyperextended; a small soft pillow can be used to accomplish alignment. Intermittent suction does not provide the constant suction needed to clear the airway.

The nurse is assessing a patient with a respiratory problem. Which questions are likely to be included in the assessment? Select all that apply. a) "Does your cough produce sputum?" b) "What medications are you taking?" c) "Do you have any shortness of breath?" d) "Do you often have long working hours?" e) "Do you have any neurological conditions?"

a) "Does your cough produce sputum?" b) "What medications are you taking?" c) "Do you have any shortness of breath?" When the nurse is assessing a patient for respiratory problems the nurse asks the patient about cough and sputum to assess the nature of cough. The nurse asks about drug history to understand what medications were prescribed earlier. The nurse also asks the patient if the patient feels shortness of breath to assess the risk for hypoxia. This also helps to understand what medical problems the patient has. Long working hours is a factor contributing to stress and is not a relevant question in this case. Asking about neurological conditions is not required to assess respiratory issues.

You are admitting a new patient onto the unit. The patient tells you that he is allergic to penicillin. In addition to the name of the medication, what is MOST IMPORTANT for the nurse to know about the patient's allergy? a) the type and severity of reaction experienced b) any other allergies to medications c)any history of treatment for this reaction

a) the type and severity of reaction experienced Understanding the type of reaction experienced is very important. For example, a patient might state that he is allergic to codeine because it causes an upset stomach. This is a side effect and not a true allergic reaction. On the other hand, a patient might experience an anaphylactic reaction to a medication. This is a severe reaction which could result in death.

The doctor has ordered a NSAID in combination with oxycodone for severe pain. Why is the NSAID being given? a) to act as an adjuvant b) to prevent adverse reactions c) to inhibit side effects d) to act as an antagonist

a) to act as an adjuvant A non-opioid drug, such as an NSAID, is given with narcotic analgesics as an adjuvant medication that potentiates and heightens the effects of the narcotic medications so that a lower dosage can be used for adequate pain relief.

The nurse is caring for a patient who is currently voicing feelings of anxiety. The nurse correctly recognizes what as the best description of the feelings that the patient is experiencing? a. A vague feeling of apprehension b. Feelings of paranoia c. Concerns about the impressions others have for her d. Emotional stability

a. A vague feeling of apprehension

What action should an LPN take when a child with second degree burns is admitted to the unit? a) cohort the child with a child of the same age who has simple acute diarrhea b) cohort the child with an older client with asthma c) initiate strict isolation d) initiate seizure precautions

b) cohort the child with an older client with asthma Children and adults with second and third degree burns are at great risk of infection. This child should share a room with another child who does not have an infectious, or potentially infectious, disease, such as that associated with acute diarrhea. There is no need for strict isolation.

Your patient is taking a beta blocker (metroprolol), 50 mg po bid. You are obtaining vital signs and note that the patient's pulse is 54 bpm. You should: a) Notify the physician b) continue to monitor the patient, obtaining vital signs as ordered c) hold the next dose of metroprolol d) administer epinephrine to increase the patient's heart rate

b) continue to monitor the patient, obtaining vital signs as ordered Bradycardia is a normal side effect of beta blockers. Providing the patient is asymptomatic, continue to monitor the patient's vital signs. There is no need to hold the next dose unless the patient is symptomatic or the heart rate decreases further.

A 15 year old male is admitted to the psychiatric unit with anorexia nervosa. Upon returning form lunch, the LPN caring for him finds him running in place in his room by himself. Which of the following is MOST appropriate action by the LPN? a) ask the patient why he is running in place in his room b) interrupt his running and offer to escort him to a group session c) allow him to continue his running routine d) notify his physician of the running activity

b) interrupt his running and offer to escort him to a group session

Any process that interferes with normal production or excretion of hydrogen ions poses the risk of causing (SELECT ALL THAT APPLY): a) respiratory acidosis b) metabolic alkalosis c) respiratory alkalosis d) metabolic acidosis

b) metabolic alkalosis d) metabolic acidosis

What is the most appropriate nursing intervention for a patient requiring finger probe pulse oximetry? a) apply a sensor probe over a finger and cover lightly with a gauze to prevent skin breakdown. b) set alarms on the oriented to at least 100%. c) identify if the patient has had a recent diagnostic test using intravenous dye. d) remove the sensor between oxygen saturation readings.

b) set alarms on the oriented to at least 100%.

A physician orders a continuous heparin infusion for a patient with a blood clot. The order reads: "administer heparin 16 units/hour/ kg IV infusion." The patient weighs 55 kg and the heparin bag comes in a 500 mL bag with 25,000 units of medication. The infusion pump should be set at: a. 16.1 mL/hr b. 17.6 mL/hr c. 24.3 mL/hr d. 10.8 mL/hr

b. 17.6 mL/hr To complete the dosage calculation, one would proceed with the following calculations: 16 units x 55 kg = 880 units/hour (25,000 units)/(500 mL) = (880 units/hour)/(x mL/hr) (25,000 units) x (x mL/hr) = 440,00 x mL/hr = 440,000/25,000 units x = 17.6 mL/hr

The physician informs a spouse that his wife is terminally ill. The spouse punches the doctor. This is an example of: a. Denial b. Acting out c. Sublimation d. Projection

b. Acting out Acting out involves performing an extreme behavior in order to express feelings or thoughts the person feels unable to express otherwise. Denial is complete rejection of a feeling or thought. Sublimation is directing one's feelings into a socially productive activity. Projection is believing that someone else has the same feeling or thought as you.

Punishment and abandonment were how mentally ill people were treated in medieval times. These practices continued until the 17th and 18th centuries. Which care practice that is still being used today did Dr. Phillipe Pinel of France advocate? a. Electroshock therapy for melancholy b. Humane care with record keeping of behaviors c. Psychoanalysis d. Home care in the community

b. Humane care with record keeping of behaviors

A 14-year-old tells the school nurse that she is self-conscious about her recent breast development. She reports that the boys in her class are teasing her. What is the first step for the nurse to take? a. Call her parents b. Have her describe what happened c. Ask who her friends are d. Provide her with a pamphlet outlining the changes associated with puberty

b. Have her describe what happened

Your client has a fracture of the radius. There is swelling at the injury site and the client complains of pain in the area. Which stage of bone healing do these signs and symptoms represent? a. Cellular proliferation b. Inflammatory c. Ossification d. Callus formation

b. Inflammatory During the inflammatory phase, bleeding occurs at the area of injury and results in a hematoma. Why plus I was rushed to the area of injury to begin to debride the dead cells. The patient will experience pain during this phase. Cellular proliferation occurs after approximately 5 days, when the interrupted blood supply is recreated and fibrin strands begin to form. The callus formation stage takes about 3 to 4 weeks, while the ossification stage may take as long as 3 to 4 months. Remodeling of the bone may take months to years.

What is the best description of personality? a. The level of mental health that a person attains in life b. The relatively consistent set of attitudes and behaviors particular to an individual c. The result of a positive self-concept and acceptable behavior d. The ability to manage stress

b. The relatively consistent set of attitudes and behaviors particular to an individual

Who was the English Quaker who advocated humane care and built an asylum to reflect a household? a. Florence Nightingale b. William Tukes c. Sigmund Freud d. Benjamin Rush

b. William Tukes

A nurse is teaching a client who had a myocardial infarction about the prescribed 1500-calorie, 2-gram-sodium, weight-reducing diet. Which low-sodium, low-calorie nutrients should the nurse recommend that the client include in the diet? a) Lean steak b) Celery sticks c) Baked chicken d) Tuna fish salad e) Mashed potatoes

c) Baked chicken e) Mashed potatoes Baked chicken is low in calories and sodium. Mashed potatoes are low in sodium and calories. Beef is high in calories. Celery sticks are high in sodium. Canned tuna fish is high in sodium.

In what situation should a nurse anticipate that a client will experience a phobic reaction? a) When seeking attention from others b) When thinking about the feared object c) When coming into contact with the feared object d) When being exposed to an unfamiliar environment

c) When coming into contact with the feared object With phobias, the individual transfers anxiety to a safer inanimate object or situation. Therefore the anxiety and resulting feelings will be precipitated only when the client is in direct contact with the object or situation. Phobias are severe anxiety reactions, not attention-seeking actions. It is not thinking about the feared object that causes anxiety; it is the possibility of having to come into contact with it. It is the presence of the phobic object or situation that triggers the anxiety, not the unfamiliarity of the environment.

Changes in the delivery of mental health care that resulted from the development of electroconvulsive therapy and psychotherapeutic drugs brought about which phenomenon in the 20th century? a. Behavioral therapy b. Personality disorganization c. Deinstitutionalization d. Brain surgery

c. Deinstitutionalization

An assembly line manager in a factory was told that he would be laid off if his line did not meet the hourly quote. He promptly went to his workers and threatened to fire anyone who was found taking even 1 minute extra on a break. What is the manager displaying? a. Denial b. Regression c. Displacement d. Identification

c. Displacement

Your patient is allergic to peanuts. You are teaching him to use an Epiphyseal pen. Choose the correct information regarding Epi pens: a. 3 doses of adrenalin should be given one after the other while the patient is en route to the hospital b. side effects of epinephrine administration are rare c. Epi pens are not curative-patients must proceed to the nearest emergency room after injecting themselves using an Epiphyseal pen d. slower absorption is achieved via the IM route

c. Epi pens are not curative-patients must proceed to the nearest emergency room after injecting themselves using an Epiphyseal pen Use of an Epi pen buys time for the patient to get to the emergency room. Other treatments are often needed in conjunction with epinephrine/adrenaline. Symptoms may return when the epinephrine wears off. Rapid absorption is achieved through the IM route. Patients should auto inject one dose before calling the ambulance. Sid effects are common and may include headache, dizziness, tachycardia, and anxiety.

In what age group should you perform the EENT (ear, eyes, nose and throat) LAST? a) babies b) toddlers c) preschoolers d) all of the above

d) all of the above Babies, toddlers and preschoolers often become very upset during and EENT exam; therefore, you should save this option of the assessment for last. If you perform the EENT exam first, the patient may cry and may be uncooperative, making the rest of the exam very difficult.

The mother of an 8-year-old tell the clinic nurse that she is concerned about the child because the child seems more attentive to friends than anything else. Using Erikson's psychological development theory, the appropriate nursing response is which of the following?

"At this age the child is developing his own personality." According to Erickson, during school-age years (6 to 12), the child begins to move toward peers and friends and away from parents for support. The child also begins to develop special interests that reflect his or her own developing personality instead of those of the parents.

An older adult comes for an annual physical and tells the nurse, "I had three respiratory infections last year. How can I prevent this from happening again?" What is the nurse's best response? Multiple choice question "Stay away from preschool and school-age children." "Avoid putting your hands near your nose and mouth." "Wear a sweater under your coat when going outside in cold weather." "Take an aspirin when you think you may be coming down with a cold."

"Avoid putting your hands near your nose and mouth." Transmission of microorganisms via the hands is one of the most common ways pathogens are transmitted from one person to another. Avoiding putting hands near the nose and mouth interrupts the chain of infection at the portal of entry phase. Staying away from preschool and school-age children is unnecessary and could cause social isolation. However, exposure to these children when they have an active infection should be avoided if possible. Precautions can be taken when around children (e.g., washing the hands, avoiding exposure to nasal and oral secretions). Wearing a sweater under the coat when going outside in cold weather will not limit the exposure to pathogenic microorganisms. However, it may make the person more comfortable because older people have less subcutaneous fat and can be more sensitive to cold environmental temperatures. Colds are caused by viruses; an aspirin will not eliminate these microorganisms. In addition, it is not within the role of a nurse to prescribe medications, even if they are over-the-counter medications.

Which statement by an older adult most strongly supports the nurse's conclusion that the client is impacted with stool? Multiple choice question "I have a lot of gas pains." "I don't have much of an appetite." "I feel like I have to go and just can't." "I haven't had a bowel movement for several days."

"I feel like I have to go and just can't A client with a fecal impaction has the urge to defecate but is unable to do so. Flatulence may occur as a result of immobility, not just obstruction. Anorexia may occur with an impaction but also may be caused by other conditions. The frequency of bowel movements varies for individuals; it may be normal for this individual not to have a bowel movement for several days.

A client with painful swelling of a distal joint of the ring finger is found to be in the early stages of rheumatoid arthritis (RA). A test for the rheumatoid factor is negative. The client asks about the reliability of the test, stating, "I don't think the result is accurate. I have been diagnosed with RA, and I am in so much pain." How should the nurse respond? (Multiple choice question) "It might help if you try not to think about your discomfort." "Don't let that upset you; eventually the tests will be positive." "These tests will have to be repeated; they are complicated tests." "Laboratory tests often are negative in the early stages of the disease."

"Laboratory tests often are negative in the early stages of the disease." The antibody called rheumatoid factor is present in 90% of advanced cases of arthritis; it frequently is not found in the early stages of the disease. The response "It might help if you try not to think about your discomfort" denies the client's discomfort and does not address the stated confusion. The response "Don't let that upset you; eventually the tests will be positive" denies the client's immediate feelings and blocks further communication of feelings. The response "These tests will have to be repeated; they are complicated tests" reinforces the client's confusion over negative test results and feeling discomfort.

A client with multiple sclerosis is informed that this is a chronic, progressive neurological condition. The client asks the nurse, "Will I experience excruciating pain?" What is the nurse's best response? Multiple choice question "Tell me about your fears regarding pain." "Analgesics will be prescribed to control the pain." "Pain is not a characteristic symptom of this disease process." "Let's make a list of the things you need to ask your health care provider."

"Pain is not a characteristic symptom of this disease process." The response "Pain is not a characteristic symptom of this disease process" is a truthful answer that provides hope for the client. Although neuropathic pain may sometimes occur, it is not a characteristic manifestation of multiple sclerosis . These clients more typically have diminished sensitivity to pain and paresthesias (e.g., tingling, burning, crawling sensations). The response "Tell me about your fears regarding pain" avoids the client's question and may increase anxiety. Analgesics are not commonly prescribed unless pain results from some other condition. The response "Let's make a list of the things you need to ask your health care provider" avoids the client's question and abdicates the nurse's responsibility.

Your patient is scheduled to undergo a radical mastectomy due to breast cancer. You are teaching the patient the recommended post-op exercises. The client asks you why the exercises are necessary during healing and states that she is afraid the exercises will be painful. What is the CORRECT rationale for performing arm exercises after a mastectomy?

"The exercises help you to regain the use of your arms, which will be affected by the surgery, and also prevent your shoulder from becoming stiff and painful."

Before a transurethral resection of the prostate (TURP), a client asks about what to expect postoperatively. The most appropriate response by the nurse is: Multiple choice question "You will have an abdominal incision and a dressing." "Your urine will be pink and free of clots." "There will be an incision between your scrotum and rectum." "There will be an indwelling urinary catheter and a continuous bladder irrigation in place."

"There will be an indwelling urinary catheter and a continuous bladder irrigation in place." The presence of an indwelling urinary catheter and a continuous bladder irrigation are routine postoperative expectations after a TURP; they provide for hemostasis and urinary excretion. An abdominal incision and dressing are present with a suprapubic, not transurethral, prostatectomy. After a TURP the client initially can expect hematuria and some blood clots; the continuous bladder irrigation keeps the bladder free of clots and the catheter patent. An incision between the scrotum and rectum is associated with a perineal prostatectomy, not a TURP.

Your patient has genital warts and has just learned that she is pregnant. She has been applying podofilox topically. She asks you whether it is safe to continue this treatment. What is the best response?

"This medication is not safe to use during pregnancy." Podofilox may induce abortion. It is not available in an oral form. Cryotherapy, laser surgery or other treatments may be used to treat genital warts.

Normal INR (internationalize ratio) lab value:

0.8-1.1

A licensed practical nurse is caring for a two-year-old boy on the pediatrics unit who underwent surgical repair of an arm fracture. The physician orders pain medication and the order reads as follows: Give morphine sulfate 0.1 mg/kilogram IV every 4 to 6 hours as needed for pain. The patient weighs 10 kg. The LPN administers:

1 mg To calculate the dosage for the pediatric patient, the following calculation would be done: 10 kg X 0.5 mg = 1 mg

Intracellular fluid compartment

1. 66% of body fluids 2. Contains dissolved particles called "solutes"

Molar pregnancy clinical manifestations

1. Anemia 2. Hyperemesis • HCG levels are elevated • causes a positive pregnancy reading 3. Bleeding 4. Gestational HTN prior to 20 weeks.

The nurse notes accelerations with fetal movement. The nurse correctly recognizes that heart accelerations most commonly: 1. Are reassuring. 2. Are caused by umbilical cord compression. 3. Warrant close observation. 4. Are caused by uteroplacental insufficiency.

1. Are reassuring.

After admitting a new patient to the maternity unit, the nurse writes a care plan. This process of determining outcomes and interventions is which stage of the nursing process? 1. Assessment 2. Planning 3. Implementation 4. Evaluation

1. Assessment

What is SIADH (syndrome of inappropriate anti-diuretic hormone)

1. Complecation: cerebral edema, pulmonary edema, hyponatremia 2. Fluid restriction 3.Urinary catheter avoided do to risk of UTI

Risks for multi fetal pregnancies

1. Congenital anomalies 2. Growth problems: One fetus grows at the expense of the other, occurs when they share the same placenta. 3. Spontaneous abortion 4. Anemia 5. Gestational HTN 6. Placenta prebia & abruptio placenta

Extracellular fluid

1. Contains O2 and CO2, glucose, amino acids, fatty acids, Na+, Ca+, Chloride, bicarbonate. 2. Interstitial - between the cells or in the tissues. * lymph, CSF, GI secretions 3. Intravascular - plasma within the vessels * serum & protein * carries nutrients & waste products

Anti-infectives: Antitubercular 1st line of drugs include:

1. Isoniazid (Laniazid, INH) 2. Rifampin (Rifadin) 3. Pyrazinamide 4. Ethambutol (Myombutol)

Isoniazid (Laniazid, INH)

1. MOST COMMON treatment for TB 2. Used for prophylaxis 3. Metabolized in the liver 4. Side effects: peripheral neuropathy, hepatotoxicity, low glucose levels

Cardiac catheterization

1. Measure blood B/P within the heart 2. Assesses blood in the heart Performed under sterile surgical conditions. Informed consent must be obtained. Check for allergies, and assess peripheral pulses: color, and sensation Q15 min X 1 hr. Check for abnormal HR, dysrhythmias, PE (indicated by respiratory difficulty).

What are the effects of Na+, K+, Ca+, and Mg+ on the heart?

1. Na+ - helps maintain fluid balance. 2. K+ - required for relaxation of the cardiac muscle. 3. Ca+ - necessary for contractions of cardiac muscle. 4. Mg+ - helps maintain correct level of electric excitability in the nerves and muscles.

Hirschsprung disease

1. No peristalsis because there are no ganglion cells in a portion of a colon. 2. S/S: failure to pass me onion within 48 hours. 3. Treatment: Surgical removal of the affected portion of the bowel. 4. Nursing intervention: Low fiber diet, high protein, high calorie diet.

Chroni's disease

1. No specific diet 2. Avoid hard to digest foods during an acute flare 3. Eat a well balanced diet

Passive transport includes what physiologic processes? (Select all that apply.) 1. Osmosis 2. Diffusion 3. Filtration 4. Sodium-potassium pump 5. Compensatory metabolic acidosis

1. Osmosis 2. Diffusion 3. Filtration

Ruptured fallopian tube

1. Pain that subsides suddenly (either on the left/right side). 2. Growth in ABD that is rigid do to bleeding of ruptured fallopian tube.

The nurse's role during a diagnostic imaging procedure is to?

1. Prepare and explain the procedure to the patient. 2. Check for any allergies to the ingredients in the dye. 3. Obtain informed consent. 4. Check the site of the catheter insertion for excess bleeding. 5. Alert the health care provider if excess bleeding is noted. 6. Check circulation in the peripherally below the site. 7. Monitor V/S and place the patient supine.

When planning a diet with a pregnant woman. What should the nurse's initial action be? 1. Review the woman's dietary intake. 2. Teach the woman about the ChooseMyPlate guide. 3. Caution the woman to avoid large doses of vitamins, especially those that are fat soluble. 4. Instruct the woman to limit the intake of fatty foods.

1. Review the woman's dietary intake.

Hemophilia

1. S/S: bone deformities, contractures, crippling, hematoma in spinal cord causes paralysis. * Genetic * No aspirin should be given to the patient * IV infusion: factor 8 & 9 = clotting factor replace

D-dimer

1. Serum blood test * shows excess presence of clots. 2. Elevated D-diner * Shows a clot somewhere in the body where it does not belong.

The nurse is preparing a patient for a scheduled cesarean section. The nurse will administer medications to reduce gastric acidity. Which medications may be used? (Select all that apply.) 1. Sodium citrate (Bicitra) 2. Ranitidine (Zantac) 3. Cimetidine (Tagamet) 4. Famotidine (Pepcid) 5. Glycopyrrolate (Robinul)

1. Sodium citrate (Bicitra) 2. Ranitidine (Zantac) 3. Cimetidine (Tagamet) 4. Famotidine (Pepcid)

Normal magnesium lab value:

1.3-2.1

Normal ammonia lab value:

10-50

Normal BUN (blood urea nitrogen) lab value:

11-23

A "multiparous" woman is considered to have had:

2 or more childbirths pasts 20 weeks gestation. The root "para" is used to describe births past 20 weeks gestation. The prefix "multi" is used to describe many. Therefore, "multiparous" is the term used in obstetrics to refer to a woman who has had more than 2 births past 20 weeks gestation.

Which regulatory system is the body's SECOND line of defense in keeping the pH within normal limits? 1. Blood buffers 2. Respiratory system 3. Renal system 4. Blood pressure

2. Respiratory system

The nurse explains to the patient that she should contact her health care provider if she experiences any of the danger signs of pregnancy. Which symptom is a danger sign during pregnancy? 1. Urinary frequency 2. Severe headaches 3. Backache 4. Heartburn

2. Severe headaches

To assess the frequency of regular labor contractions, what should the nurse record? 1. The interval between the peaks of the contractions 2. The start of one contraction to the start of the next 3. The end of one contraction to the start of the next 4. How many contractions she has in 15 minutes.

2. The start of one contraction to the start of the next

A pregnant woman is attending childbirth classes. She asks the nurse teaching the class when she would have an internal fetal monitor applied. What is the best nursing response? 1. "Because you have had a low-risk pregnancy, you are not considered a candidate for internal monitoring." 2. "Your health care provider decides when to apply the internal monitor." 3. "Your cervix must be 2 to 3 cm dilated and your membranes ruptured before an internal monitor can be applied." 4. "We can apply the internal monitor at any time after your membranes rupture."

3. "Your cervix must be 2 to 3 cm dilated and your membranes ruptured before an internal monitor can be applied."

A 23-year-old primigravida arrives at the labor unit in early labor. Which assessment finding indicates that labor has begun? 1. Decreased vaginal secretions 2. Weight gain of 1 to 3 lb 3. Bloody show 4. Increased fetal movement

3. Bloody show

A woman has progressed through her labor without difficulty. However, the fetal heart rate has been decreasing with each contraction for the past 15 minutes. The rate decreases from 150 to 125 bpm after the peak of the contraction and returns to 150 bpm 15 seconds after the contraction is finished. What phenomena do the clinical manifestations most support? 1. Variable decelerations. 2. Early decelerations. 3. Late decelerations. 4. Combination decelerations.

3. Late decelerations.

A patient is scheduled for an ultrasound scan. She is at 22 weeks gestation. The patient asks how they will be able to tell the gestational age of her fetus. When planning the response, the nurse correctly recognizes that which measurements will be used in this determination? (Select all that apply.) 1. Abdominal circumference 2. Biparietal diameter 3. Quantity of amniotic fluid 4. Femur length 5. Crown rump length

3. Quantity of amniotic fluid 4. Femur length

A pregnant patient at her third office visit asks the nurse, "What can I do when my leg goes into a cramp?" The patient demonstrates understanding of the nurse's instruction regarding relief of leg cramps if she: 1. wiggles and points her toes during the cramp. 2. applies cold compresses to the affected leg. 3. extends her leg and dorsiflexes her foot during the cramp. 4. avoids weight bearing on the affected leg during the cramp.

3. extends her leg and dorsiflexes her foot during the cramp.

The doctor has ordered 1000 cc of 0.9% sodium chloride over 8 hours. How many drops per minute would you administer if you are using an IV set that delivers 15 drops per cc?

31 gets/min The answer is calculated as follows: 1000/8 = 125 cc per hour; 125/4 = 31.25 cc, which is rounded to 31 gets per minute.

To determine fetal lie, presentation, and position, the caregiver uses which assessment technique? 1. Abdominal ultrasound scan 2. Fetal heart tone auscultation 3. Palpation of contractions 4. Leopold's maneuvers

4. Leopold's maneuvers

The movement of water from an area of lower concentration to an area of higher concentration occurs through which of the following? 1. Diffusion 2. Filtration 3. Active transport 4. Osmosis

4. Osmosis

When obtaining a reproductive health history from a female patient. What should the nurse do? 1. Limit the time spent on exploration of intimate topics. 2. Explain the purpose of questions asked and how they will be used. 3. Avoid asking questions that may embarrass the patient. 4. Use only acceptable medical terminology when referring to body parts and functions.

4. Use only acceptable medical terminology when referring to body parts and functions.

Which client is considered at the highest risk for a dissecting aneurysm? Multiple choice question 50-year-old white male with moderate hypertension 55-year-old black male with uncontrolled hypertension 40-year-old white female with uncontrolled hypertension 42-year-old black female with peripheral vascular disease

55-year-old black male with uncontrolled hypertension The highest incidence of dissecting aneurysm is in people 40 to 70 years of age; it is seen four times more frequently in men than women, with a higher incidence in black men. It occurs most often in older clients with hypertension. Unless the 40-year-old female is pregnant or in labor, she is not at as great a risk as a black male. The 50-year-old white male with moderate hypertension and the 42-year-old black female with peripheral vascular disease are not at as high a risk as a black male.

Normal iron lab value:

75-175

Normal Ca+ (calcium) lab value:

8.5-10.5

The nurse is caring for a child who has acute laryngotracheobronchitis (LTB). What does the primary health care provider instruct the nurse during an emergency? A) Administer aerosolized epinephrine. B) Offer adequate oral fluids to the child. C) Give sedatives to the child to induce sleep. D) Administer betamethasone (Celestone) to the child.

A) Administer aerosolized epinephrine. LTB is managed by maintaining the patency of airway. Therefore, the primary health care provider would instruct the nurse to administer aerosolized epinephrine to the child. This helps to decrease airway edema by vasoconstriction and improve oxygenation by bronchodilation. Children with acute LTB are maintained in NPO status (nothing by mouth) since typically rapid respirations may cause aspiration. Sedatives impair the level of consciousness and make it difficult to determine hypoxia, hence should be avoided in children with LTB. Betamethasone (Celestone) is a corticosteroid given to pregnant women before delivery to increase the production of surfactant in the premature infant.

Patient teaching after a tonsillectomy and adenoidenctomy would include which instruction(s)? (SELECT ALL THAT APPLY). A) Avoid attempting to clear the throat, coughing, and sneezing. B) Avoid vigorous nose blowing for 1 to 2 weeks. C) Resume foods and fluids as tolerated. D) Take aspirin, gr 10 (10 grains), every 4 hours. E) Notify the health care provider Incase of increased pain, fever, or bleeding

A) Avoid attempting to clear the throat, coughing, and sneezing. E) Notify the health care provider Incase of increased pain, fever, or bleeding

A child with diabetes is brought to the emergency room with dizziness. The nurse learns that the child had forgotten a dose of insulin. Which findings indicate that the child has ketoacidosis? Select all that apply. A) Blood pH of 7.25 B) Presence of casts in urine C) Absence of ketones in blood D) Presence of glucose in urine E) Blood sugar level of 380 mg/dL

A) Blood pH of 7.25 D) Presence of glucose in urine E) Blood sugar level of 380 mg/dL Diabetic ketoacidosis is a condition where the body uses ketones as an alternate form of energy in the absence of insulin. As a result, the ketone bodies increase in the blood, decreasing the pH; a pH less than 7.30 indicates metabolic acidosis caused by ketones. Due to absence of insulin, the blood glucose levels increase. Blood glucose levels greater than 330 mg/dL indicate ketoacidosis. The increased glucose in the blood spills to urine because the kidneys are unable to filter it out. As a result, the urine may contain variable amounts of glucose. Presence of casts in urine indicates urinary tract infection; it is unrelated to ketoacidosis. In ketoacidosis, the ketone levels of blood are increased.

Which patient report indicates that phenazopyridine hydrochloride (pyridium) is being effected? A) Decreased bladder spasms B) Decrease in burning sensation C) Increased urinary output D) Increased pain tolerance

A) Decreased bladder spasms

A child has iron deficiency anemia. Characteristics of this disease process may include which? Select all that apply. A) Fatigue B) Bradycardia C) Weight gain D) Spoon fingernails E) Pale mucous membranes

A) Fatigue D) Spoon fingernails E) Pale mucous membranes Iron deficiency anemia is characterized by numerous signs and symptoms, depending on the severity of the anemia. Children with mild to moderate anemia may demonstrate irritability, fatigue, and decreased play activity. If the anemia is severe, the child may have anorexia (not weight gain), pallor, concave or spoon fingernails, tachycardia (not bradycardia), and pale mucous membranes.

A worried parent reports, "My child doesn't play with other children, and does not eat and sleep properly these days." Which medication does the nurse expect the primary health care provider (PHP) to prescribe? A) Fluoxetine (Prozac) B) Octreotide (Sandostatin) C) Acetaminophen (Tylenol) D) Erythromycin (Erythrocin)

A) Fluoxetine (Prozac) Symptoms, such as eating and sleeping disturbances and lack of interest in play, indicates that the child is depressed. The child should be treated with antidepressants and psychological therapies. Fluoxetine (Prozac) is a tricyclic antidepressant used to treat depression. Administering growth hormones, such as octreotide (Sandostatin), does not reduce depression. Acetaminophen (Tylenol) is given to treat fever and pain. Erythromycin (Erythrocin) is an antibiotic used to treat pertussis (whooping cough); it is not used to treat depression.

A nurse is caring for a 4-year-old child with abdominal pain related to an intestinal obstruction. What is the most common cause of intestinal obstruction in a child this age? A) Intussusception B) Inguinal hernias C) Hirschsprung disease D) Hypertrophic pyloric stenosis

A) Intussusception Intussusception is the result of one portion of the intestine telescoping into another. It occurs most commonly in a child between the ages of 3 months and 6 years. Hirschsprung disease is a functional intestinal obstruction and is commonly seen in children with trisomy 21. Hernia is the protrusion of organs or portions of an organ through a structural defect or weakened muscle. Hypertrophic pyloric stenosis is an obstruction in which the gastric outlet is mechanically blocked by a congenitally hypertrophied pyloric muscle. It is most commonly seen in infants younger than age 6 months.

Which medication is the nurse likely to find to be prescribed for a patient who is diagnosed with human immunodeficiency virus (HIV) infection? A) Lamivudine (Epivir) B) Palivizumab (Synagis) C) Naproxen (Naprosyn) D) Sulfasalazine (Azulfidine)

A) Lamivudine (Epivir) Antiretroviral drugs such as lamivudine (Epivir) would be effective in treating HIV infection. Palivizumab (Synagis) is a monoclonal antibody used to treat respiratory syncytial virus (RSV) infection. Naproxen (Naprosyn) and sulfasalazine (Azulfidine) are the drugs used to treat juvenile idiopathic arthritis.

A child who is suspected of suffering from meningitis has papilledema, neurologic deficits, and a bulging fontanelle. Which test, if advised for the child, should be questioned by the nurse? A) Lumbar puncture B) Complete blood count C) Computed tomography scan D) Magnetic resonance imaging (MRI)

A) Lumbar puncture In a child with meningitis, presence of papilledema, neurologic deficits, and a bulging fontanelle indicates increased intracranial pressure. Lumbar puncture is contraindicated in increased intracranial pressure, because it may cause herniation of the brainstem. A complete blood count is required to determine the baseline values of the blood cells. A computed tomography scan and MRI are needed to assess the level of meningeal irritation.

A child is demonstrating manifestations of attention deficit hyperactivity disorder (ADHD). The nurse is discussing the possible reasons for the child's condition with the mother. What reasons does the nurse include in the discussion? Select all that apply. A) Maternal cigarette smoking during pregnancy B) Maternal alcohol consumption during pregnancy C) Increased exposure to lead in the early childhood D) Increased consumption of salt in the early childhood E) Increased consumption of sugar in the early childhood

A) Maternal cigarette smoking during pregnancy B) Maternal alcohol consumption during pregnancy C) Increased exposure to lead in the early childhood A child is likely to develop ADHD if the mother smoked cigarettes or consumed alcohol during the pregnancy. Increased exposure to lead in early childhood may also lead to the development of ADHD in the later stages of childhood. Increased consumption of salt and sugar in the early childhood is unrelated to the development of ADHD in the laterstages of childhood.

Careful preparation of a patient for an IVP is necessary. What nursing intervention would be included in the preparation? (SELECT ALL THAT APLLY). A) NPO for about 8 hours before examination. B) Ascertaining whether patient has allergy to magnesium C) Giving prescribed bowel prep D) Instructing patient concerning IVP E) Discussing the anesthesia needed for the procedure

A) NPO for about 8 hours before examination. C) Giving prescribed bowel prep

A nurse is collecting information from a patient's mother while in the clinic. It is suspected that the patient has scarlet fever, and the nurse will assess the child for what signs or symptoms of scarlet fever? Select all that apply. A) Pharyngitis B) Koplik spots C) Peeling skin on palms D) Sandpaper-like red rash E) White strawberry tongue

A) Pharyngitis D) Sandpaper-like red rash E) White strawberry tongue A sandpaper-like red rash, white strawberry tongue, and pharyngitis are all associated with scarlet fever. Koplik spots are associated with the disease rubeola. Peeling of the skin on the palms of the hands is associated with Kawasaki disease.

The nurse is caring for a child who is undergoing anti-D antibody therapy. Which laboratory report of the patient should the nurse regularly monitor? A) Platelet count B) Creatinine levels C) Blood urea levels D) White blood cells (WBCs)

A) Platelet count Anti-D antibody therapy is given to children afflicted with idiopathic thrombocytopenia purpura (ITP). ITP is characterized by lowered platelet levels in the body. Anti-D antibody therapy is aimed at increasing the platelet count. Therefore, the nurse should regularly monitor platelet level of the child affected with ITP. Creatinine and blood urea levels are monitored in children with renal dysfunction. Renally-compromised children are treated with diuretics and not with anti-D antibody therapy. WBC count increases if the child has infection; anti-D antibody therapy is not used to treat infection.

The nurse is reviewing the UA report on an assigned patient . The nurse recognizes which findings to be normal? (SELECT ALL THAT APPLY) A) Turbidity clear B) pH 6.0 C) Glucose negative D) Red blood cells, 15-20 E) White blood cells, 1-3

A) Turbidity clear B) pH 6.0 C) Glucose negative

Which are necessary for the maturation of a red blood cell? (SELECT ALL THAT APPLY) A) Vitamin B-12 B) Folic acid C) Renal erythropoietin factor D) Capric acid E) Iron

A) Vitamin B-12 B) Folic acid C) Renal erythropoietin factor E) Iron

Which represents the correct medical management for the patient with DIC? (SELECT ALL THAT APPLY). A) addressing and correcting the underlying cause B) Transfusion replacement C) Cryoprecipitate D) Administering colony-stimulating factor (filtration [neupogen]) E) Heparin therapy

A) addressing and correcting the underlying cause B) Transfusion replacement C) Cryoprecipitate E) Heparin therapy

What does the patient with respiratory acidosis demonstrate? (SELECT ALL THAT APPLY). A) disorientation B) pH less than 7.35 C) pH more than 7.44 D) rapid respiration's E) slow deep respirations

A) disorientation B) pH less than 7.35

A patient with TB has a nursing diagnosis of noncompliance. What does the nurse recognize as the most common etiology factor? A) fatigue and lack of energy to manage self-care B) Lack of knowledge about how the disease is transmitted C) Little or no motivation to adhere to a long-term drug regimen D) Feelings of shame and the response to the social stigma with TB

A) fatigue and lack of energy to manage self-care

A 52-year-old patient had a laryngectomy due to cancer of the larynx. Discharge instructions are given to the patient and his family. Which response, by written communication from the patient or verbal response by the family, indicates that the instructions need to be clarified? A) report swelling, pain, or excessive drainage. B) the auctioning at home must be a clean procedure, not sterile C) cleanse skin around stoma twice daily (BID), using hydrogen peroxide; rinse with water; pat dry. D) It is acceptable to take OTC medications now that the condition is stable

A) report swelling, pain, or excessive drainage.

Which statement concerning Hodgkin's lymphoma is correct? A) the 10 year survival rate for stage I or II Hodgkin's lymphoma is more than 90%. B) Hodgkin's lymphoma commonly occurs between the ages 15 and 35 years. C) Hodgkin's lymphoma is not a curable disease. D) The incidence of Hodgkin's lymphoma in the female population has increased.

A) the 10 year survival rate for stage I or II Hodgkin's lymphoma is more than 90%.

After performing amniocentesis, the primary health care provider determines that the fetal lungs are not developed. Which intervention would the primary health care provider prescribe for the patient before delivery?

Administer betamethasone (Celestone) to the patient. Amniocentesis is used to detect fetal lung maturity before delivery. Premature infants are more likely at a risk of respiratory distress syndrome (RDS) due to deficiency of lung surfactant. Therefore, the primary health care provider would administer corticosteroids, such as betamethasone (Celestone), to the patient before delivery to increase the production of surfactant in the premature infant. Ventilator support is provided for patients who have decreased respiratory rate. Digoxin (Lanoxin) is administered to treat patients with congenital heart disease. IV fluids are given to pregnant women if theyhave low levels of amniotic fluid before delivery.

A client's laboratory report indicates the presence of hypokalemia. For which clinical manifestations associated with hypokalemia should the nurse assess the client?

Anorexia and leg cramps The gastrointestinal manifestations associated with hypokalemia are caused by decreased neuromuscular irritability of the gastrointestinal tract; this results in anorexia, nausea, vomiting, and decreased peristalsis. Because of potassium's role in the sodium-potassium pump, hypokalemia results in altered neuromuscular functioning, which precipitates leg cramps. Thirst is associated with hypernatremia. Rapid, thready pulse is associated with dehydration and hyponatremia. Dry mucous membranes are associated with hypernatremia.

The nurse is caring for a patient with an infected surgical wound with wet to dry dressings. A wet to dry dressing involves:

Applying gauze soaked in Saline solution to a wound, then covering it with a dry gauze dressing. Hey wet to dry dressing is often ordered for a patient with an infected wound, in order to debrief any necrotic tissue. This type of dressing involves application of a gauze soaked in saline into or on top of the wound, then covering the wet gauze with a dry gauze. As drying occurs, the necrotic tissue is absorbed into the gauze. The dressing is changed when the gauze becomes dry.

Give an example of an anticholinergic. How does it work? (Drugs used for lower respiratory disorders)

Atrovant - used in conjunction with albuterol for prophylactic and maintenance treatment

A mother with a 6-year-old child reports that over the past few weeks her child has complained of stomach pain and trouble concentrating at school and rarely wants to play anymore. The mother also reports seeing her child eating dirt recently. The nurse should prepare to test the child for what disorder? A) Benign tumor B) Lead poisoning C) Sickle cell disease D) Hormone deficiency

B) Lead poisoning A child suspected of lead poisoning may have weight loss (anorexia), abdominal pain, decreased attention span, learning difficulties, and craving for or eating of nonfood substances (pica). Benign tumors rarely cause symptoms unless the tumors are pressing on vital organs. A hormone deficiency would not cause the child to eat dirt; however, the other symptoms could occur. A child with sickle cell disease may complain of pain; however, the pain is usually localized to where the sickling is taking place.

A nurse is caring for an infant born with a cleft lip. Which concern would be the highest priority? A) Parental coping B) Nutritional intake C) Tissue or skin integrity D) Potential for aspiration

B) Nutritional intake The highest priority for a child with a cleft lip is adequate nutritional support. Before surgical repair, feedings can be difficult. Skin or tissue integrity is a priority after surgery and during the process of healing. Parental coping is in relation to the parents and learning how to accept the deformity and the corrective measures to take, but would not be a priority over nutritional intake. The infant primarily needs nutritional support. The prevention of aspiration is a goal while obtaining adequate fluid intake.

A nurse is teaching the parents of a newborn with a cleft lip and palate. The nurse identifies that teaching has been successful when the parents demonstrate which feeding intervention? A) Feed newborn lying on back B) Place newborn's head upright C) Use regular nipples on bottles D) Burp only after feeding is completed

B) Place newborn's head upright In a newborn with cleft lip and palate, a primary goal for nursing is to prevent aspiration. The best method for feeding is to support the infant's head in an upright position. A newborn with a cleft lip and palate should be burped frequently because these infants tend to swallow large amounts of air when feeding. A newborn should never be fed while lying on the back because this position increases the risk for aspiration as well as otitis media. Newborns with cleft lip and palate cannot generate enough suction to feed through a normal nipple or breast. They require special nipples (cleft palate nipples).

The spleen is located in which quadrant of the ABD cavity? A) Upper right B) Upper left C) Lower left D) Lower right

B) Upper left

Which laboratory finding is a strong indicator of disseminated intravascular coagulation (DIC)? A) an elevated platelet count B) an elevated D-dimer test result C) a normal PT D) an elevated fibrinogen level

B) an elevated D-dimer test result

In discussing dietary needs with a patient with ESRD, the nurse indicates that potassium rich foods should be limited in the diet; these include: A) apples, applesauce, grapes and raisins. B) bananas, nuts, and chocolate. C) grapefruit, tomatoes, oranges, and bananas. D) milk, grapefruit, orange juice, and sugar.

B) bananas, nuts, and chocolate.

Which nursing diagnosis for a patient with an acute asthma attack has the highest priority? A) anxiety related to difficulty in breathing. B) ineffective airway clearance related to bronchoconstriction and increased mucus production C) infective health maintenance related to lack of knowledge about attack triggers and appropriate use of medications D) ineffective breathing pattern related to anxiety

B) ineffective airway clearance related to bronchoconstriction and increased mucus production

The healthcare provider ordered a culture and sputum specimen for a patient who has pneumonia. When should the nurse collect these specimens? (SELECT ALL THAT APPLY). A) after initiation of antibiotic therapy B) the morning after admission C) before initiation of antibiotic therapy D) at the first sign of elevated pressure E) after the patient has had breakfast

B) the morning after admission C) before initiation of antibiotic therapy

The diagnostic marker used in patients with CHF is called:

B-type natriuretic peptide B-type natriuretic peptide (BNP) is secreted from the ventricles or lower chambers of the heart in response to changes in pressure that occur when heart failure develops and worsens. The level of BNP in the blood increases and worsens. The level of BNP in the blood increases when heart failure symptoms worsen, and decreases when the heart failure condition is stable.

The nurse prepares a list of recommended foods for a client with hypertension that is to begin a 2-gram sodium diet. The list should include which foods? Multiple selection question Beef steaks Aged cheeses Luncheon meats Cooked broccoli Dehydrated soups

Beef steaks Cooked broccoli Beef is low in sodium. Broccoli does not have significant sodium levels. Aged cheeses are high in sodium as well as saturated fat. Luncheon meat is processed and has high sodium levels to help with its preservation. Dehydrated soups are high in sodium unless they specifically state on the package that they are low in sodium.

Your patient has just returned to her room following a cystoscopy. What should you monitor the patient for?

Bleeding Cystoscopy, in which the bladder and urethra are viewed with a scope, has the potential to cause bleeding. Monitor the color of the patient's urine to assess for bleeding.

What statement by a patient indicates the need for further teaching before renal angiography? A) "I will miss having breakfast." B) "I know the nurse will be checking my pulse after the test." C) "I'm glad I don't have to stay in bed after the test." D) "I had a test similar to this 3 years ago."

C) "I'm glad I don't have to stay in bed after the test."

On assessment, the nurse should expect what finding in a child with coarctation of the aorta? A) Clear lungs bilaterally to auscultation B) Apical heart rate of 60 bpm and respiratory rate of 20 C) Blood pressure in upper extremities higher than in lower extremities D) Bounding pulses in upper extremities and thready pulses in lower extremities

C) Blood pressure in upper extremities higher than in lower extremities Coarctation of the aorta is a narrowing of the lumen of the aorta (usually at the site of the ductus arteriosus). This results in increased pressure proximal to the defect (head and upper extremities) and decreased pressure distal to the defect (body and lower extremities). Blood pressure in the arms is 20 mm Hg higher than in the legs. Bounding pulses would be located in the lower extremities instead of in the upper extremities. An apical heart rate of 60 and respiratory rate of 20 in a newborn are extremely abnormal but are not associated with this heart defect. Children with this type of heart defect usually demonstrate heart failure symptoms, such as crackles and shortness of breath; therefore the lungs will not be clear to auscultation.

The goal of treatment for a child with patent ductus arteriosus is to close the defect by using a prostaglandin inhibitor. An example of this class of medication would be which? A) Lisinopril (Privinil) B) Amitriptyline (Elavil) C) Indomethacin (Indocin) D) Metformin (Glucophage)

C) Indomethacin (Indocin) The administration of indomethacin (a prostaglandin inhibitor) has often been effective in closing the patent ductus arteriosus in full-term and premature newborns. Lisinopril is used to treat hypertension. Glucophage is used to treat diabetes mellitus. AMITRIPTYLINE is used to treat anxiety or to help patients sleep.

Juvenile idiopathic arthritis is considered a chronic arthritis and has many different treatment modalities, including which? Select all that apply. A) Glycosides B) Antihypertensive drugs C) Slow-acting antirheumatic drugs (SAARDs) D) Non-steroidal anti-inflammatory drugs (NSAIDs) E) Disease-modifying antirheumatic drugs (DMARDs)

C) Slow-acting antirheumatic drugs (SAARDs) D) Non-steroidal anti-inflammatory drugs (NSAIDs) E) Disease-modifying antirheumatic drugs (DMARDs) NSAIDs such as naproxen (Naprosyn) and ibuprofen (Motrin) are used as the first line of defense in treating juvenile arthritis. When NSAIDs no longer provide pain relief, SAARDs, such as methotrexate (Rheumatrex) and hydroxychloroquine (Plaquenil) are added. When other treatments prove no longer effective, then DMARDs, such as methotrexate (Trexall) and sulfasalazine (Azulfidine), are used in conjunction with NSAIDs. Antihypertensive medications are used to treat high blood pressure, not arthritis. Glycosides are used to treat diabetes, not arthritis.

Hodgkin's lymphoma occurs more frequently in individuals with which risk factors? A) a history of cancer treated with radiation B) exposure to nuclear explosions C) infection with the Epstein-Barr virus D) infection with Helicobacter pylori

C) infection with the Epstein-Barr virus

Ineffective breathing pattern, related to decreased lung expansion during an acute attack of asthma, is an appropriate nursing diagnosis. Which nursing interventions are correct? (SELECT ALL THAT APPLY). A) place patient in a supine position B) administer oxygen therapy as ordered C) remain with the patient during an acute attack to decrease fear and anxiety D) incorporate rest periods into activities and interventions E) maintain semi-Fowler's position to facilitate ventilation

C) remain with the patient during an acute attack to decrease fear and anxiety D) incorporate rest periods into activities and interventions E) maintain semi-Fowler's position to facilitate ventilation

Hematopoiesis

Cells production within the bone marrow, starts out as a stem cell and differentiates into several different types of blood cell.

A client just had a suprapubic prostatectomy. Which action should the nurse implement to prevent a secondary bladder infection? Multiple choice question Observe for signs of uremia Attach the catheter to suction Clamp off the connecting tube Change the dressings frequently

Change the dressings frequently After a suprapubic prostatectomy, leakage of urine generally is identified around the suprapubic tube; this creates an environment in which bacteria can flourish if the dressing is not changed frequently. Uremia is caused by inadequate kidney function; it is not directly related to bladder infection. Negative pressure on the bladder may traumatize the delicate tissue; urine should flow because of gravity. Clamping off the tube causes urinary stasis, which increases the risk for infection.

A client who is diagnosed as having a herniated nucleus pulposus complains of pain. The nurse concludes that the pain is caused by the: Multiple choice question Inflammation of the lamina of the involved vertebra Shifting of two adjacent vertebral bodies out of alignment Compression of the spinal cord by the extruded nucleus pulposus Increased pressure of cerebrospinal fluid within the vertebral column

Compression of the spinal cord by the extruded nucleus pulposus Pain results because herniation of the nucleus pulposus into the spinal column irritates the spinal cord or the roots of spinal nerves. Inflammation of the lamina of the involved vertebra is not involved; the lamina is that portion of the vertebra removed during surgery to gain access to the site. The vertebral bodies themselves are not shifting. Circulation of cerebrospinal fluid is not affected.

A client is admitted via the emergency department with the tentative diagnosis of diverticulitis. Which test commonly is prescribed to assess for this problem? Multiple choice question Computed tomography (CT) scan Gastroscopy Colonoscopy Barium enema

Computed tomography (CT) scan A CT scan with contrast is the test of choice for diverticulitis because it effectively reflects the involved colon. An endoscopy assesses the upper, not lower, gastrointestinal tract. Colonoscopy is contraindicated because of the possibility of perforation and peritonitis. Barium enema is contraindicated because of the possibility of perforation and peritonitis.

Normal bilirubin lab value:

Conjugated: 0.1-0.4 Total: 0.3-1.1

An 8-month-old child with developmental dysplasia of the hip (DDH) is admitted for the placement of a hip spica cast. The nurse demonstrates understanding of the purpose of the cast when she makes which statement? A) "The hip spica cast will keep the pelvis tilted in an upward position." B) "The hip spica cast will internally rotate the feet to maintain correct position." C) "The hip spica cast will support the pelvis and the spine and expedite spinal fusion." D) "The hip spica cast will externally rotate the feet, which keeps the femur head in the acetabulum."

D) "The hip spica cast will externally rotate the feet, which keeps the femur head in the acetabulum." Treatment for a child with DDH between the ages of 6 and 24 months may require manipulation of the head of the femur into the socket (acetabulum). This is accomplished by placing the child in a hip spica cast. The child's feet are externally rotated, causing the hips to rotate in, keeping the femur head in the acetabulum. The purpose of the hip spica cast is to keep the femur head in the acetabulum, not to keep the pelvis tilted upward. The purpose of the hip spica cast is to keep the femur head in the acetabulum, not to support the spine and pelvis or to expedite spinal fusion. The hip spica cast causes external rotation of the feet, which causes the hip to rotate inward. With internal rotation of the feet, the hips would rotate outward, causing the head of the femur to become displaced.

Full-term, healthy infants should have adequate iron stores from their mothers until what age? A) 12 months B) 18 months C) 6 to 8 months D) 4 to 6 months

D) 4 to 6 months Full-term, healthy infants have sufficient iron stores for the first 4 to 6 months of life. If an infant is breastfed, the infant should receive adequate iron for the first 6 months of life. If an infant is not breastfed, the infant should receive iron-fortified formulas for the first 9 to 12 months of life. Once a child has been introduced to solid food (usually around 12 months), the child should receive adequate iron from the diet.

Which laboratory finding, in conjunction with the presenting symptoms, indicates nephrotic syndrome? A) Low specific gravity B) Normal platelet count C) Decreased hematocrit D) Increased urine protein

D) Increased urine protein Analysis of the urine should reveal a high specific gravity and proteinuria. Hemoglobin and hematocrit are usually normal or elevated. The platelet count is elevated as a result of hemoconcentration.

A distraught mother brings her 7-year-old child into the clinic for testing. The child has missed school because of headaches and abdominal pain. She explains that the pain is worse in the morning but is gone by mid-afternoon. The nurse recognizes that these symptoms could be associated with what? A) Child abuse B) Malnutrition C) Hypoglycemia D) School avoidance

D) School avoidance A child who misses school because of physical symptoms that are related to an emotional origin is demonstrating school avoidance. Further investigation regarding the child's situation at school is necessary. The timing and duration of the painful incidents suggests that the problem is something other than child abuse, hypoglycemia, or malnutrition.

A child is diagnosed with hypospadias. What would the nurse find during the examination of the child's genital organs? A) The testes in the patient have failed to descend into the scrotum. B) The opening of the foreskin of the penis is narrowed or stenosed. C) The urethral opening is located along the dorsal surface of the penile shaft. D) The urethral opening is located along the ventral surface of the penile shaft.

D) The urethral opening is located along the ventral surface of the penile shaft. Hypospadias is a condition where the urethral opening is located along the ventral surface of the penile shaft. Cryptorchidism is the failure of the testes to descend into the scrotum. If the opening of the foreskin of the penis is narrowed or stenosed, the patient is said to have phimosis. In epispadias, the urethral opening is located along the dorsal surface of the penile shaft.

A patient was seen in a clinic for an episode of epistaxis, which was controlled by placement of anterior nasal packing. During discharge teaching, what instructions will the nurse discuss with the patient? A) avoid vigorous nose blowing and strenuous activity B) use aspirin or aspirin containing compounds for pain relief C) apply ice compresses to the nose every 4 hours for the first 48 hours D) leave the packing in place for 7-20 days until it is removed by the health care provider

D) leave the packing in place for 7-20 days until it is removed by the health care provider

An 85-year-old client has a three-day history of nausea, vomiting, and diarrhea. The client develops weakness and confusion and is admitted to the hospital. To best monitor the client's rehydration status, the nurse should assess the client's: Multiple choice question Skin turgor Daily weight Urinary output Mucous membranes

Daily weight A continuous increase in serial weight determinations indicates a movement toward correction in the dehydration; 1 L of fluid weighs 2.2 pounds. The skin in older adults has less fluid and subcutaneous fat than younger adults, which results in a subjective and inaccurate assessment of rehydration. In older adults there can be a 50% decrease in renal blood flow and tubular function; therefore, urinary output does not provide an accurate assessment of rehydration therapy. The mucous membranes in older adults are drier than in younger adults because of the decrease in salivary secretions and therefore do not provide an accurate assessment of rehydration therapy.

A client with rheumatoid arthritis has been taking a steroid medication for the past year. For which complication of prolonged use of this medication should the nurse assess the client? Multiple choice question Decreased white blood cells Increased C-reactive protein Increased sedimentation rate Decreased serum glucose levels

Decreased white blood cells Prolonged use of steroids may cause leukopenia as a result of bone marrow depression. Increased C-reactive protein and sedimentation rate elevate in acute inflammatory diseases; steroids help decrease it. Serum glucose levels increase with steroid use.

The indirect Coombs test is performed on pregnant woman in order to:

Determine if the mother has antibodies to the fetus's blood. An indirect Coombs test determines whether there are antibodies to the Rh factor in the mother's blood. A normal, or negative, result means that the mothers not developed antibodies against the fetus's blood. A negative Coombs test indicates that the fetus is not presently in danger from problems relating to Rh incompatibility. An abnormal, or positive, result means that the mother had developed antibodies to the fetal RBCs and is sensitized. This result requires the injection of a medication called Rho(D) immune globulin.

The nurse evaluates that the teaching about myasthenic and cholinergic crises is understood when a client who is diagnosed with myasthenia gravis states that a characteristic common to both crises is: Multiple choice question Diarrhea Salivation Difficulty breathing Abdominal cramping

Difficulty breathing Because of the decrease in tone and strength of the respiratory muscles, difficulty breathing is a prominent feature of both crises. Diarrhea occurs in cholinergic crisis; it is an effect of an overdose of the medications (anticholinesterases) used to treat myasthenia gravis. Salivation occurs in cholinergic crisis; it is an effect of an overdose of the medications (anticholinesterases) used to treat myasthenia gravis. Abdominal cramping occurs in cholinergic crisis; it is an effect of an overdose of the medications (anticholinesterases) used to treat myasthenia gravis.

passive transport

Do not require cellular energy (ex: diffusion, osmosis, filtration move substance through the cell)

Duplex scanning

Doppler and ultrasound are used for a better picture of the blood flow.

Atrial fibrillation (A-Fib)

Electrical activity in the atria is disorganized, causing the atria to fibrillate, or quiver, rather than contract as a unit. rate: 350-600bpm. ventricular response: 100-180 bpm. S/S: pulse deficit, palpitations, dyspnea, angina, light headedness, syncope, fatigue, changes in LOC, pulmonary edema. THROMBI may form in the atria as a result of ineffective atrial contraction.

Your elderly demented patient cannot complete a.m. care without assistance. To help maintain independence you should do what?

Encourage the patient to perform as much of the care as possible while you supervise and cue the patient.

When assessing a client's abdomen, the nurse palpates the area directly above the umbilicus. This area is known as the: Multiple choice question Iliac area Epigastric area Hypogastric area Suprasternal area

Epigastric area

A client has a history of gastroesophageal reflux disease (GERD). Why should the nurse also monitor the client for clinical manifestations of heart disease? Multiple choice question Esophageal pain may imitate the symptoms of a heart attack. GERD may predispose to heart disease. Strenuous exercise may exacerbate both cardiac and reflux problems. Similar changes in laboratory studies may occur in both cardiac and reflux problems.

Esophageal pain may imitate the symptoms of a heart attack. Symptoms associated with myocardial infarction may be interpreted by a client as esophageal reflux and therefore ignored. GERD does not predispose to heart disease. Exercise tends to aggravate cardiovascular problems to a much greater extent than esophageal problems. Laboratory workups help differentiate these two diagnoses. Tests, such as cardiac enzymes, can help to reveal a myocardial infarction, thereby facilitating differentiation between these problems.

After a laryngectomy is scheduled, the most important factor for the nurse to include in the preoperative teaching plan is: (Multiple choice question) Establishing a means for communicating postoperatively Explaining that there will be a feeding tube postoperatively Demonstrating how to care for a permanent laryngeal stoma Teaching how to cough to expectorate bronchial secretions effectively

Establishing a means for communicating postoperatively Communication is a priority; it facilitates interaction, limits anxiety, and promotes safety. A nasogastric tube can cause trauma to the suture lines; total parenteral nutrition may be used. Demonstrating how to care for a permanent laryngeal stoma is done postoperatively as the client begins to accept the laryngectomy. After a laryngectomy the client cannot cough; expectoration occurs through the stoma.

When assessing for hemorrhage after a client has a total hip replacement, the most important nursing action is to: Multiple choice question Measure the girth of the thigh Examine the bedding under the client Check the vital signs every four hours Observe for ecchymosis at the operative site

Examine the bedding under the client Because of the recumbent position, drainage may flow by gravity under the client and not be noticed unless the bedding is examined. Measuring the girth of the thigh is inaccurate when there is a dressing in place. In the immediate postoperative period, vital signs should be taken more frequently than every four hours; in addition, observation of the site is a more reliable indicator of hemorrhage. Dressings impede an accurate assessment of the site for ecchymosis

A paracentesis is prescribed for a client recently admitted to a medical unit. The nurse recalls that the procedure is performed for what reasons? Multiple selection question Extract peritoneal fluid Improve respiratory status Decrease intrapleural fluid Increase intraabdominal tension Obtain peritoneal fluid for cultures

Extract peritoneal fluid Improve respiratory status Obtain peritoneal fluid for cultures When a client has ascites, a peritoneal tap (paracentesis) may be prescribed to remove fluid for diagnostic purposes and relieve discomfort. The removal of intraabdominal fluid relieves pressure against the diaphragm, which will improve the client's respiratory status. A culture of peritoneal fluid may provide information about the cause of the ascites. Closed-chest drainage, not a paracentesis, removes fluid from the pleural space. A paracentesis is done to decrease, not increase, intraabdominal pressure.

Heart failure (CHF) mnemonic: "F.A.I.L.U.R.E."

F - FAULTY valves - stenosis (too narrow), regurgitated (it leaks), infected. A - ARRHYTHMIAS - Afib (atrium is queebering, blood pools), tachycardia. I - INFARCTION (MI or CAD) - muscle is ischemic and dies. L - LINEAGE (congenital [family Hx]). U - UNCONTROLLED HTN - causes stiffening of wall ventricles. R - RECREATIONAL DRUGS USE - cocaine and alcohol. E - EVADORS - viruses/infections that attack the heart.

A nurse is caring for a client who had a kidney transplant. What sign indicates that the client may be rejecting the transplanted kidney? Multiple choice question Fever Hematuria Moon face Yellow sclera

Fever Fever is a characteristic of the systemic inflammatory response to the antigen (transplanted kidney). Hematuria is not indicative of rejection; however, its occurrence necessitates further investigation. A moon face (moon facies) is an effect of steroid therapy and does not indicate rejection. Jaundice is unrelated to rejection of a transplanted kidney.

During a client's routine physical examination, a chest x-ray film reveals a lesion in the right upper lobe. What information in the client's history supports the health care provider's diagnosis of pulmonary tuberculosis? Multiple selection question Fever Dry cough Night sweats Frothy sputum Engorged neck veins Blood-tinged sputum

Fever Night Sweats Blood-tinged sputum Tuberculosis is an infectious disease in which recurrent fevers are present, usually in the late afternoon. Profuse diaphoresis at night (night sweats) is a classical sign of tuberculosis. Blood-tinged sputum (hemoptysis) results from pathophysiological trauma to mucous membranes. The cough is productive, not dry, because the inflammatory process causes purulent mucus. Frothy sputum is present with pulmonary edema, not tuberculosis. Engorged neck veins are symptomatic of heart failure.

During change of shift report the night nurse indicates that a client cannot tolerate the prescribed intermittent tube feedings. The nurse receiving report should first: Multiple choice question Suggest that an antiemetic be prescribed Change the feeding schedule to omit nights Request that the type of solution be changed Gather more data from the night nurse about the technique used

Gather more data from the night nurse about the technique used Rapid administration, incorrect positioning, and inadequate solution temperature are common causes of intolerance to tube feedings. Although suggesting that an antiemetic be prescribed may be done eventually, the feeding technique should be assessed first. Feedings generally are tolerated better if given frequently in small amounts over the entire 24 hours. Although changing the feeding schedule to omit nights and requesting that the type of solution be changed may be done eventually, the feeding technique should be assessed first.

The license practical nurse is reviewing laboratory data for a patient with COPD. She notes that the glucose levels are abnormally elevated, but the patient has no history of diabetes. Which medication is most likely to be responsible for elevated gluclose levels?

Glucocorticoids Glucocorticoids are a type of steroid that maybe administered for patients with a history of asthma or breathing problems, among other indications. They are administered to reduce inflammation in the body. A well known side effect of steroids is hyperglycemia, which may be treated with short acting insulin if necessary.

A client is diagnosed as having malabsorption syndrome secondary to celiac sprue. The client asks the nurse if there is anything that can help improve symptoms of the syndrome. The nurse encourages the client to incorporate which addition for symptom improvement? Multiple choice question Folic acid Vitamin B 12 Corticosteroids Gluten-free diet

Gluten-free diet Gluten, a cereal protein, appears to be responsible for morphologic changes of the intestinal mucosa with nontropical sprue (adult celiac disease). Folic acid, along with antimicrobial agents, is used to treat tropical, not celiac, sprue; it causes dramatic improvement in tropical sprue. Vitamin B 12 may be administered if macrocytic anemia or achlorhydria develops; however, it does not correct the major pathology. The use of corticosteroids may be advantageous with either form of sprue; however, this does not produce the dramatic effect achieved by a gluten-free diet .

The nurse is caring for a client with a diagnosis of acute kidney failure associated with drug toxicity. When the client complains of thirst, the nurse should offer: Multiple choice question Ice chips Warm milk Hard candy Carbonated soda

Hard candy Sucking on candy will relieve thirst and provide calories without supplying extra fluid. Ice chips add to the restricted fluid intake. Milk contains both fluids and proteins, which should be restricted with acute kidney failure. Carbonated beverages may be high in sodium and provide additional fluid; both should be restricted.

A nurse epidemiologist is responsible for wound consults at the hospital where a client has been admitted with an infected wound. The client asks, "What is the primary role of a nurse epidemiologist?" The nurse explains that the nurse epidemiologist: Multiple choice question Helps health care providers to control infections Decides what antibiotics should be prescribed for infections Works in the laboratory identifying bacteria that cause infection Is responsible for collecting specimens of potentially infectious drainage

Helps health care providers to control infections The nurse epidemiologist helps to devise an infection control strategy. The role of a primary health care provider is to decide what antibiotics should be prescribed for infections. The role of the laboratory technician or technologist is to work in the laboratory to identify bacteria causing infections. The nurse usually is responsible for collecting specimens of potentially infectious drainage

While receiving a blood transfusion, a client develops flank pain, chills, fever, and hematuria. What type of transfusion reaction does the nurse conclude that the client probably is experiencing? Multiple choice question Allergic Pyrogenic Hemolytic Anaphylactic

Hemolytic Hemolytic reaction results from the incompatibility of a recipient's antibodies with transfused red blood cells (RBCs); the reactions result from RBC hemolysis, agglutination, and capillary plugging. Allergic reaction results from an immune sensitivity to foreign serum protein and also is called type I hypersensitivity; signs include urticaria, wheezing, dyspnea, and shock. Bacterial pyrogens are present in contaminated blood and can cause a febrile transfusion reaction; signs include fever and chills. Anaphylactic reaction also is called type II hypersensitivity. A transfusion reaction is not called by this name, although anaphylaxis occurs with an allergic transfusion reaction.

A client is admitted to the hospital with the diagnosis of cancer of the thyroid and a thyroidectomy is scheduled. What is important for the nurse to consider when caring for this client during the postoperative period? Multiple choice question Hypercalcemia may result from parathyroid damage. Hypotension and bradycardia may result from thyroid storm. Tetany may result from underdosage of thyroid hormone replacement. Hoarseness and airway obstruction may result from laryngeal nerve damage.

Hoarseness and airway obstruction may result from laryngeal nerve damage Laryngeal nerve injury can cause laryngeal spasms, resulting in airway obstruction. Parathyroid damage results in hypocalcemia, not hypercalcemia. Thyroid storm (thyroid crisis) is characterized by the release of excessive levels of thyroid hormone, which increases the metabolic rate. An increase in the metabolic rate increases vital signs, resulting in hypertension, not hypotension, and tachycardia, not bradycardia. Tetany is caused by a decrease in parathormone, a parathyroid hormone, not a thyroid hormone.

A client appears anxious, exhibiting 40 shallow respirations per minute. The client complains of feeling dizzy and lightheaded and of having tingling sensations of the fingertips and around the lips. The nurse concludes that the client's complaints probably are related to: Multiple choice question Eupnea Hyperventilation Kussmaul's respirations Carbon dioxide intoxication

Hyperventilation The client is hyperventilating and blowing off excessive carbon dioxide, which leads to these adaptations; if uninterrupted this can result in respiratory alkalosis. Eupnea is normal, quiet breathing; the client has shallow, rapid breathing. Kussmaul's respirations are deep, gasping respirations associated with diabetic acidosis and coma. These adaptations are related to a decreased carbon dioxide level in the body.

A client is admitted to the hospital for an adrenalectomy. The nurse is providing postoperative care before the client's replacement steroid therapy is regulated fully. The nurse should monitor the client for: Multiple choice question Hypotension Hyperglycemia Sodium retention Potassium excretion

Hypotension Because of instability of the vascular system and the lability of circulating adrenal hormones after an adrenalectomy, hypotension frequently occurs until the hormonal level is controlled by replacement therapy. Hyperglycemia is a sign of excessive adrenal hormones; after an adrenalectomy, adrenal hormones are not secreted. Sodium retention is a sign of hyperadrenalism; it does not occur after the adrenals are removed. Potassium excretion is a response to excessive adrenal hormones; after an adrenalectomy, adrenal hormones are decreased until replacement therapy is regulated.

Sickle cell anemia

Hypoxia, dehydration, and acidosis are associated with this disease and improved with oxygen and fluids. 1. Opioids manage pain 2. Antibiotics - for underlying bacterial infection from acute chest pain 3. Treatment: Blood transfusion * risk for iron overload and immunologic intolerence

During chest physiotherapy (CPT), a client reports fatigue, and the client's heart rate increases from 90 to 140 beats per minute. What should the nurse do next? Multiple choice question Interrupt the therapy. Encourage deep breathing. Place the client in the low-Fowler position. Have the client complete the therapy before resting.

Interrupt the therapy. The client's response indicates lack of physiological tolerance to the procedure, and it must be interrupted. Encouraging deep breathing may be encouraged, but it is not the first intervention. Deep breathing must be done cautiously because it may precipitate respiratory alkalosis . The high-Fowler or orthopneic position is more appropriate for clients who are experiencing cardiopulmonary difficulties. Having the client complete the therapy before resting is contraindicated because the client is not tolerating the procedure. The therapy is intended to clear the respiratory passages of sputum and increase oxygenation. The heart rate should remain the same or decrease, not increase.

A nurse is counseling a client who has gonorrhea. What additional fact about gonorrhea, besides the fact that it is highly infectious, should the nurse teach this client? Multiple choice question It is easily cured. It occurs very rarely. It can produce sterility. It is limited to the external genitalia.

It can produce sterility. Inflammation associated with gonorrhea may lead to destruction of the epididymis in males and tubal mucosal destruction in females, causing sterility. Many gonococci have become penicillin resistant and difficult to treat. Gonorrhea is a common sexually transmitted infection. Neisseria gonorrhoeae will invade internal structures, particularly the epididymis in males and the fallopian tubes in females.

A nurse is caring for a client with a diagnosis of type 1 diabetes who has developed diabetic coma. Which element excessively accumulates in the blood to precipitate the signs and symptoms associated with this condition? Multiple choice question Sodium bicarbonate, causing alkalosis Ketones as a result of rapid fat breakdown, causing acidosis Nitrogen from protein catabolism, causing ammonia intoxication Glucose from rapid carbohydrate metabolism, causing drowsiness

Ketones as a result of rapid fat breakdown, causing acidosis Ketones are produced when fat is broken down for energy. Although rarely used, sodium bicarbonate may be administered to correct the acid-base imbalance resulting from ketoacidosis; acidosis is caused by excess acid, not excess base bicarbonate. Diabetes does not interfere with removal of nitrogenous wastes. Carbohydrate metabolism is impaired in the client with diabetes.

A nurse is assisting a health care provider to perform a sigmoidoscopy. In which position should the nurse place the client for this procedure? Multiple choice question Sims Prone Lithotomy Knee-chest

Knee-chest Knee-chest position maximally exposes the rectal area and facilitates entry of the sigmoidoscope . The Sims position does not expose the rectal area to the same extent as does the knee-chest position; it can be used for a sigmoidoscopy if a client is unable to maintain the knee-chest position. Although prone refers to a facedown position, the rectal area is not exposed. The lithotomy position is appropriate for gynecological examinations.

A nurse is caring for a client who has had type 1 diabetes for 25 years. The client states, "I have been really bad for the last 15 years. I have not paid attention to my diet and have done little to control my diabetes." What signs of common complications of diabetes might the nurse expect to identify when assessing this client? Multiple selection question Leg ulcers Loss of visual acuity Thick, yellow toenails Increased growth of body hair Decreased sensation on the feet

Leg ulcers Loss of visual acuity Thick, yellow toenails Decreased sensation on the feet Leg ulcers are a common response to the microvascular and macrovascular changes associated with diabetes. Retinopathy, damage to the microvascular system of the retina (e.g., edema, exudate, and local hemorrhage), occurs as a result of the occlusion of the small vessels in the eyes, causing microaneurysms in the capillary walls. Thick, yellow toenails result from prolonged inadequate arterial circulation to the feet. Pedal pulses diminish, which can result in gangrene, necessitating amputation. Diabetic neuropathies affect 60% to 70% of people with diabetes. It is theorized that consistent hyperglycemia causes a buildup of sorbitol and fructose in the nerves that results in impairment via an unknown process. Inadequate arterial circulation to hair follicles results in a lack of hair on the feet and ankles. The skin becomes dry and cracks, predisposing to leg ulcers and infection.

A nurse is obtaining a history and performing a physical assessment of a client who has cancer of the tongue. Which clinical findings should the nurse expect to identify? Multiple selection question Halitosis Leukoplakia Bleeding gums Substernal pain Alterations in taste Enlarged cervical lymph nodes

Leukoplakia Alterations in taste Enlarged cervical lymph nodes Leukoplakia are white, thickened patches that tend to fissure and become malignant; ulcerations in the mouth or on the tongue may indicate cancer. Taste buds in the tongue may be impaired, resulting in alterations in taste. Regional lymph nodes enlarge as cancer cells begin to metastasize. Halitosis is not an early sign of or specific to cancer of the mouth. Bleeding gums occur in gingival diseases. Pain associated with cancer of the tongue does not radiate to the substernal area.

If the kidneys are unable to correct metabolic acidosis, the __________ will respond in an attempt to correct the imbalance.

Lungs

Insensible loss of water from the body occurs mainly in the:

Lungs and skin Sensible sources of water loss include sweat, urine and water lost from the intestines. Insensible sources of water loss include the lungs and skin.

Normal creatinine lab value:

Males: 0.2-0.5 Females: 0.3-0.9

Normal Hgb (hemoglobin) lab value:

Males: 13-18 Females: 12-16 Newborns: 16.5-19.5

Normal erythrocyte platelet lab value:

Males: 4.6-6.2 Females: 4.2-5.4

Normal Hct (hematocrit) lab value:

Males: 40-54 Females: 37-47

Normal creatinine kinase (CK) lab value:

Males: 55-170 Females: 30-135

Urine specific gravity

Measure of urine concentration * > 1.030 = dehydration * < 1.003 = over hydration (diluted) Normal: 1.003 - 1.030

sensible water loss

Measureable * urine * feces * vomiting * wound drainage

When assessing the breath sounds of a client with chronic obstructive pulmonary disease (COPD), the nurse hears coarse rhonchi. They are described best as: (Multiple choice question) Snorting sounds during the inspiratory phase Moist rumbling sounds that clear after coughing Musical sounds more pronounced during expiration Crackling inspiratory sounds unchanged with coughing

Moist rumbling sounds that clear after coughing Coarse rhonchi, particularly on expiration, indicate partial airway obstruction because of bronchiolar alterations associated with COPD. Snorting sounds are made in the nose. Wheezes are musical sounds usually heard during expiration; they are caused by rapid vibration of bronchial walls. Crackling sounds heard on inspiration that are unchanged by coughing are known as fine crackles; they result when air passes through alveoli that partially are filled with fluid.

incompetent cervix treatment

Monitor v/s, watch for discharge, no heavy lifting or prolonged standing

What is the difference of monozygotic vs. dizygotic?

Monozygotic - at one point 1 egg, splits into 2 (identical twins). Dizygotic - 2 separate eggs, occupy the uterus at the same time. • ovaries release two eggs • ovulation from both ovaries occurs

A client diagnosed with Bell's palsy has many questions about the course of the disorder. The nurse explains that: Multiple choice question Pain occurs with transient ischemic attacks (TIAs) Cool compresses decrease facial involvement Most clients recover from the effects in several weeks Body changes should be expected with residual effects

Most clients recover from the effects in several weeks The client should be assured that the symptoms are not caused by a stroke; the majority of clients recover in a few weeks. Bell's palsy is not caused by a TIA. Paresis or paralysis of cranial nerve VII occurs; discomfort may or may not be present. Moist heat, not a cool compress, increases blood circulation to the nerve. The majority of clients recover without residual effects; occasionally some clients are left with evidence of Bell's palsy. Exercises may help to maintain muscle tone; also, surgery may be necessary.

Give an example of a muculytic agent. How do they work? (Drugs used for lower respiratory disorders)

Mucumyst (acetylcysteine) - acts by dissolving chemical bonds with mucus itself. 1.Side effects: N/V (pungent odor)

You are working in the ED when you are notified that the ambulance is bringing in a woman who has taken an overdose of Tylenol. You should anticipate administering which of the following drugs? a) Narcan b) N-acetylcysteine c) Epinephrine d) 1 L bolus to dilute the effects of Tylenol in the blood stream

N-acetylcysteine The antidote to acetomeniphen overdose is N-acetylcysteine (NAC). It is most effective when given within 8 hours of ingesting acetaminophen. NAC can prevent liver failure if given early enough.

What happens when Na+ and Ca+ enter the cardiac cell?

Na+ - enters rapidly to start depolarization (myocardial contraction). Ca+ - enter later to maintain it.

You are assigned charge nurse responsibilities for the upcoming month. In creating the schedule, you assign a fellow nurse that you do not get along with every holiday shift, despite the requirements being one holiday per season. This action is a violation of which ethical principle?

Non-maleficence Non-maleficence is the ethical principle that refers to "doing no harm." It can refer to doing no harm to patients, or doing no harm to fellow health care workers as well. Within this principle, one should act with empathy and without malicious intent.

The LPN is reviewing lab results for a patient on her unit. She notes a hemoglobin level of 5.9 gm/dL. The most appropriate initial response is:

Notifying the physician immediately A hemoglobin level of 5.9 gm/dL is considered critically low. The most appropriate initial response is to notify the physician. Pending the physician's recommendation, repeat tests and diagnostics exams may then occur. In general, a hemoglobin level above 12 gm/dL would be considered normal for an adult.

A 70-year-old obese males admitted to the cardiac unit with new onset of atrial fibrillation. While in the hospital, the night shift nurse notes that the patient is snoring loudly, then waking abruptly. In the early morning , he reports being excessively tired during the day. The nurse is suspicious for which of the following?

Obstructive sleep apnea Obstructive sleep apnea is a disorder found most often in older obese males. It is the lack of air flow due to an obstruction of the pharynx during sleep. Patients with obstructive sleep apnea often snore loudly and awaken frequently throughout the night following episodes of apnea. They often report daytime tiredness, sore throat, and headaches. For severe cases of sleep apnea, a device called a continuous positive airway pressure (CPAP) machine may be utilized.

A client has a nasogastric feeding tube inserted, and the health care provider prescribes the feeding to be instituted immediately. What should the nurse do first? Multiple choice question Instill normal saline into the tube to maintain patency. Obtain an x-ray to verify that the tube is in the stomach. Auscultate the epigastric area while instilling 15 mL of air. Withdraw 30 mL of stomach contents to verify tube placement.

Obtain an x-ray to verify that the tube is in the stomach. X-ray verification of tube placement is required before anything is instilled into the nasogastric tube. Administering a feeding through a misplaced tube can cause the formula to enter the client's lungs. Instilling normal saline into the tube to maintain patency is unsafe. The normal saline will enter the client's lungs if the tube is in the wrong place. Auscultating the epigastric area while instilling 15 mL of air and withdrawing 30 mL of stomach contents to verify tube placement are not definitive ways to ensure correct placement of the nasogastric tube. Once placement is verified by an x-ray, these methods may be used before initiating a feeding.

Within the first 2½ hours after a radical neck dissection, 40 mL of medium red, bloody fluid is collected in the portable wound drainage system. What should the nurse do first? Multiple choice question Obtain the vital signs. Notify the health care provider. Chart the amount on the Intake & Output (I&O) record. Continue to monitor the amount for another 2½ hours.

Obtain the vital signs. Obtaining the vital signs shows an excessive amount of drainage in 2½ hours because 80 to 120 mL of drainage is expected within the first 24 postoperative hours. The client's status should be assessed first, then the health care provider should be notified. Charting the amount on the I&O record should be done eventually, but it is not the priority at this time. Continuing to monitor the amount for another 2½ hours is unsafe; the client is bleeding excessively.

venous stasis ulcers

Occur slowly and over time. Treatment: elevate the patient extremities and compress.

What does the acronym "P.A.T.C.H.E.S." Mean?

P - no pulses (pulse ["3" = normal]) A - appearance (pale, blue, ashy, gray do to lack of O2) T - temp (cool to the touch) C - capillary reflex (slow = > 3 seconds) H - hardness (tissue starts to break down) E - edema (no pitting edema, hard edema) S - sensation (complain of numbness and tingling)

Which information should be included in the teaching plan for the client diagnosed with epilepsy? Multiple choice question Antiseizure medication must be taken for life. People taking phenytoin (Dilantin) must floss regularly. People with epilepsy can never be issued a driver's license. Loss of consciousness during a seizure requires emergency evaluation.

People taking phenytoin (Dilantin) must floss regularly. Gingival hyperplasia is a common side effect of phenytoin. Regular brushing and flossing decrease gingival hyperplasia. While lifelong treatment with antiseizure medication often is required, some people are able to be weaned from antiseizure medication after they have been seizure free for a period of several years (generally 3--5) and have a normal EEG and neurological examination. Driving laws for people with epilepsy vary from state to state. For example, some states require a seizure-free period of several months and some states require a seizure-free period of up to a year before a driver's license can be issued or reinstated. It is not necessary for the person who has experienced a single seizure to be taken to the hospital unless it is a first time seizure, the seizure is prolonged, or the seizure results in bodily harm.

Eclampsia

Presence of seizure, the uterus contracts (not good for the fetus during a seizure). May lead to a coma that lasts hrs-days. Magnesium sulfate - tie traded to patient response, a lot of the times the patient becomes a 1:1 care with the nurse having vital signs checked Q5-15 min.

A client is admitted to the hospital with a history of cancer of the liver and jaundice. In relation to the jaundice, the nurse expects the client to report the presence of: Multiple choice question Pruritus Diarrhea Blurred vision Bleeding gums

Pruritus Itching associated with jaundice is believed to be caused by accumulating bile salts in the skin. Diarrhea, blurred vision, and bleeding gums are not related to jaundice.

Give an example of a corticosteroid. How does it work? (Drugs used for lower respiratory disorders)

Pulmicort inhaler - Treats chronic asthma for their anti-inflammatory effects. 1. Directly work on smooth muscle relaxation 2. If inhaled works faster and limits side effects systemically

A client has a total hip replacement for long-standing degenerative bone disease of the hip. When assessing this client postoperatively, the nurse considers that the most common complication of hip surgery is: Multiple choice question Pneumonia Hemorrhage Wound infection Pulmonary embolism

Pulmonary embolism A pulmonary embolism is the most common complication of hip surgery because of high vascularity and the release of fat cells from the bone marrow. The occurrence of pneumonia is rare because of early activity after surgery. In addition, the operative area is not in proximity to the diaphragm and lungs; therefore, it does not impede deep breathing. Postoperative hemorrhage with hip surgery is rare because bleeding at the operative site is not covert. The incidence of wound infection is no greater than with other postoperative clients.

Sinus tachycardia

Rapid regular rhythm originating in the SA node. Rate: 100-150bpm or more Causes: excessive caffeine, exercise, anxiety, fever, shock, meds, HF, recreational drugs, tobacco use. Clinical manifestations: occasional palpitations, many patients are asymptomatic, decreased B/P, angina, cardiovascular disease.

A client who has type 1 diabetes is admitted to the hospital for major surgery. Before surgery the client's insulin requirements are elevated but well controlled. Postoperatively, the nurse anticipates that the client's insulin requirements will: Multiple choice question Decrease Fluctuate Increase sharply Remain elevated

Remain elevated Emotional and physical stress may cause insulin requirements to remain elevated in the postoperative period. Insulin requirements will remain elevated rather than decrease. Fluctuating insulin requirements usually are associated with noncompliance, not surgery. A sharp increase in the client's insulin requirements may indicate sepsis, but this is not expected.

Active transport

Require cellular energy, moving molecules against pressure (ex: sodium potassium pump)

According to Elizabeth Kubler-Ross, all of the following are considered stages of grief except:

Resentment Elizabeth Kubler-Ross, a psychiatrist, proposed a model that describes 5 stages commonly seen in those experiencing grief. These stages, which can occur in any order, include: denial, anger, bargaining, depression, and acceptance. The acronym "DABDA: can be used to help recall the 5 stages of grief.

Care for the patient with active TB involves wearing a:

Respirator mask To prevent droplet spread of TB, health care workers must DON an appropriately fitting respirator mask. Surgical masks do not provide an appropriate seal to prevent the transmission of droplets, and therefore are not recommended in the care of the patient with active TB. Hospitals and health care organizations often conduct fit testing for TB respirator masks to determine the appropriate size mask for each employee.

A client is admitted to the hospital after sustaining serious burns that involve a large surface of the skin. The nurse is caring for the client during the emergent phase after the injury. Which nursing objective is the priority during this phase? Multiple choice question Alleviate pain Prevent infection Replace blood loss Restore fluid volume

Restore fluid volume In the first 48 hours after a severe burn, fluid moves into the tissues surrounding the injured area. Fluid also is lost in drainage and from evaporation; this fluid loss results in a decreased circulating blood volume, which can cause hypovolemic shock. Although pain relief is an important aspect in the care of clients with burns, the immediate priority is to replace fluid losses to prevent death. The burn wound is considered sterile for the first 24 hours; if fluid losses are not replaced immediately, the client may die before the development of an infection. Blood loss usually is minimal; the loss of fluid, colloids, and electrolytes is what causes the hypovolemia.

A client with cancer of the prostate requests the urinal at frequent intervals but either does not void or voids in very small amounts. What does the nurse conclude is most likely the causative factor? Multiple choice question Edema Dysuria Retention Suppression

Retention An enlarged prostate constricts the urethra, interfering with urine flow and causing retention. When the bladder fills and approaches capacity, small amounts can be voided, but the bladder never empties completely. Edema does not cause the client to void frequently in small amounts. Dysuria is painful or difficult urination, which is not part of the client's responses. The urge to void is caused by stimulation of the stretch receptors as the bladder fills with urine; in suppression, little or no urine is produced.

S/S hyponatremia ("SALT LOSS")

S - seizures & stupor A - ABD cramping & confusion L - Lethargic T - Tendon reflexes, amnesia, trouble contracting L - Loss of urine & appetite O - orthostatic hypotension, overactive bowel sounds S - Shallow respiratrions (happen late do to skeletal muscle weakness) S - Spasms of muscles

B-type natriuretic peptide (BNP)

Secreted by the heart in response to ventricular expansion and pressure overload. BNP > 100 = HF

Before a client has a cardiac catheterization, an electrocardiogram (ECG) is performed and hypokalemia is suspected. The nurse expects that the diagnosis will be confirmed by which diagnostic test? Multiple choice question Complete blood count Serum potassium level X-ray film of long bones Blood cultures times three

Serum potassium level Hypokalemia is suspected when the T wave on an ECG tracing is depressed or flattened; a serum potassium level less than 3.5 mEq/L indicates hypokalemia. Complete blood count, x-ray film of long bones, and blood cultures times three will have no significance in the diagnosis of a potassium deficit.

When teaching about the dietary control of gout, the nurse evaluates that the dietary teaching is understood when the client states; "I will avoid eating: Multiple choice question Eggs." Shellfish." Fried poultry." Cottage cheese."

Shellfish." Shellfish contains more than 100 mg of purine per 100 grams. Eggs, fried poultry, and cottage cheese are low in purine.

Give an example of an anti-leukotriene agent. How do they work? (Drugs used for URIs)

Singluair (Montelukast) - work at the cellular level to reduce asthma attacks. 1. Side effects: headaches, dizziness, insomnia

A client who is suspected of having salmonellosis asks the nurse how the diagnosis is confirmed. The nurse responds that the medical diagnosis is established by a: Multiple choice question Urinalysis Stool culture Febrile agglutinin test Complete blood count

Stool culture The Salmonella bacilli can be visualized via microscopic examination of stool. Although a urinalysis might be done, it is not definitive for the diagnosis of salmonellosis. Although a febrile agglutinin test might be done, it is not definitive for the diagnosis of salmonellosis. Although a complete blood count might be done, it is not definitive for the diagnosis of salmonellosis.

A client has laparoscopic surgery to remove a calculus from the common bile duct. What postoperative client response indicates to the nurse that bile flow into the duodenum is reestablished? Multiple choice question Stools become brown Liver tenderness is relieved Colic is absent after ingestion of fats Serum bilirubin level returns to the expected range

Stools become brown The return of brown color to the stool indicates that bile is entering the duodenum and being converted to urobilinogen by bacteria. Liver tenderness is unrelated to bile flow. The absence of biliary colic is related to the removal of the calculus, not the flow of bile. The serum bilirubin level is not affected.

A patient is receiving treatment with a continuous heparin infusion for a DVT. They have a PTT level checked, which comes back Supra-therapeutic. Based on the results the nurse will:

Stop the infusion Patients receiving treatment with heparin infusion will have blood levels monitored using partial thromboplastin time, or PTT. The PTT level will determine whether the rate of infusion will need to be changed. For a patient who has a Supra-therapeutic PTT level, the infusion will be stopped and then restarted at a lower rate. Failure to respond to higher than normal bleeding times can lead to exessive bleeding and shock.

Supraventricular tachycardia (SVT)

Sudden onset of a rapid heart beat. Originates in the atria. Typically not associated with heart diseases. rate: 150-250bpm causes: meds, alcohol, mitral valve prolapse, emotional stress, smoking, hormone imbalance.

After three months of rehabilitation after a craniotomy, a client continues to have motor speech difficulties. To promote the client's use of speech the nurse should: Multiple choice question Support the client's efforts to communicate Correct verbal mistakes immediately Use simple words with short sentences Explain why the client is having difficulty speaking

Support the clients efforts to communicate Recognition of effort is motivating. Correcting mistakes may decrease both self-esteem and motivation. The problem is a motor, not a sensory (receptive), problem. Explaining the reason for the difficulty may decrease self-esteem and motivation.

A client with rheumatoid arthritis asks the nurse about ways to decrease morning stiffness. The nurse should suggest: Multiple choice question Wearing loose but warm clothing Planning a short rest break periodically Avoiding excessive physical stress and fatigue Taking a hot tub bath or shower in the morning

Taking a hot tub bath or shower in the morning Moist heat increases circulation and decreases muscle tension, which help relieve chronic stiffness. Although wearing loose but warm clothing is advisable for someone with arthritis, it does not relieve morning stiffness. Inactivity promotes stiffness. Avoiding excessive physical stress and fatigue is related to muscle fatigue, not to stiffness of joints.

The organization responsible for promoting safer, higher quality care among hospital organizations in addition to evaluating and providing accreditation is known as:

The joint commission (TJC) The Joint Commission is a non-profit organization that works to promote safer, higher quality care in hospital organizations. The Joint Commission is also responsible for evaluating and designating organizations with accreditation. Finally, the Commission publishes annual patient safety goals in order to improve overall patient safety.

A nurse reviews the clients electrolyte laboratory report and notes that the potassium level is 3.2. What would the nurse most likely note on the electrocardiogram As a result of the lab value?

The nurse would most likely note A serum potassium level lower than 3.5 indicate hypokalemia. Potassium deficiency is a common electrolyte imbalance and is potentially life-threatening. EKG changes Include inverted T-waves, ST segment depression, and prominent U-waves.

Structures that protect and support the developing fetus during pregnancy involve:

The placenta, membranes, the umbilical cord, and amniotic fluid.

Blood flows into the heart muscle by what two vessels?

The right and left coronary arteries. They wrap around the myocardium and bring O2 and nutrients. They have tiny branches that provide collateral circulation.

A nurse is monitoring a postpartum client in the fourth stage of labor. Which of the following findings, If noted by the nurse, would indicate a complication related to a laceration of the birth canal?

The saturation of more than one peripad per hour In the first 24 hours after birth, the uterus will feel like a firmly contracted ball, roughly the size of a large grape fruit. One can easily locate the uterus at the level of the umbilicus. Saturation of more than one peripad per hour is considered excessive in the early postpartum period.

Give an example of a Xanthine derivate. How does it work? (Drugs used for lower respiratory disorders)

Theophlline (Theo-Dur) - consists of plant alkaloids: Theobromine and THEOPHYLLINE (only used as a bronchodilator) 1. Slow onset of action 2. Causes the smooth muscle in the lungs to relax , gives vessels increased air movement in and out of the lungs 3. Side effects: Increased HR, N/V, increased blood glucose, anorexia

Jaundice

There are two types to this condition (physiologic jaundice and pathologic jaundice). 1. Physiologic jaundice treatment: sunlight * an increased level of RBCs occurs. 2. Pathologic jaundice * often seen within 24 hrs. occurs with blood incompatability

A client says, "I take baking soda in water when I get heartburn." The nurse suggests an antacid containing aluminum and magnesium hydroxide instead of baking soda. What is the advantage these antacids have over baking soda? Multiple choice question They contain little, if any, sodium. Absorption by the stomach mucosa is markedly enhanced. There is no direct effect on the systemic acid-base balance when taken as directed. Fewer side effects, such as diarrhea or constipation, are experienced when they are used properly.

There is no direct effect on the systemic acid-base balance when taken as directed. Nonsystemic antacids are not readily absorbed, so they do not alter acid-base balance. Sodium bicarbonate is absorbed and can alter acid-base balance. These preparations do contain sodium. Nonsystemic antacids are insoluble and not readily absorbed. Diarrhea and constipation are side effects of nonsystemic antacids.

An 80-year-old male presents to the emergency department with dyspnea and a history of COPD. The licensed practical nurse teaches him about which type of the following positions to relieve dyspnea?

Tripod Proper positioning can provide relief for patients with COPD. The tripod position, in which the patient sits or stands leaning forward with the arms supported, forces the diaphragm down and forward and stabilizes the chest while reducing the work of breathing. Purse-lipped breathing may also be encouraged to control dyspnea and shortness of breath.

A nurse is evaluating a client's response to fluid replacement therapy. Which clinical finding indicates adequate tissue perfusion to vital organs? Multiple choice question Urinary output of 30 mL in an hour Central venous pressure reading of 2 mm Hg Baseline pulse rate of 120 that decreases to 110 beats/min within a 15-minute period Baseline blood pressure of 50/30 that increases to 70/40 mm Hg within a 30-minute period

Urinary output of 30 mL in an hour A urinary output rate of 30 mL/hour is considered adequate for perfusion of the kidneys, heart, and brain. A central venous pressure reading of 2 mm Hg indicates hypovolemia. A baseline pulse rate of 120 that decreases to 110 beats/min within a 15-minute period and a baseline blood pressure of 50/30 that increases to 70/40 mm Hg within a 30-minute period indicate improvement but not necessarily adequate tissue perfusion.

A nurse is caring for a client who is positive for hepatitis A. Which precautions should the nurse take? Multiple choice question Wear a gown when entering the client's room. Use caution when bringing in the client's food. Use gloves when removing the client's bedpan. Wear a protective mask when entering the client's room.

Use gloves when removing the client's bedpan. The virus is present in the stool of clients with hepatitis A; therefore, standard precautions should be followed when handling excretions. The virus also may be present in urine and nasotracheal secretions. The Centers for Disease Control and Prevention (CDC) indicates that only standard precautions are necessary when caring for a client who is positive for the presence of hepatitis A; if a client is incontinent or using an incontinence device, the CDC recommends contact precautions be implemented. Bringing food to a client requires no precautions; however, disposable utensils should be used and utensils discarded following standard precautions because the client's nasotracheal secretions contain the virus. Hepatitis A usually is not transmitted via the air.

A nurse is monitoring a client in labor. The nurse suspects umbilical cord compression if which of the following is noted on the external monitor tracing during contraction?

Variable declarations Variable declarations occur if the umbilical cord is compressed, reducing blood flow between the placenta and the fetus. Early declarations result from pressure on the fetal head during a contraction. Late declarations are an ominous pattern in labor because they suggest uteroplacental insufficiency during a contraction.

The fourth stage of labor, the time of stabilization, requires careful nursing assessment of the mother and assessment of what?

Vital signs and perform fundal checks to detect excessive blood loss.

The nurse teaches a group of clients that nutritional support of natural defense mechanisms indicates the need for a diet high in: Multiple choice question Essential fatty acids Dietary cellulose and fiber Tryptophan, an amino acid Vitamins A, C, E, and selenium

Vitamins A, C, E, and selenium Vitamins A, C, E, and selenium stimulate the immune system. The role of fatty acids in natural defense mechanisms is uncertain. Dietary cellulose and fiber have no known effect on natural defense mechanisms. Tryptophan has no known effect on natural defense mechanisms.

A nurse is caring for a client who has been taking several antibiotic medications for a prolonged time. Because long-term use of antibiotics interferes with the absorption of fat, the nurse anticipates a prescription for: Multiple choice question High fat diet Supplemental cod liver oil Total parenteral nutrition (TPN) Water-soluble forms of vitamins A and E

Water-soluble forms of vitamins A and E Vitamins A, D, E, and K are known as fat-soluble vitamins because bile salts and other fat-related compounds aid their absorption. A high fat diet will not achieve the uptake of fat-soluble vitamins in this client. Supplemental cod liver oil will not achieve the uptake of fat-soluble vitamins in this client. TPN is unnecessary; a well-balanced diet is preferred. Water-miscible forms of vitamins A and E can be absorbed with water-soluble nutrients.

Intravascular space

Where the inflammatory response is triggered. Venous and arterial blood flow.

Explain how antihistamines work? (Drugs used for URIs)

Work best when given early in an allergic reaction. Antihistamines have a drying effect: dries mucus membranes all over the body.

Your patient is scheduled for an ileostomy. You are providing patient teaching regarding the ileostomy care. What would you teach your patient?

Your ileostomy will be most active first thing in the morning and before meals.

The type of injection technique used to prevent staining and damage to the skin and subcutaneous tissue is known as:

Z-track The Z-track technique is a type of injection used to prevent staining and damage to the skin and subcutaneous tissue. It involves displacing the skin lateral to the injection site prior to injecting the medication. The common injection sites for this technique is the upper outer region of the gluteal.

hypertonic solution

a solution that causes a cell to shrink because of osmosis

isotonic solution

a solution whose solute concentration is equal to the solute concentration inside a cell

The nurse finds a client with schizophrenia lying under a bench in the hall. The client says, "God told me to lie here." What is the best response by the nurse? a) "I didn't hear anyone talking; come with me to your room." b) "What you heard was in your head; it was your imagination." c) "Come to the dayroom and watch television; you'll feel better." d) "God wouldn't tell you to lie there in the hall. God wants you to behave reasonably."

a) "I didn't hear anyone talking; come with me to your room." The nurse is focusing on reality and trying to distract and refocus the client's attention. "What you heard was in your head; it was your imagination" is too blunt and belittling; this approach rarely is effective. "Come to the dayroom and watch television; you'll feel better" is false reassurance; the nurse does not know that the client will feel better. "God wouldn't tell you to lie in the hall; God wants you to behave reasonably" may be interpreted as belittling or an attempt to convince the client that the behavior is irrational, which is usually ineffective.

The nurse repeats instructions to prevent nursing bottle caries in an infant. Which statement by the mother indicates the need for additional teaching? a) "I should use sugar-coated pacifiers for the child." b) "I should use water in the bedtime bottle for the child." c) "I should avoid using juice in the bedtime bottle for the child." d) "I should clean the child's oral cavity before putting down the child to sleep."

a) "I should use sugar-coated pacifiers for the child." Nursing bottle caries are common in bottle-fed infants, as they tend to use milk or juice as their last feed. This feed remains in the mouth, and, as salivary secretion is lessened during the night, results in dental caries. Using sugar-coated pacifiers increases the risk of dental caries, because the sugar on the pacifiers promotes growth of microorganisms. Sugar-coated pacifiers should be avoided. Using water instead of juices and milk in the bedtime feeding bottle decreases the risk of dental caries. The child's oral cavity should be cleansed before putting the child to bed, to prevent cavities. Juices should be avoided in the bedtime feeding bottles, because the glucose in the juice can promote the formation of cavities.

Potassium supplements are prescribed for a client receiving diuretic therapy. What client statement indicates that the teaching about potassium supplements is understood? a) "I will report any abdominal distress." b) "I should use salt substitutes with my food." c) "The drug must be taken on an empty stomach." d) "The dosage is correct if my urine output increases."

a) "I will report any abdominal distress." Potassium supplements can cause gastrointestinal ulceration and bleeding. Most salt substitutes contain potassium, and their use with potassium supplements can cause hyperkalemia. Because they can be irritating to the stomach, potassium supplements should not be taken on an empty stomach. An increase in urine output is the therapeutic effect of diuretic therapy, not potassium supplements. An adverse effect of potassium supplements is oliguria.

A client hospitalized with a severe myocardial infarction tells the nurse, "My life is over. I may as well just give up." What is the best response by the nurse? a) "You feel your life is over?" b) "Have you nothing to live for?" c) "We are not going to let you die." d) "Everything will be fine. Do not worry."

a) "You feel your life is over?" The response "You feel your life is over?" invites the client to expand on the statement, and feelings and fears may be discussed. The response "Have you nothing to live for?" addresses the future rather than the present; the statement may make the client defensive and may close off communication. The response "We are not going to let you die" is not a client-centered response and is false reassurance; the nurse does not know whether the client will recover. The nurse's statement "Everything will be fine. Do not worry." is also giving the client false reassurance. Test-Taking Tip: Be aware that information from previously asked questions may help you respond to other examination questions.

A patient diagnosed with anemia has activity intolerance due to hypoxia and dyspnea. Which nursing interventions should the nurse perform to increase activity tolerance in the patient? Select all that apply. a) Administer oxygen to the patient. b) Encourage the patient to meet visitors. c) Place articles within easy reach of the patient. d) Encourage the patient to accept phone calls. e) Assist the patient with self-care activities as needed

a) Administer oxygen to the patient. c) Place articles within easy reach of the patient. e) Assist the patient with self-care activities as needed. Anemic patients may have activity intolerance, ineffective tissue perfusion, and impaired gas exchange. The nurse should administer oxygen as ordered to relieve dyspnea. Placing articles within easy reach of the patient reduces physiological demands on the body. The nurse should assist the patient with self-care activities as needed. The nurse should encourage the patient to limit visitors, phone calls and unnecessary activities to conserve energy.

A client is admitted with thrombocytopenia. What specific nursing actions are appropriate to include in the plan of care for this client? a) Avoid intramuscular injections. b) Institute neutropenic precautions. c) Monitor the white blood cell count. d) Administer prescribed anticoagulants. e) Examine the skin for ecchymotic areas.

a) Avoid intramuscular injections. e) Examine the skin for ecchymotic areas. Intramuscular injections should be avoided because of the increased risk of bleeding and possible hematoma formation. Decreased platelets increase the risk of bleeding, which leads to ecchymoses. Neutropenic precautions are for clients with decreased white blood cells (WBCs), not platelets. Thrompbocytopenia refers to decreased platelets, not WBCs. Anticoagulants are contraindicated because of the increased bleeding risk.

A client is receiving hydrochlorothiazide (HCTZ). What should the nurse monitor to best determine the effectiveness of the client's hydrochlorothiazide therapy? a) Blood pressure b) Decreasing edema c) Serum sodium level d) Urine specific gravity

a) Blood pressure Diuretics promote urinary excretion, which reduces the volume of fluid in the intravascular compartment, thus lowering blood pressure. Assessing the extent of edema is subjective and difficult; blood pressure is an objective assessment that measures intravascular pressure. The serum sodium level remains stable unless the dosage is excessive; an altered sodium level is not a therapeutic response. Although specific gravity decreases with increased urinary output, this does not reflect the desired reduction in intravascular pressure.

Which nutrient-related problem is common to a newborn infant, a client after a cholecystectomy, and a client receiving anticoagulant therapy after a myocardial infarction? a) Blood-clotting function of vitamin K b) Neuromuscular function of vitamin B 1 c) Calcium-absorbing function of vitamin D d) Hemoglobin-forming function of vitamin B 12

a) Blood-clotting function of vitamin K A neonate lacks the ability to produce vitamin K because of a lack of bacteria in the intestine. After a cholecystectomy a client experiences interference with absorption of the fat-soluble vitamin K because of disruption in bile flow. A client who is receiving anticoagulants experiences inhibition of vitamin K-dependent activation of clotting factors. Neuromuscular function of vitamin B 1, Calcium-absorbing function of vitamin D, and Hemoglobin-forming function of vitamin B 12 are not common nutritional problems for these clients.

A nurse is planning to teach facts about hyperglycemia to a client with the diagnosis of diabetes. What information should the nurse include in the discussion about what causes diabetic acidosis? a) Breakdown of fat stores for energy b) Ingestion of too many highly acidic foods c) Excessive secretion of endogenous insulin d) Increased amounts of cholesterol in the extracellular compartment

a) Breakdown of fat stores for energy In the absence of insulin , which facilitates the transport of glucose into cells, the body breaks down proteins and fats to supply energy; ketones, a by-product of fat metabolism, accumulate, causing metabolic acidosis (pH below 7.35). The pH of food ingested has no effect on the development of acidosis. The opposite of excessive secretion of endogenous insulin is true. Cholesterol level has no effect on the development of acidosis.

A nurse is caring for a patient receiving chemotherapy who is on neutropenic precautions. The unlicensed assistive personnel reports to the nurse that the patient's temperature is 38.8° C. What should the nurse's initial response be? a) Call the health care provider b) Offer the patient warm blankets c) Administer an antipyretic medication d) Place cool, wet towels in the patient's axilla and groin areas

a) Call the health care provider The patient on neutropenic precautions is at an increased risk for infection, so the nurse should alert the health care provider immediately. Cool, wet towels and an antipyretic may be necessary, but the health care provider should be alerted first. Warm blankets are not appropriate at this time.

A nurse sees the telemetry monitor of a patient change from a heart rate of 85 with regular rhythm to a heart rate of 250 that still maintains a regular rhythm. The nurse hurries to the patient's room and expects the patient to exhibit which symptom(s)? Select all that apply. a) Dyspnea b) Hypertension c) Hyperthermia d) Complains of chest pain e) Report of heart palpitations

a) Dyspnea d) Complains of chest pain e) Report of heart palpitations The patient is experiencing supraventricular tachycardia and is likely to have dyspnea, chest pain, and heart palpitations. The patient may not necessarily have hypertension or hyperthermia.

A client with scleroderma reports having difficulty chewing and swallowing. What should the nurse recommend to safely facilitate eating? a) Eat a mechanical soft diet b) Liquefy food in a blender c) Take frequent sips of water with meals d) Use a local anesthetic mouthwash before eating

a) Eat a mechanical soft diet Scleroderma causes chronic hardening and shrinking of the connective tissues of any organ of the body, including the esophagus and face; a mechanical soft diet includes foods that limit the need to chew and are easier to swallow. Liquefied foods are difficult to swallow; esophageal peristalsis is decreased, and liquids are aspirated easily. Taking frequent sips of water with meals will not help; it is equally difficult to swallow solids and liquids, and aspiration may result. Using a local anesthetic mouthwash before eating is not necessary; oral pain is not associated with scleroderma.

The nurse recommends human immunodeficiency virus (HIV) testing for a patient. Which risk factors the nurse might have found in this patient? Select all that apply. a) Engaging in anal intercourse b) Working as a prostitute 2 years ago c) Being in a monogamous relationship d) Testing positive for a chlamydial infection e) Sharing needles when using intravenous drugs

a) Engaging in anal intercourse b) Working as a prostitute 2 years ago d) Testing positive for a chlamydial infection e) Sharing needles when using intravenous drugs The HIV virus is transmitted through blood and body fluids, including semen and vaginal secretions. Patients who have high-risk behaviors should be tested for HIV. Patients with a history of prostitution have frequent exposure to semen and vaginal secretions. Patients with other sexually transmitted diseases, like chlamydia, are at higher risk for HIV infection. Since HIV is spread through blood and body fluids, sharing needles when injecting drugs also increase the patient's risk. Patients who engage in anal intercourse, whether male or female, are at high risk for contracting HIV. Patients in a monogamous relationship have only one sexual partner and are therefore at low risk of acquiring HIV.

A patient reports a loss of appetite with severe headache and lethargy. On examination, the nurse finds that the patient has dry mucous membrane, urinary output of 400 mL/24hour, and elevated blood urea nitrogen (BUN) and serum creatinine. What medications will the nurse expect in the patient's prescription? a) Furosemide (Lasix) b) Flutamide (Eulexin) c) Doxazosin (Cardura) d) Propantheline (Pro-Banthine)

a) Furosemide (Lasix) Loss of appetite associated with severe headache and lethargy are the symptoms of acute renal failure. It is assessed by the presence of dry mucous membrane, decreased urinary output to less than 400 mL/24 hours. It is diagnosed by blood urea nitrogen (BUN) and creatinine test of the urinary sample. Furosemide (Lasix), a diuretic, is prescribed for acute renal failure to increase the urinary output and prevent edema. Flutamide (Eulexin) is a nonsteroidal anti androgenic drug prescribed for prostate cancer. Doxazosin (Cardura) is prescribed for benign prostate hypertrophy (BPH). Doxazosin (Cardura) does not increase urinary output. Propantheline (Pro-Banthine) is prescribed for patients with hydronephrosis.

A client's diet is modified to eliminate foods that act as cardiac stimulants. The nurse should teach the client to avoid what foods? a) Iced tea b) Red meat c) Club soda d) Hot cocoa e) Chocolate pudding

a) Iced tea d) Hot cocoa e) Chocolate pudding Tea contains caffeine, which stimulates catecholamine release and acts as a cardiac stimulant; tea should be avoided. Hot cocoa contains chocolate, which contains caffeine; it stimulates catecholamine release and acts as a cardiac stimulant. Cocoa should be avoided. The chocolate in chocolate pudding has a high caffeine content, which may stimulate catecholamine release and act as a cardiac stimulant; chocolate should be avoided. Red meat does not stimulate the myocardium; however, it should be decreased or eliminated if serum cholesterol levels are elevated. Club soda does not contain caffeine and does not stimulate the myocardium; however, most club sodas contain sodium, which promotes fluid retention and should be avoided by a client with a cardiac condition.

After emergency surgery, the nurse teaches a client how to use an incentive spirometer. What client behavior indicates to the nurse that the spirometer is being used correctly? a) Inhales deeply through the mouthpiece, relaxes, and then exhales. b) Inhales deeply, seals the lips around the mouthpiece, and exhales. c) Uses the incentive spirometer for 10 consecutive breaths per hour. d) Coughs several times before inhaling deeply through the mouthpiece.

a) Inhales deeply through the mouthpiece, relaxes, and then exhales. Inhaling deeply through the mouthpiece, relaxing, and then exhaling are correct techniques; deep inhalation promotes alveolar expansion, and exhalation promotes lung recoil. Inhaling deeply, sealing the lips around the mouthpiece, and exhaling are incorrect techniques; inhalation should occur through the mouthpiece. The breaths should not be taken in succession; they should be spaced by several normal breaths to avoid fatigue. Coughing is done after deep breathing.

What clinical manifestations does a nurse expect a client with systemic lupus erythematosus (SLE) most likely to exhibit? a) Joint pain b) Facial rash c) Pericarditis d) Weight gain e) Hypotension

a) Joint pain b) Facial rash c) Pericarditis SLE is a chronic, autoimmune disease that affects connective tissue; joint pain is common. A butterfly rash is characteristic of SLE. Pericarditis is the most common cardiac indicator of SLE. Weight loss, not gain, is a classic sign of SLE because of gastrointestinal effects. Renal impairment with SLE may cause hypertension, not hypotension.

Your elderly patient with CHF was admitted with dehydration. IV fluids have been running at the rate of 125cc per hour for 18 hours. You assess the patient and note that the patient's Spo2 has decreased from 97% on room air to 91% on room air. The patient is coughing and you auscultate coarse crackles to both lung bases. The patient's legs and feet have 2+ edema. The patient has a foley and you note that the patient has only produced 200cc of urine in the last 12 hours. You should anticipate administering which of the following medications? a) Lasix b) digoxin c) solumedrol d) morphine

a) Lasix The patient's S/S are compatible with CHF. In this case, the amount of IV fluids the patient was given to counteract the dehydration likely exacerbated the patient's CHF. Administering a diuretic will help get rid of excess fluid in the lungs and decrease edema. You would also anticipate decreasing the rate of the IV infusion.

A nurse is counseling a patient to quit smoking to reduce the risk of cancer. Which types of cancer should the patient understand that smoking is associated with? Select all that apply. a) Lung b) Breast c) Uterus d) Bladder e) Esophagus f) Lymphoma

a) Lung c) Uterus d) Bladder e) Esophagus Smoking is associated with cancers of the lung, uterus, bladder, and esophagus. It is not associated with breast cancer and lymphoma.

A nurse is caring for a patient who has recently developed symptoms of increased intracranial pressure (ICP) including decreased level of consciousness. The nurse anticipates administering which medication to emergently decrease intracranial pressure? a) Mannitol b) Phenytoin (Dilantin) c) Midazolam (Versed) d) Cimetidine (Tagamet)

a) Mannitol Mannitol is an osmotic diuretic that pulls water from the brain tissue to decrease ICP. Versed and Dilantin are commonly used in the treatment of increased ICP, but these are used to sedate and control seizures, respectively, not decrease ICP. Tagamet is prescribed if dexamethasone (Decadron) is prescribed.

The nurse concludes that a client is experiencing hypovolemic shock. Which physical characteristic supports this conclusion? a) Oliguria b) Crackles c) Dyspnea d) Hypertension

a) Oliguria Urine output decreases to less than 20 to 30 mL/h because of decreased renal perfusion secondary to a decreased circulating blood volume. Crackles are associated with pulmonary edema, not hypovolemic shock. Dyspnea may be associated with hypervolemia, not hypovolemia, as well as with pulmonary edema and respiratory disorders. Hypotension, not hypertension, is associated with hypovolemic shock.

A client is brought to the emergency department with moderate substernal chest pain radiating to the inner aspect of the left arm, unrelieved by rest and nitroglycerin. The pain is associated with slight nausea and anxiety. What is the priority nursing intervention for this client? a) Provide pain medication b) Obtain an electrocardiogram (ECG) c) Transfer to the coronary care unit d) Have a blood specimen drawn for enzyme studies

a) Provide pain medication Providing pain medication (usually morphine) for comfort reduces anxiety and subsequently decreases catecholamine release, indirectly decreasing myocardial oxygen requirements. Obtaining an electrocardiogram is important, but pain relief is the priority; the ECG is significant to examine for progressive myocardial changes. The client's condition should be stabilized before transfer; relief of pain facilitates stabilization. Securing blood for enzyme studies is not an emergency intervention, although a blood sample for cardiac enzymes is important for a definitive diagnosis.

A patient is undergoing a skin test for allergies. Which symptoms are associated with an anaphylactic reaction? Select all that apply. a) Red blotches b) Itching and urticaria c) High blood pressure d) Diarrhea and vomiting e) Erythema and angioedema

a) Red blotches b) Itching and urticaria e) Erythema and angioedema Red blotches, itching, urticaria, erythema, and angioedema are the common symptoms associated with anaphylactic reactions. During this reaction, vasodilation occurs and blood is lost from the vascular system; the client becomes hypotensive, not hypertensive. Diarrhea and vomiting are not directly related to this reaction.

How should the nurse make the bed of a client who is in the acute phase after a myocardial infarction? a) Replace the top linen and only the necessary bottom linen. b) Lift the client from side to side while changing the bed linen. c) Change the linen from top to bottom without lowering the head of the bed. d) Slide the client onto a stretcher to remake the bed and then slide the client back to the bed.

a) Replace the top linen and only the necessary bottom linen. Until a client's condition has reached some degree of stability after a myocardial infarction, routine activities such as changing sheets are avoided so that the client's movements will be minimized and the cardiac workload reduced. Lifting the client from side to side while changing the bed linen is contraindicated because it increases oxygen consumption and cardiac workload; also, it may strain the health team members who are lifting the client. Changing all the linen causes unnecessary movement, which increases oxygen demand and makes the heart work harder. Any activity is counterproductive to rest; rest must take precedence so that the cardiac workload is reduced. Test-Taking Tip: Identify option components as correct or incorrect. This may help you identify a wrong answer. Example: If you are being asked to identify a diet that is specific to a certain condition, your knowledge about that condition would help you choose the correct response (e.g., cholecystectomy = low fat, high protein, low-calorie diet ).

A client is admitted with the diagnosis of tetanus. For which clinical indicators should the nurse assess the client? a) Restlessness b) Muscular rigidity c) Atony of facial muscles d) Respiratory tract spasms e) Spastic voluntary muscle contractions

a) Restlessness b) Muscular rigidity d) Respiratory tract spasms e) Spastic voluntary muscle contractions Toxins from bacilli invade nervous tissue, causing restlessness. Toxins from bacilli invade nervous tissue, causing muscle spasms and muscular rigidity . Toxins from the bacillus invade nervous tissue; respiratory spasms may result in respiratory failure. Toxins from bacilli invade nervous tissue, causing spastic contraction of voluntary muscles. Tetanus causes spasms of facial muscles, resulting in a grotesque grinning expression (risus sardonicus) and spasms of masticatory muscles (trismus), not atony of facial muscles.

An older client with dementia of the Alzheimer's type is residing in a nursing home. When in bed, the client consistently is found sleeping in the semi-Fowler position. What area of the client's body does the nurse determine is at the greatest risk for developing a pressure ulcer? a) Sacrum b) Scapulae c) Ischial spine d) Greater trochanter

a) Sacrum The sacrum is the center of the greatest body mass; an elevated torso exerts pressure toward this area. Although the scapulae is at risk, it does not bear the greatest body weight as when the client is in the semi-Fowler position. The ischial spine bears the greatest pressure when the client is in an upright sitting position. Greater trochanter is at risk when the client is in a side-lying position.

While playing with her child, a mother takes a toy from the child and hides it. She then observes that the child begins to look for the hidden object. According to Piaget, which developmental stage is indicated by this behavior? a) Sensorimotor b) Preoperational c) Formal operations d) Concrete operations

a) Sensorimotor The ability to recognize the permanence of objects is seen in the sensorimotor stage. The child has learned that objects in the environment are still present even when they are not seen. During the preoperational stage, the child develops symbolic mental abilities. A child learns to think abstractly during the stage of formal operations. Logical and objective thinking are present in the stage of concrete operations.

A nurse receives a call from the laboratory reporting a critical value from the complete blood count with differential of 15% bands. The nurse knows this value is indicative of which condition? a) Sepsis b) Cellulitis c) Dehydration d) Fluid volume overload

a) Sepsis A laboratory result of greater than 8% bands is indicative of a serious infection, such as sepsis, in which the bone marrow has used up its reserve. Cellulitis is not as serious as sepsis, although it can progress to sepsis if not managed appropriately. Dehydration and fluid volume overload do not lead to an excess of bands above 8%.

The nurse is caring for a child who has undergone splenectomy 1 day ago. The nurse finds that the child also has a respiratory tract infection. What further complication does the nurse expect to monitor for in the child? a) Septicemia b) Pancytopenia c) Rheumatic fever d) Myasthenia gravis

a) Septicemia Septicemia is a serious, life-threatening infection that quickly worsens. It can be caused due to infections occurring throughout the body, including infections in the lungs, abdomen, and urinary tract. A splenectomy in children is very dangerous and may lead to septicemia, even due to a simple respiratory tract infection. Pancytopenia is the reduction of red blood cell, white blood cell, and platelet count in the body; it is not only associated with the splenectomy. Rheumatic fever and myasthenia gravis are autoimmune diseases and not associated with the splenectomy.

A pregnant client who has a history of cardiac disease asks how she can relieve her occasional heartburn. The nurse should instruct the client to avoid antacids containing: a) Sodium b) Calcium c) Aluminum d) Magnesium

a) Sodium If the client consumes more than the usual daily sodium intake, excess fluid retention results; this will increase the cardiac workload. Antacids that do not contain sodium do not cause fluid retention; it is best for this client to seek medical advice before taking an antacid.

A client has a thyroidectomy for cancer of the thyroid. To evaluate for nerve injury that may be the result of surgery-related trauma, the nurse assesses the client's ability to: a) Speak b) Swallow c) Purse the lips d) Turn the head

a) Speak The laryngeal nerve is close to the operative site and can be damaged inadvertently. Loss of the gag reflex occurs with general anesthesia; the ability to swallow signifies its return. The ability to purse the lips tests the seventh cranial (facial) nerve, which is not affected in thyroid surgery. The nerves involved in turning the head are not near the thyroid gland.

A nurse is caring for a patient with dehydration. The patient is on intravenous fluids with 40 mEq of potassium. The nurse notes that the patient's urine output is less than 30 mL/hr. What is an important next step by the nurse? a) Stop potassium administration and notify the physician. b) Check the patient for insomnia. c) Check if the patient has hypotension. d) Check for anorexia and give the patient food.

a) Stop potassium administration and notify the physician. If the urine output of the patient is less than 30 mL/hr, potassium should not be given. Giving potassium when urine flow is inadequate may cause kidney damage. The nurse will thus stop potassium administration and notify the physician of the urinary output. The patient is checked for insomnia if hypomagnesemia is suspected. Hypotension is assessed to check the presence of hypermagnesemia. Hyperphosphatemia manifests with anorexia, nausea, and vomiting.

A client is hospitalized with a diagnosis of emphysema. The nurse provides teaching and should begin with which aspect of care? a) The disease process and breathing exercises b) How to control or prevent respiratory infections c) Using aerosol therapy, especially nebulizers d) Priorities in carrying out everyday activities

a) The disease process and breathing exercises Clients need to understand the disease process and how interventions, such as breathing exercises, can improve ventilation. Learning to control or prevent respiratory infections is important, but it should be taught later. Although it is helpful to know about aerosol therapy and nebulizers, knowing how to use aerosol therapy, especially nebulizers, should be taught later. Although it is important to teach the client how to set priorities in carrying out everyday activities, this should be taught later. Test-Taking Tip: Being prepared reduces your stress or tension level and helps you maintain a positive attitude.

The nurse is reviewing the cholesterol report of 30-year-old woman and finds it to be normal. Which values did the nurse find in the report? a) Total cholesterol 180 mg/dL; LDL 90 mg/dL; HDL 55 mg/dL b) Total cholesterol 230 mg/dL; LDL 80 mg/dL; HDL 40 mg/dL c) Total cholesterol 240 mg/dL; LDL 140 mg/dL; HDL 60 mg/dL d) Total cholesterol 250mg/dL; LDL 160 mg/dL; HDL 65 mg/dL

a) Total cholesterol 180 mg/dL; LDL 90 mg/dL; HDL 55 mg/dL Total cholesterol of 180 mg/dL, LDL of 90 mg/dL, and HDL of 55 mg/dL indicates normal cholesterol levels in a woman. As total cholesterol less than 200 mg/dL is desirable, LDL less than 100 mg/dL is optimal, and HDL should be between 50-60 mg/dL in a woman. If total cholesterol is 230 mg/dL, LDL is 80 mg/dL, and HDL is 40 mg/dL, this indicates borderline total cholesterol and very low HDL. If total cholesterol is 240 mg/dL, LDL is 140 mg/dL, and HDL is 60 mg/dL, this indicates high total cholesterol, high LDL, and high HDL. If total cholesterol is 250 mg/dL, LDL is 160 mg/dL, and HDL is 65 mg/dL, this indicates very high cholesterol levels.

An LPN/LVN is reading over the nursing care plan for a newly admitted female patient. One nursing concern written is impaired urinary elimination. The LPN/LVN is not certain exactly what that means, but on further reading of the care plan discovers nursing interventions including "remind patient to perform Kegel exercises four times per shift" and "assist patient to toilet every 2 hours." Considering the care plan information, what is the patient most likely experiencing? a) Urinary incontinence (UI) b) Urinary tract infection (UTI) c) Neurogenic bladder: flaccid type d) Neurogenic bladder: spastic type

a) Urinary incontinence (UI) This patient probably is experiencing UI. UI is the involuntary loss of urine from the bladder. It may be permanent or temporary. Although it affects all age groups, it affects older adults more frequently. Kegel exercises and scheduled toileting are two of the interventions that might help the patient achieve continence. A UTI is the presence of microorganisms in the urinary tract structure. Whereas medical care will focus on treating the patient with the correct antibiotics, the primary focus of nursing care of the patient with a UTI is maintaining adequate hydration and hygiene. The patient with a neurogenic bladder has a lesion of the central nervous system that interferes with normal nerve conduction to the urinary bladder; in the flaccid type, caused by a lower motor neuron lesion, the bladder continues to fill and distend, with pooling of urine and incomplete emptying. Because of the loss of sensation, the patient may not even be aware that he or she has a distended bladder. The patient with a neurogenic bladder has a lesion of the central nervous system that interferes with normal nerve conduction to the urinary bladder; in the spastic type, caused by a lesion above the voiding reflex arc, there is a loss of sensation to void and a loss of motor control. There is resultant bladder wall atrophy and a diminished capacity. The bladder releases urine on reflex, with little or no conscious control.

Laboratory results of a client's blood after chemotherapy indicate bone marrow depression. What should the nurse encourage the client to do? a) Use a soft toothbrush. b) Sleep with the head of the bed elevated. c) Increase activity levels and take frequent walks. d) Drink more citrus juices and eat more citrus fruits. e) Read the ingredients in over-the-counter drugs before taking them.

a) Use a soft toothbrush. e) Read the ingredients in over-the-counter drugs before taking them. The gums are vascular tissue and prone to bleed easily if the platelet count is low. Drugs such as ibuprofen (e.g., Motrin, Advil) and salicylates (aspirin) in any analgesic or cold medicine should be avoided because they increase the risk of bleeding by inhibiting platelet function. With bone marrow depression, red blood cells are decreased and the oxygen-carrying capacity of the blood is decreased; this position will not increase the number of red blood cells. With bone marrow depression there is a decrease in red blood cells; rest should be encouraged. Citrus fruits and juices will not change the bone marrow depression; they should be avoided because they are acidic and aggravate stomatitis.

An infant is admitted to the pediatric floor with severe dehydration. Intravenous fluids are started and orders are received to monitor the infant's output. The most accurate way to assess urinary output with an infant is which? a) Weigh diaper before and after soiling b) Weigh the infant every day fully clothed c) Calculate the total dietary intake of fluids d) Calculate the amount of intravenous fluid infusing only

a) Weigh diaper before and after soiling For children who are not toilet trained, wet diapers should be weighed to assess the amount of output. By subtracting the weight (in grams) of a dry diaper from the weight of the wet diaper, the nurse can calculate the actual fluid content of the diaper. An infant should be weighed without any clothes on. However, if a diaper is needed, a dry diaper should be weighed first and that value subtracted from total weight. Calculation of total dietary fluids consumed and total amount of intravenous fluids infused would give the nurse the total intake, not the total output.

Involuntary commitment of a patient with psychiatric illness can be pursued by all of the following EXCEPT: a) a 911 dispatcher b) law enforcement officers c) physician d) family members

a) a 911 dispatcher Involuntary commitment or admission occurs when a patient is admitted to a hospital for psychiatric treatment and admission is not initiated by the patient. There are strict criteria specific to each state or territory that must be met for a patient to be involuntary committed. There are several individual who may pursue involuntary commitment of an individual; these include law enforcement officials such as a judge or police officer, physicians or other health care team members, and family members of the patient. A 911 dispatcher may take a call regarding a patient with a psychiatric emergency, but they would not be responsible for involuntary commitment of the individual.

Incident reports are intended to: a) alert the facility's insurance company of a potential claim b) provide evidence for disciplinary action for the staff member involved c) provide information for the patient record d) allow the opportunity for the nurse to provide his or her opinion regarding the incident

a) alert the facility's insurance company of a potential claim

Your patient is to receive 2 units of packed red blood cells. You are inserting an intravenous in preparation. What gauge of cathlon is recommended to infuse blood? a) at least a 20 gauge b) 16 gauge c) 24 gauge d) size makes no difference

a) at least a 20 gauge It is recommended that at least a 20 gauge cathlon be used for blood transfusion. 16 gauge cathlons may be used for trauma when it is necessary to infuse blood as quickly as possible, but a cathlon this large is unecessary for a non-emergency blood transfusion. A 24 gauge cathlon is small and may result in hemolysis of red blood cells. If there is no other choice, a 24 gauge cathlon can be used, but you should avoid using an infusion pump and should monitor the site carefully for infiltration.

You are caring for a 22 year old female with acute myelogenous leukemia. She is currently receiving chemotherapy and as result has severe stomatitis. In teaching her proper oral care, you reinforce which of the following? a) avoid alcohol-based mouthwashes and toothpastes b) drink hot liquids to promote comfort c) avoid bruising your teeth daily d) remove any blistering or patches that you see

a) avoid alcohol-based mouthwashes and toothpastes Stomatitis is an inflammation of the mucous lining of the mouth, which may involve the cheeks, gums, tongue, lips, and roof or floor of the mouth. It may lead to severe pain and, in cases where a patient cannot eat, may result in nutritional deficiencies. Good hygiene is critical to treating stomatitis. Alcohol based mouthwashes and toothpastes should be avoided, and a soft bristled toothbrush should be used. Alcohol based washes can disrupt the normal protective flora in the mouth. Additionally, white patches or blistering should never be removed ; and tepid , not hot, liquids can provide comfort.

A child has been diagnosed with a congenital birth defect. The father asks the nurse if he could have caused his child to have the defect. The nurse responds correctly to the father when she lists which environmental or genetic factors that may have played a role? Select all that apply. a) diabetes mellitus b) maternal alcoholism c) advanced paternal age d) maternal drug ingestion e) chromosomal abnormalities

a) diabetes mellitus b) maternal alcoholism d) maternal drug ingestion e) chromosomal abnormalities Genetics and maternal ingestion of certain medications may play a role in a congenital birth defect in children. Environmental factors such as intrauterine rubella exposure, maternal alcoholism, diabetes, and advanced maternal age may also play a role in congenital birth defects in children. The paternal or father's age does not play a role in this type of birth defect.

Hypoproteinemia may cause which of the following symptoms? a) edema b) peripheral neuropahty c) chest pain d) all of these e) tachycardia

a) edema Hypoproteinemia is a condition where there is an abnormally low level of protein in the blood. Decreased serum protein decreases the osmotic pressure of the blood, leading to loss of fluid from the intravascular compartment, or the blood vessels, to the interstitial tissues, resulting in edema.

The LPN has just administered atropine IV to a patient with bradycardia. The LPN anticipates all of the following EXCEPT: a) miosis b) urinary retention c) constipation d) xerostomia

a) miosis Atropine is an antimuscarinic agent that is indicted for a variety of uses. Atropine inhibits the action of acetylcholine or other cholinergic stimuli at postganglionic cholinergic receptors. When administered via IV route, it is most commonly for a cardiac indication such as bradycardia. Side effects of atropine include mydriasis (dilated pupils), constipation, urinary retention, and xerostomia (dry mouth). Miosis refers to constricted pupils and would not be considered a side effect of atropine.

The nurse is explaining the importance of breast-feeding to a new mother. Which hormone does the nurse mentions that stimulates milk production and uterine contractility? a) oxytocin b) estrogen c) progesterone d) calcitonin

a) oxytocin Oxytocin is a hormone produced by the posterior pituitary gland. It stimulates milk secretion and uterine contractility. Calcitonin is produced by thyroid. It affects the bone tissue, and regulates calcium and phosphorus blood levels. Estrogen produced by the ovaries stimulates the development of secondary sex characteristics, prepares the uterus for fertilization and fetal development, and stimulates bone growth. Progesterone maintains the lining of the uterus necessary for a successful pregnancy.

You are instructing a new mother on how to take care for her infant's umbilical cord. Which of the following is TRUE? (SELECT ALL that apply) a) position the diaper below the cord to prevent infection b) cleanse the entire length of the cord using a cotton-tipped swab every time the diaper is changed c) sponge bathe the infant until the cord comes off d) apply antibiotic ointments to prevent infection e) never pull on the cord, it will begin to fall off on its own

a) position the diaper below the cord to prevent infection b) cleanse the entire length of the cord using a cotton-tipped swab every time the diaper is changed c) sponge bathe the infant until the cord comes off e) never pull on the cord, it will begin to fall off on its own Position the diaper below the cord to prevent infection and allow to air dry. Cleanse the entire length of the cord using a cotton-tipped swab every time the diaper is changed, however be advised that some hospital are now refraining from this (always check with your doctor). sponge bathe the infant until the cord comes off and never pull on the cord, it will begin to fall off on its own. Antibiotic ointments are unnecessary unless the cord becomes infected.

Fall risk assessments should be completed at all of the following time points EXCEPT: a) recovery room b) admission c) after there is a change in the patient's status d) after a fall e) at time of transfer to a different wards or units

a) recovery room

All of the following routes are considered parental methods of administration EXCEPT: a) rectal b) intraosseous c) intradermal d) intravenous

a) rectal Enteral administration involves the esophagus, stomach, and small and large intestines (e.g., the gastrointestinal tract). Methods of administration include oral, sublingual, and rectal. Parental routes do not include intraosseous, intravenous, intradermal, and intramuscular, to name a few.

What would the nurse teach a patient who is taking anticholinergic therapy for parkinsonism? (Select all that apply.) a. Avoid alcohol, cigarettes, and caffeine. b. Relieve dry mouth with hard candy or ice chips. c. Use sunglasses to reduce photophobia. d. Urinate 2 hours after taking the drug. e. Receive routine eye examinations.

a. Avoid alcohol, cigarettes, and caffeine.

The student nurse is working on a presentation regarding OBRA. What was the result of this landmark legislation? a. Deinstitutionalization b. Approved surgical treatment for schizophrenia c. Prohibition of electroshock therapy d. Increased construction of state facilities for residential mental health care

a. Deinstitutionalization

A patient is taking rivastigmine (Exelon) to improve cognitive function. What should the nurse teach the patient/family member to do? (Select all that apply.) a. Rise slowly to avoid dizziness. b. Remove obstacles from pathways to avoid injury. c. Closely follow the drug dosing schedule. d. Have frequent checks for hypertension. e. Receive regular liver function tests.

a. Rise slowly to avoid dizziness. b. Remove obstacles from pathways to avoid injury. c. Closely follow the drug dosing schedule.

You are providing chest compressions. Another colleague is providing ventilations with a bag valve mask. When should you and your colleagues switch positions? a. after 2 minutes of CPR, or before if you are becoming fatigued b. after 3 minutes of CPR c. after 10 minutes of CPR d. never you should continue with the roles you are in

a. after 2 minutes of CPR, or before if you are becoming fatigued

Assessment of clients with suicidal ideation should include: a. asking them whether they have a specific plan b. administering an antidepressant medication c. referring the client to a psychiatrist d. suggesting the client join a support group

a. asking them whether they have a specific plan Asking whether the patient has a specific plan in an assessment question. The other answers are interventions.

A nurse falsifying the narcotics record within a patient's medication administration record is an example of: a. felony b. unintentional tort c. negligence d. misdemeanor

a. felony

Which of the following types of tracheostomy tubes allows the patient to speak when it is capped? a. fenestrated tube b. double lumen tube c. cuffed tube d. single lumen tube

a. fenestrated tube A tracheostomy is a direct opening into the trachea to maintain potency of the airway. A tube is then inserted into the opening; there are different types of tubes. The fenestrated tube is a type that has an opening along the posterior aspect of the outer cannula. When it is capped, the patient is able to breathe through their upper airway and speak as well. It is important to note that the cuff must always be deflated prior to capping a fenestrated tube.

Immediately after an amniotomy has been performed, the nurse should first assess which of the following? a. the fetal heart rate pattern b. cervical dilation c. bladder distension d. the maternal blood pressure

a. the fetal heart rate pattern The FHR is assessed immediately after amniotomy to detect any changes that may indicate cord compression or prolapse. Bladder distention or maternal blood pressure would not be the first thing to check after the amniotomy

What are the phases of shock in correct sequential order? a. the initial stage, the compensatory stage, the progressive stage, and the final or refractory stage b. the initial stage, the progressive stage, the compensatory stage, and the final or refractory stage c. the initial stage, the refractory stage, the compensatory stage, and the progressive stage d. the primary stage, the compensatory stage, the progressive stage, and the final or refractory stage

a. the initial stage, the compensatory stage, the progressive stage, and the final or refractory stage The initial stage is characterized by hypoxia and increases of lactic acid; the compensatory phase is marked with hyperventilation and other compensatory mechanisms; the progressive phase is accompanied by metabolic acidosis and the failure of compensatory mechanisms; and the final, or refractory, phase has a very poor prognosis; death is imminent

You are a home health care nurse who is conducting an environment safety survey of the client's home. Which safety risk must be immediately addressed and corrected? a. the unknown date of battery changes in the smoke alarms b. absence of food in the refrigerator c. not enough lighting in the house

a. the unknown date of battery changes in the smoke alarms Batteries in smoke alarms should be changed at least every 6 months. When the date of the last change is not known, the nurse should not assume that the batteries are still good. The nurse. therefore, must change the batteries and note the battery change date in order to protect the home client from fire hazards.

A client with hypertension is scheduled for a scan and electrolyte studies. During an interview with the nurse, the client exclaims, "I don't know why I just can't get a prescription for high blood pressure pills; that probably is all it is. I'm missing work by being here." Which is the best response by the nurse? a) "It might not be high blood pressure. We have to be sure." b) "It's frustrating to miss work and not know for sure what's wrong." c) "I know it's frustrating, but you need to have a diagnostic workup." d) "Maybe you could ask your health care provider if the tests could be done on separate days."

b) "It's frustrating to miss work and not know for sure what's wrong." The response "It's frustrating to miss work and not know for sure what's wrong" indicates that the nurse has heard the verbal message and has empathy for the client; it encourages further verbalization. The response "It might not be high blood pressure. We have to be sure." may increase the client's anxiety. The response "I know it's frustrating, but you need to have a diagnostic workup" minimizes the client's concerns. The response "Maybe you could ask your health care provider if the tests could be done on separate days" depersonalizes the client's concerns; it focuses on the tests and the scheduling rather than the client's needs.

Your diabetic patient is taking 750 mg of Metformin twice daily. Pharmacy send you tablets that are 250 mg in strength. How many tablets should you give your diabetic patient? a) 4 b) 3 c) 5 d) 3 1/2

b) 3

A client is found to have paranoid schizophrenia, and the practitioner prescribes a typical antipsychotic medication. After a 1-month hospitalization the client is discharged home with instructions to continue the antipsychotic and a referral for weekly mental health counseling. The picture illustrates the client's physical status as observed by the nurse on the client's first visit to the community mental health clinic. What extrapyramidal side effect has developed? a) Dystonia b) Akathisia c) Tardive dyskinesia d) Pseudoparkinsonism

b) Akathisia Akathisia, an extrapyramidal side effect of typical antipsychotics, is motor restlessness. The client is unable to sit or stand still and feels the need to move, pace, rock, swing the legs, or tap the feet. The condition occurs within 5 to 90 days of the initiation of therapy. Dystonia is muscle spasms of the face, tongue, head, neck, jaw, or back, usually resulting in exaggerated posturing. This extrapyramidal side effect of typical antipsychotics occurs within 1 hour to 1 week of the initiation of therapy. Tardive dyskinesia is facial, ocular, oral/buccal, lingual/masticatory, and systemic movements. This extrapyramidal side effect of typical antipsychotics may occur 6 months or more after the initiation of therapy. Pseudoparkinsonism has characteristics similar to those of Parkinson's disease (e.g., shuffling gait, tremors, rigidity, bradykinesia). This extrapyramidal side effect of typical antipsychotics may occur anytime after the initiation of therapy.

Where should the nurse expect the first heart sound (S 1) to be the loudest when auscultating a client's heart? a) Base of the heart b) Apex of the heart c) Left lateral border d) Right lateral border

b) Apex of the heart The first heart sound is produced by closure of the mitral and tricuspid valves; it is best heard at the apex of the heart. The base of the heart is where the second heart sound (S 2 ) is best heard; S 2 is produced by closure of the aortic and pulmonic valves. Left lateral border covers a large area; the auscultatory areas that lie near it are the pulmonic and mitral areas. Right lateral border covers a large area; the only auscultatory area near it is the aortic area.

A primary health care provider prescribes atenolol (Tenormin) 20 mg by mouth four times a day for a client who has had double coronary artery bypass surgery. What information is most important for the nurse to include in the discharge teaching plan for this client? a) Drink alcoholic beverages in moderation. b) Avoid abruptly discontinuing the medication. c) Increase the medication if chest pain develops. d) Report a pulse rate less than seventy beats per minute.

b) Avoid abruptly discontinuing the medication. Clients should never increase medications without a health care provider's direction. Alcohol is contraindicated for clients taking atenolol because it can cause additive hypotension. An abrupt discontinuation of atenolol may cause an acute myocardial infarction. The pulse rate can go much lower as long as the client feels well and is not dizzy.

A nurse provides dietary instruction to a client who has iron deficiency anemia. Which food choices by the client does the nurse consider most desirable? a) Raw carrots b) Boiled spinach c) Sweet potatoes d) Brussels sprouts e) Asparagus spears

b) Boiled spinach c) Sweet potatoes One cup of boiled spinach contains 6.42 mg of iron. One cup of mashed sweet potatoes contains 3.4 mg of iron. One cup of cut carrots contains 1 mg of iron. One cup of Brussels sprouts contains 1.1 mg of iron. One cup of cut asparagus contains 1.2 mg of iron.

For symptomatic relief of various symptoms of premenstrual syndrome (PMS), which drug is the best for the nurse to suggest to the patient? a) Vitamin B6 for cramping pain b) Buspirone (BuSpar) for anxiety c) Spironolactone (Aldactone) for negative mood d) Ibuprofen (Motrin, Advil) for premenstrual dysphoric disorder

b) Buspirone (BuSpar) for anxiety Buspirone (BuSpar) is an antianxiety drug taken in the luteal phase, which helps to alleviate symptoms associated with premenstrual syndrome. Vitamin B6 acts as a mood elevator. Spironolactone (Aldactone) is used in patients with fluid retention due to its diuretic action. Ibuprofen (Motrin, Advil) is used for painful conditions of premenstrual syndrome.

A teenage girl refuses the human papilloma virus (HPV) vaccine. Which complication of HPV infection should the nurse teach the teenager about? a) Asthma b) Cervical cancer c) Gastroenteritis d) Uterine fibroids

b) Cervical cancer The human papilloma virus (HPV) infection is common in sexually active females and may lead to cervical cancer. HPV vaccines are administered to lower the risk of cervical cancer in females. HPV infection does not cause asthma, gastroenteritis, or uterine fibroids. Asthma can be allergic and psychological in origin. Gastroenteritis is a multicausal condition, and can be cased due to infection by bacteria or virus. Uterine fibroids are abnormal growths in the uterus and are unrelated to HPV.

A nurse asks a client with ischemic heart disease to identify the foods that are most important to restrict. The nurse determines that the client understands the dietary instructions when the client identifies the following foods. a) Olive oil b) Chicken broth c) Enriched whole milk d) Red meats, such as beef e) Vegetables and whole grains f) Liver and other glandular organ meats

b) Chicken broth c) Enriched whole milk d) Red meats, such as beef f) Liver and other glandular organ meats Chicken broth is high in sodium and should be avoided to prevent fluid retention and an elevated blood pressure. Enriched whole milk is high in saturated fats and contributes to hyperlipidemia; skim milk is the healthier choice. Red meats, such as beef, are high in saturated fats and should be avoided. Liver and other glandular organ meats are high in cholesterol and should be avoided. Olive oil is an unsaturated fat, which is a healthy choice. Vegetables and whole grains are low in fat and have soluble fiber, which may reduce the risk for heart disease.

When monitoring a client for hyponatremia, what clinical findings should the nurse consider significant? a) Thirst b) Confusion c) Tachycardia d) Pale coloring e) Poor tissue turgor

b) Confusion e) Poor tissue turgor Confusion is associated with hyponatremia. Cellular swelling and cerebral edema are associated with hyponatremia; as extracellular sodium level decreases, the cellular fluid becomes relatively more concentrated and pulls water into cerebral cells. Poor tissue turgor is associated with hyponatremia; as extracellular sodium level decreases, cellular fluid becomes more concentrated, and in response more fluid is pulled into the cells. Thirst is a symptom of hypernatremia; it may indicate dehydration. Tachycardia is associated with hypovolemia, not hyponatremia. Pallor is not a sign of hyponatremia; it may indicate anemia.

A client has left ventricular heart failure. For which clinical indicators should the nurse assess the client? a) Ascites b) Crackles c) Peripheral edema d) Dyspnea on exertion e) Jugular vein distention

b) Crackles d) Dyspnea on exertion Pressure in the pulmonic circulation increases when the left ventricle fails; fluid moves from the intravascular compartment into the alveoli, causing crackles. Pressure in the pulmonic circulation increases when the left ventricle fails; fluid in the alveoli impairs gas exchange, which causes dyspnea. Ascites, a sign of right ventricular failure, results from an increased hydrostatic pressure in the systemic circulation; fluid moves out of the intravascular compartment into the abdominal cavity. Peripheral edema, a sign of right ventricular failure, results from an increased hydrostatic pressure in the systemic circulation. Fluid moves out of the intravascular compartment into the interstitial compartment. Jugular vein distention, a sign of right ventricular failure, results from hypervolemia.

A nurse is interpreting the electrocardiogram (ECG) of a patient on the telemetry floor. The patient's ECG has the following characteristics: rate of 85, P wave before every QRS complex, and occurrence of each complex at a regular interval. What should the nurse do first? a) Call the provider b) Document findings c) Obtain a full set of vital signs d) Prepare the patient for cardioversion

b) Document findings The patient's ECG shows a regular rate and rhythm. The nurse should document the findings. It is not necessary to call the provider, obtain vital signs, or prepare the patient for cardioversion.

A nurse is caring for a patient 12 hours after a lung tissue biopsy. The nurse notes streaks of blood in the patient's sputum. What should the nurse's action be at this time? a) Obtain vital signs b) Document the findings c) Prepare the patient for surgery d) Call the primary health care provider

b) Document the findings Blood streaks in the sputum are normal for several days after a lung tissue biopsy. The nurse should document the findings and be alert for increased blood in the sputum. It is not necessary to obtain vital signs, call the primary health care provider, or prepare the patient for surgery.

Which drugs used in the treatment of cancer may increase the risk of stomatitis in the patient? Select all that apply. a) Granisetron (Kytril) b) Doxorubicin (Rubex) c) 5-fluorouracil (5-FU) d) Methotrexate (MTX) e) Metoclopramide (Reglan)

b) Doxorubicin (Rubex) c) 5-fluorouracil (5-FU) d) Methotrexate (MTX) Stomatitis is a condition of inflamed, sore, and ulcerated areas within the patient's mouth due to destruction of normal cells. Doxorubicin (Rubex), 5-fluorouracil (5-FU), and methotrexate (MTX) are the chemotherapeutic drugs that increase the risk of stomatitis as a side effect. Granisetron (Kytril) and metoclopramide (Reglan) are antiemetic drugs and do not produce stomatitis.

A primary health care provider decides to omit a treatment that was part of a course of chemotherapy for a client because the client demonstrates myelosuppression. What information would be appropriate for the nurse to give to the client regarding myelosuppression? a) Calcium and vitamin D must be increased in the diet because of the effects of myelosuppression b) Eating a balanced diet, resting, and trying to avoid bleeding and infections are appropriate at this time c) The development of myelosuppression explains why the client has nausea, vomiting, anorexia, and alopecia d) Frequent testing for restlessness, muscle control, and pupillary response is necessary because the meninges may be irritable

b) Eating a balanced diet, resting, and trying to avoid bleeding and infections are appropriate at this time Myelosuppression involves a decreased number of red blood cells (anemia), resulting in a reduced oxygen-carrying capacity of the blood and fatigue. A decreased number of white blood cells (leukopenia) result in a potential for infection. A decreased number of platelets ( thrombocytopenia) result in a potential for bleeding. Myelosuppression is not related directly to calcium and vitamin D; myelosuppression, a reduction in bone marrow activity, results in decreased numbers of red blood cells (RBCs), white blood cells (WBCs), and platelets. Myelosuppression is not related to nausea, vomiting, anorexia, or alopecia. Myelosuppression is related to bone marrow activity, not the nervous system.

The nurse is caring for a 70-year-old patient with pneumonia. What would be the best nursing intervention to promote expectoration in the patient? a) Provide 700 calories intake per day. b) Encourage 3L of fluid intake per day. c) Avoid vigorous coughing and breathing. d) Avoid upright positioning of the patient.

b) Encourage 3L of fluid intake per day. As the elderly patient faces difficulty in expectoration, the nurse should adequately hydrate the patient to promote expectoration. A minimum of 3 L fluid intake orally or intravenously is administered to a patient with pneumonia. An elderly patient is provided a minimum of 1500 calories per day to maintain strength in the body. Coughing and deep breathing are encouraged to promote expectoration in patients with pneumonia. Upright and side lying positions are comfortable and promote the patient's breathing pattern in conditions like pneumonia.

What medication is often used for cancer-related pain? a) adalimumab b) gabapentin c) anastrozole d) metoprolol

b) Gabapentin Gabapentin, originally used for epilepsy, is now being used for neuropathic cancer pain, chronic lower back pain, and restless leg syndrome in addition to neurological epilepsy. This medication is taken orally every 12 hours. Adalimumab is used for plaque psoriasis; anastrozole is used to prevent a recurrence of breast cancer among women with early breast cancer; and metoprolol is a beta adrenergic blocker that is used for the treatment of hypertension.

A nurse is caring for a client with a diagnosis of acute salpingitis. Which condition most commonly causing inflammation of the fallopian tubes should the nurse include when planning a teaching program for this client? a) Syphilis b) Gonorrhea c) Hydatidiform mole d) Spontaneous abortion

b) Gonorrhea Gonorrhea frequently is an ascending infection that affects the fallopian tubes. Syphilis, if untreated, may spread to the nervous system via the blood; it usually does not cause ascending infection of the fallopian tubes. Hydatidiform mole is an aberrant growth; it will not cause inflammation of the fallopian tubes. A spontaneous abortion should not cause inflammation of the fallopian tubes.

What is the most important information the nurse and the rapid response team must keep in mind when caring for a client who had a cardiac arrest? a) age of the client b) How long the client was anoxic c) Heart rate of the client before the arrest d) Emergency medications available for the client

b) How long the client was anoxic Irreversible brain damage will occur if a client is anoxic for more than four minutes. The age of the client does not affect the response by the arrest team. The earlier heart rate is of minimal importance; the rhythm is more significant. Although a variety of emergency medications must be available, their administration is prescribed by the health care provider.

What is the most severe risk for a patient with agranulocytosis? a) Pain b) Infection c) Malnutrition d) Spread of germs to other patients

b) Infection The most severe risk for a patient with agranulocytosis is infection. Agranulocytosis is a potentially fatal condition of the blood characterized by a severe reduction in the number of granulocytes. Both the white blood cell (WBC) count and the differential neutrophil count are extremely low. The patient with this disorder is highly susceptible to a life-threatening infection. Although pain may be a symptom of the disease, it is not the most severe risk for the patient. Although malnutrition may result from the general malaise that accompanies the disease, it is not the most severe risk for the patient. The risk for the patient with agranulocytosis is acquiring—not spreading—an infection.

A nurse is caring for a client with a diagnosis of renal calculi secondary to hyperparathyroidism. Which type of diet should the nurse explore with the client when providing discharge information? a) Low purine b) Low calcium c) High phosphorus d) High alkaline ash

b) Low calcium Calcium and phosphorus are components of these stones; foods high in calcium and phosphorus should be avoided. Low purine and high alkaline ash diets are indicated for clients with gout. Foods high in phosphorus must be avoided.

A nurse is caring for a patient in the end stages of human immunodeficiency virus/acquired immunodeficiency syndrome (HIV/AIDS). The nurse knows care planning is based on which concept? a) Allowing family members to say goodbyes b) Managing symptoms and improving quality of life c) Helping the patient regain health and independence d) Preventing complications and increasing the length of the patient's life

b) Managing symptoms and improving quality of life In the end stages of HIV/AIDS, care planning is centered on managing the patient's symptoms and improving his or her quality of life. Finding a cure, regaining health and independence, and increasing the length of the patient's life are not the primary goals of care.

What client factor does the nurse consider to have the greatest impact on the effectiveness of bariatric surgery? a) Freedom from concurrent high-risk conditions. b) Motivation to cooperate with required lifestyle changes. c) Willingness to have a panniculectomy a year after weight is stabilized. d) Ability to tolerate the large abdominal incision necessary for this surgery.

b) Motivation to cooperate with required lifestyle changes. Bariatric surgery requires the client to engage in significant postoperative lifestyle changes (e.g., radically modifying eating habits, taking nutritional supplements, meeting numerous emotional challenges, engaging in exercise); clients who cannot cooperate with the postoperative program are not considered candidates for bariatric surgery. Clients with concurrent high-risk conditions may be considered candidates for this surgery because the benefits outweigh the risks. Panniculectomy, surgery performed to remove excess abdominal skin folds, is elective surgery that not all clients choose to receive. Many bariatric surgeries can be done laparoscopically and do not require a large abdominal incision; although bariatric surgery can be done laparoscopically, it is still major surgery for obese clients.

A client has been experiencing extreme fatigue lately. The nurse suspects anemia and examines the client to identify additional clinical manifestations to support this inference. What locations on the client's body should the nurse assess? a) Sclera b) Nail beds c) Lining of eyelids d) Palms of hands e) Bony prominences

b) Nail beds c) Lining of eyelids d) Palms of hands Nail beds lose their pink coloration because of reduced hemoglobin. A reduced amount of hemoglobin decreases pink color of the lining of the eyelids. Palms of the hands will become pale because of the decreased hemoglobin. Sclerae are observed for signs of jaundice, not anemia, when they become pale yellow to orange. Bony prominences are not assessed when a client has anemia. Bony prominences are examined for redness caused by pressure that, if prolonged, can lead to a break in the skin and development of pressure ulcers.

An older adult is brought to the emergency department after being found in the street without a coat during a snowstorm. What actions should the nurse implement? a) Massage extremities. b) Obtain a rectal temperature. c) Assess the fingers for areas of frostbite. d) Determine client's level of consciousness. e) Ask for client identification.

b) Obtain a rectal temperature. c) Assess the fingers for areas of frostbite. d) Determine client's level of consciousness. e) Ask for client identification. A rectal temperature provides the most accurate temperature. Older adults have less subcutaneous fat and inefficient temperature regulating mechanisms, which makes them vulnerable to extremes in environmental temperature. The extremities are more distal sites of circulation and are at increased risk for frostbite. Hypothermia decreases cerebral perfusion, which will result in confusion and a decreased level of consciousness. Getting client identification will help in learning more about the client's previous health history and contacting family members. Massage is contraindicated because it may injure tissues that have sustained frostbite.

Which responses should a nurse expect a client experiencing hypoglycemia to exhibit? a) Nausea b) Palpitations c) Tachycardia d) Nervousness e) Warm, dry skin f) Increased respirations

b) Palpitations c) Tachycardia d) Nervousness Palpitations are of neurogenic origin associated with hypoglycemia; the sympathetic nervous system is stimulated by the decline in blood glucose. Tachycardia occurs with low serum glucose levels because of sympathetic nervous system activity. Nervousness, anxiety, and shakiness occur as a result of sympathetic nervous system stimulation associated with hypoglycemia. Nausea, vomiting, and abdominal cramps are associated with hyperglycemia. The client will feel hungry with hypoglycemia. Warm, dry skin is a sign of hyperglycemia, caused by dehydration associated with osmotic diuresis related to glycosuria. The skin will be cool and moist with hypoglycemia. Increased respirations are signs of ketoacidosis from insufficient insulin to prevent fat breakdown for energy; they are compensatory responses that occur in an attempt to blow off carbon dioxide and raise the serum pH. There is no particular change in respirations with hypoglycemia.

Which instruction should the nurse give the parents of an infant to prevent the risk of sudden infant death syndrome (SIDS)? a) Allow the infant to sleep on beds and sofas. b) Place the infant on the back while sleeping. c) Use soft pillows under the infant while sleeping. d) Place soft towels inside the infant's crib while sleeping.

b) Place the infant on the back while sleeping. The infant should be placed on the back to prevent the risk of SIDS. A prone position causes oropharyngeal obstruction, increasing the risk of SIDS.The infant should not be allowed to sleep on beds and sofas due to potential risk for accidental entrapment and suffocation. Soft pillows under the infant and towels inside the infant's crib can cause asphyxia while the infant is asleep and should be avoided.

Along with assessing for any mental health disorders, what else should the pediatric nurse assess for when trying to reduce pediatric deaths by homicide? a) Arguments with siblings b) Potential violent behavior c) Parents' history of larcenies d) Number of hours a day spent watching television

b) Potential violent behaviors When attempting to reduce pediatric deaths by homicide, the pediatric nurse must assess for potential violent behavior along with mental health disorders. Arguments with siblings are normal for children. Knowing a parents' history of larcenies is not related, nor is the number of hours a child spends watching television.

Which is the main diagnostic feature of Hodgkin disease? a) Bence Jones protein b) Reed-Sternberg cells c) Reticulum cell sarcoma d) Philadelphia chromosome

b) Reed-Sternberg cells Reed-Sternberg cells are atypical histiocytes consisting of large, abnormal, multinucleated cells in the lymphatic system found in Hodgkin disease. Reticulum cell sarcoma is a common type of lymphoma but is not the diagnostic feature of Hodgkin disease. Bence Jones protein is found in the urine of a patient with multiple myeloma. Philadelphia chromosome is found in a patient with chronic myelogenous leukemia.

A client who has acromegaly and insulin-dependent diabetes undergoes a hypophysectomy. The nurse identifies that further teaching about the hypophysectomy is necessary when the client states, "I know I will: a) Be sterile for the rest of my life." b) Require larger doses of insulin than I did preoperatively." c) Have to take cortisone or a similar drug for the rest of my life." d) Have to take thyroxine or a similar medication for the rest of my life."

b) Require larger doses of insulin than I did preoperatively." The hypophysis (pituitary gland) does not directly regulate insulin release. This is controlled by serum glucose levels. Because somatotropin release will stop after the hypophysectomy , any elevation of blood glucose level caused by somatotropin also will stop. Infertility may be expected after a hypophysectomy because the follicle-stimulating hormone and its releasing factor will no longer be present to stimulate spermatogenesis. When adrenocorticotropic hormone (ACTH) is absent, cortisone will have to be administered. Thyroid-stimulating hormone will not be present; extrinsic thyroxine will have to be taken.

A 37-year-old client with a nontender palpable breast mass has a questionable mammogram. She is undergoing further diagnostic tests to determine whether the mass is malignant. What information should the nurse take into consideration before planning health teaching for this client? a) Squamous cell carcinomas are neoplasms arising from glandular tissues. b) Results of a biopsy are necessary before a specific form of therapy is selected. c) Mammographies should be repeated to confirm the presence of malignancies. d) Waiting for several weeks before receiving confirmation of cancer is helpful to the client.

b) Results of a biopsy are necessary before a specific form of therapy is selected. The therapy selected depends on whether there is a malignancy and, if so, the type of cancer cells, the extent of nodal involvement, and the presence and extent of metastasis. Adenocarcinomas, not squamous cell carcinomas, arise from glandular tissue; squamous cell carcinomas arise from epithelial tissue. Only a biopsy will confirm the diagnosis of a malignancy. Waiting several weeks for a diagnosis is not advisable; an extended waiting period increases the client's stress and anxiety.

A sexually active client presents with a sore throat and a generalized rash. The client states that a chancre that had been present healed approximately three months ago. The physical assessment and the serologic test findings indicate a diagnosis of syphilis. The nurse recognizes that the client is experiencing what stage of syphilis? a) Primary b) Secondary c) Latent d) Tertiary

b) Secondary The client has secondary syphilis, which occurs one to three months after healing of the primary lesion and lasts for several weeks to as long as a year; it is the stage at which the individual is most infectious. Primary syphilis is the stage of initial infection and is characterized by the presence of a chancre, a painless lesion at the site of infection. Latent syphilis occurs after the secondary stage and before the late stage of syphilis; in latent syphilis the immune system is able to suppress the infection and there are no clinical signs and symptoms. Tertiary syphilis, also known as late syphilis, is the final stage of syphilis; 20% to 40% do not demonstrate signs and symptoms during this stage. At this stage it is a slowly progressive inflammatory disease that can involve many organs; common complications include paresis, brain attack, dementia, psychosis, aortitis, and meningitis.

A nurse is preparing to care for a patient with hemianopia. The nurse knows this results in which condition? a) Seeing two of each object in the visual field b) Seeing objects in only half of the visual field c) Seeing only objects that are close to the eyes d) Seeing only objects that are far away from the eyes

b) Seeing objects in only half of the visual field Hemianopia is a condition in which the patient is unable to see half of the visual field. Diplopia is seeing two of each object. Seeing only objects that are close to the eyes is myopia. Presbyopia is seeing only objects that are far from the eyes.

When teaching a client about using a diaphragm as a form of contraception, the nurse should tell her that the diaphragm: a) May or may not be used with a spermicidal lubricant b) Should remain in place for at least 6 hours after intercourse c) Must be inserted with the dome facing down to be maximally effective d) Often appears puckered but that this will not interfere with its effectiveness

b) Should remain in place for at least 6 hours after intercourse The should remain in place for at least 6 hours after intercourse because the spermicidal jelly or cream requires this amount of time to be effective. The diaphragm must always be used with a spermicide to be effective. The diaphragm may be inserted with the dome facing either up or down and still be effective. Puckering, especially near the rim, may indicate thin spots that could rupture during intercourse; the diaphragm should be replaced if puckering is found.

An anxious client reports experiencing pain in the abdomen and feeling empty and hollow. A diagnostic workup reveals no physical causes of these clinical findings. What term best reflects what the client is experiencing? a) Dissociation b) Somatization c) Stress response d) Anxiety reaction

b) Somatization Somatization is erroneously attributing an anxious feeling to a body system or part. Dissociation is separating an overwhelming event from one's consciousness. The stress response results from being exposed to a threatening stimulus. An anxiety reaction is the body's reaction to a stressful event.

The nurse is caring for a patient with two enlarged lymph nodes on the same side of the diaphragm. Which stage of the Hodgkin's disease does the patient's medical condition indicate? a) Stage I b) Stage II c) Stage III d) Stage IV

b) Stage II The appropriate stage of Hodgkin's lymphoma must be established before selecting a proper treatment plan. Two or more abnormal lymph nodes on the same side of the diaphragm are observed in Stage II. Abnormal single lymph nodes are seen in Stage I. Abnormal lymph node regions on both sides of the diaphragm are observed in Stage III. Stage IV is characterized by diffuse and disseminated involvement of one or more extralymphatic tissues and/or organs—with or without lymph node involvement.

A client with varicose veins asks a nurse what is involved when ligation and stripping are performed rather than sclerotherapy. What should the nurse consider when planning a response in language the client will understand? a) Plaque from within the veins is abraded. b) The dilated saphenous veins are removed. c) Superficial veins are anastomosed to deep veins. d) An umbrella filter is placed in the large affected veins.

b) The dilated saphenous veins are removed. During a ligation, the saphenous vein is removed. Plaque is an arterial, rather than a venous, problem. Anastomosing superficial veins to deep veins is not done during this surgery; superficial and deep veins usually are attached by communicating veins. An umbrella filter placed in the large affected veins prevents emboli from traveling to the lung; it is not a vein ligation and stripping.

A student nurse is preparing for a sterile procedure. What action if taken indicates the student nurse has mastered the process? a) A perforated glove is used. b) The dominant hand is gloved first. c) The nondominant hand is gloved first. d) Clean gloves are worn under the sterile gloves.

b) The dominant hand is gloved first. The dominant hand is gloved first because it improves dexterity. Perforation in a glove allows pathogens to enter through the gloves. Wearing clean gloves under sterile gloves is inappropriate because once the sterile gloves touch the clean gloves, they are considered contaminated.

A client with burns develops a wound infection. The nurse plans to teach the client that local wound infections primarily are treated with what type of antibiotics? a) Oral b) Topical c) Intravenous d) Intramuscular

b) Topical Topical antibiotics are applied directly to the wound and are effective against many gram-positive and gram-negative organisms found on the skin. Although oral, intravenous, and intramuscular antibiotics may be administered, they are most effective for systemic rather than local infections; the vasculature in and around a burn is impaired and the medication may not reach the organisms in the wound.

Which drug does the primary health care provider prescribe for a patient with human immunodeficiency virus (HIV)? a) Dronabinol (Marinol) b) Zidovudine (Retrovir) c) Pravastatin (Pravachol) d) Rosiglitazone (Avandia)

b) Zidovudine (Retrovir) Zidovudine (Retrovir) is used to treat HIV. Dronabinol (Marinol) is given to stimulate appetite in patients who have anorexia. Pravastatin (Pravachol) is an anticholesterol drug that is given to treat hyperlipidemia. Rosiglitazone (Avandia) is an oral hypoglycemic agent given to treat diabetes.

Which of the following scenarios is consistent with orthostatic hypotension? a) supine BP: 135/75, standing BP: 150/80 b) supine BP: 125/80, standing BP: 90/50 c) supine BP: 125/80, standing BP: 120/70 d) supine BP: 125/80, standing BP: 120/85

b) supine BP: 125/80, standing BP: 90/50 Orthostatic hypotension occurs when a patient's blood pressure decreases by 15 to 20 mm Hg or more when moving from a lying position to a standing position. In the answer, the patient's systolic blood pressure dropped by 35 mm Hg upon standing. Orthostatic hypotension can cause dizziness and syncope. It can be caused by dehydration, heart problems, or nervous system disorders.

A nurse is monitoring a postpartum patient in the fourth stage of labor. Which of the following findings, if noted by the nurse, would indicate a complication related to a laceration of the birth canal? a) palpation of the fundus at the level of the umbilicus b) the saturation of more than one peripad per hour c) presence of dark red lochia d) palpation of the uterus as a firm contracted ball

b) the saturation of more than one peripad per hour In the first 24 hours after birth, the uterus will feel like a firmly contracted ball, roughly the size of a large grapefruit. One can easily locate the uterus at the level of the umbilicus. Saturation of more than one peripad per hour is considered excessive in the early postpartum period.

5.Which is a nursing intervention for a patient taking carbidopa-levodopa for parkinsonism? a. Encourage the patient to adhere to a high-protein diet. b. Inform the patient that perspiration may be dark and stain clothing. c. Advise the patient that glucose levels should be checked with urine testing. d. Warn the patient that it may take 4 to 5 days before symptoms are controlled.

b. Inform the patient that perspiration may be dark and stain clothing.

Which client is at greatest risk for rejection of a bone marrow transplant? a. The one who received a syngeneic bone marrow transplant b. The one who received an allogeneic bone marrow transplant c. The one who received a heterogenous bone marrow transplant d. The one who received an autologous bone marrow transplant

b. The one who received an allogeneic bone marrow transplant There are 3 types of bone marrow transplants. Transplants can be allogeneic, syngeneic, or autologous. Allogeneic bone marrow transplants may or may not be an HLA match, and the donor can be related or unrelated to the recipient. This type of transplant is associated with the greatest risk of rejection. Syngeneic transplants come from an identical twin, and autologous transplants are from the patient himself. They are therefore less risky than allogeneic transplants.

All of the following are teaching points for the patient with a colostomy except: a. change the colostomy bag while in front of a mirror b. change your pouch daily c. wash the stoma and skin around the stoma d. apply skin barrier prep or wipes before applying a new pouch

b. change your pouch daily pouches should be changed every 5 to 7 days or as needed in between

You are caring for a child with epiglottitis. What equipment is most important to have at the bedside? a. chest tube and drainage system b. cricothyrotomy kit c. oxygen saturation minitor d. blood pressure monitor

b. cricothyrotomy kit airway management is most important. Patients may deteriorate quickly, and airway equipment, including equipment needed for cricothyrotomy, should be present at the patient's bedside. You would not need to insert a chest tube in a patient with epiglottitis.

You are caring for a child who has a temperature of 104 degrees. The child suddenly begins to rhythmically convulse. What is the first thing you should do? a. gently restrain the child's movement b. ensure the child's safety c. call the doctor d. administer an antipyretic medication as ordered

b. ensure the child's safety Seizures place clients, both children and adults, at risk of injuries, some of which can be life threatening. The first priority is safety. Later, when the child is out of danger, you should call the doctor and administer antipyretic medications as ordered. Client movement should be protected but not restrained during seizures or convulsions.

A nurse is assessing a newborn infant to a mother who is addicted to drugs. Which of the following assessment findings would the nurse expect to note during the assessment of this infant? a. sleepiness b. incessant crying c. lethargy d. cuddling when held

b. incessant crying A new born infant who is born to a drug addicted mother is irritable. The infant is easily overloaded by sensory stimulation. The infant may be difficult to console. The infant would hyperextend and posture rather than cuddle when held.

Your patient's abdominal wound dehisces, and bowel can be seen protruding through the opened incision. Nursing interventions include: a. assessment of the patient's vital signs every hour b. positioning the patient in bed with the knees bent and the head of the bed no higher than 20 degrees to prevent an increase in intraabdominal pressure c. covering the wound with gauze soaked in betadine d. gently reducing the exposed viscera

b. positioning the patient in bed with the knees bent and the head of the bed no higher than 20 degrees to prevent an increase in intraabdominal pressure Positioning the patient appropriately will decrease abdominal pressure. Cover the wound with gauze or towels soaked in sterile saline. Do not attempt to replace any exposed organs. Assess vital signs every 15 minutes to watch for signs of shock.

Your male patient has had a TURP (transurethral resection of the prostate). He has a 3 way catheter, and CBI (continuous bladder irrigation) is ongoing. He suddenly complains of lower abdominal discomfort. He is diaphoretic and tachycardia. You notice that his catheter is bypassing. What should you do first? a. increase the flow of the irrigation b. stop the irrigation immediately c. call the physician d. attempt to manually irrigate the catheter

b. stop the irrigation immediately You should turn the bladder irrigation immediately to prevent further distention of the bladder. It is likely that the catheter had become blocked, resulting in the patient's symptoms. You may manually irrigate the catheter if it is within your scope of practice. The physician should be notified, but the most important first step is to stop the irrigation

A patient reports of frequent bleeding from a nostril. Which advice should the nurse provide to the patient to prevent recurrent hemorrhage? a) "Use a vaporizer frequently." b) "Sneeze with the mouth closed." c) "Avoid blowing the nose vigorously." d) "Expectorate any blood rather than swallowing it."

c) "Avoid blowing the nose vigorously." Epistaxis is a condition of frequent bleeding from one or both the nostrils. The patient should be instructed to avoid blowing the nose vigorously in order to prevent recurrent hemorrhage. Using a vaporizer or nasal lubricant helps to keep the nasal membranes moist. Patients with epistaxis should be taught to sneeze with the mouth open to prevent the risk of aspiration. Blood and blood clots should be expectorated out rather than swallowing. However, that does not prevent recurrent hemorrhage in the patient.

Four teenagers have been admitted to the floor for unsuccessful suicide attempts. Which patient statement is most concerning to the nurse? a) "I will attend counseling twice a week after I am discharged." b) "I plan to attend all of my classes at school when I return home." c) "I am so excited that my dad is buying me a gun for my birthday next week." d) "I will talk openly with my parents and friends if I begin to feel overwhelmed."

c) "I am so excited that my dad is buying me a gun for my birthday next week." This statement should alarm the nurse because this patient could be planning the next suicide attempt. The fact that the teen will possess a firearm increases the chance of success the next time. The nurse should be encouraged if the patient states that the patient plans to attend counseling sessions and classes at school after discharge. The nurse should also be encouraged by the statement that the patient will talk to parents and friends if beginning to feel overwhelmed. This is an excellent coping mechanism.

The nurse is teaching parents about the care to be taken with respect to diaper rashes. Which statement made by the parents indicates the need for further teaching? a) "I should change diapers as soon as they are soiled." b) "I should apply zinc oxide ointment on reddened skin." c) "I should use perfumed soaps and wipes to clean the skin." d) "I should keep the skin dry and use superabsorbent diapers."

c) "I should use perfumed soaps and wipes to clean the skin." Perfumed soaps and wipes should be avoided because they may cause irritation to the skin. Mild soaps should be used to clean the skin. Diapers should be changed as soon as they are soiled to prevent skin irritation. Zinc oxide is used to protect erythematous skin from infection. Keeping the skin dry is a primary intervention to promote healing.

A child newly diagnosed with juvenile idiopathic arthritis (JIA) has come to the clinic for a checkup. While there, the parents ask about activities at school. What is the nurse's most appropriate response? a) "Provide pain medications only during school activities." b) "Do not allow your child to participate in school activities." c) "Talk with the school nurse to arrange for medications and rest periods." d) "Walk your child to all his classes to make sure other kids do not pick on him."

c) "Talk with the school nurse to arrange for medications and rest periods." JIA is a chronic disorder that affects the activities of daily living (ADLs) of a child. Allowing for as much independence as possible is necessary, with the need for pain management and rest periods. JIA is a chronic disorder that affects a child's activities of daily living (ADLs), not only school activities. Test-Taking Tip: Once you have decided on an answer, look at the stem again. Does your choice answer the question that was asked? If the question stem asks "why," be sure the response you have chosen is a reason. If the question stem is singular, then be sure the option is singular, and the same for plural stems and plural responses. Many times, checking to make sure that the choice makes sense in relation to the stem will reveal the correct answer.

Which statement or question is best when asking a toddler about breakfast? a) "Do you want to have breakfast?" b) "Why don't you have a cheese sandwich?" c) "You can have an egg and toast or cereal with milk." d) "Would you like to have toast with jam and cheese?"

c) "You can have an egg and toast or cereal with milk." Toddlers exhibit negativism and tend to say "no" to everything. So it is better to give them options rather than asking them questions that can be answered with a "no." Therefore, the toddler should be given a choice between an egg and toast or cereal with milk. The question "Do you want to have breakfast?" can be easily answered with a "no." The statement "Why don't you have a cheese sandwich?" doesn't offer any option, and the child may refuse. The question "Would you like to have toast with jam and cheese?" could also be answered with a "no." Test-Taking Tip: Sometimes the reading of a question in the middle or toward the end of an exam may trigger your mind with the answer or provide an important clue to an earlier question.

A mother tells the clinic nurse that her 6-year-old child has been wetting the bed for the past 3 weeks. Previously there had been no problems. How should the nurse respond? a) "Try to eliminate fluids after dinner." b) "Children sometimes wet the bed when they're angry." c) "You did the right thing to bring your child in to be examined." d) "Wake your child every few hours at night to go to the bathroom."

c) "You did the right thing to bring your child in to be examined." This child, who is older than 5 years, had control of the bladder until 3 weeks ago; the first step in evaluating enuresis is to rule out any physical problems that may be causing it. Suggesting interventions for the problem is inappropriate until the cause of the problem is identified. Stating that children wet the bed when they are angry is inappropriate because it implies that this is a voluntary act. Waking the child every few hours will interfere with the child's rest; advice is inappropriate until the cause of the problem is identified.

A client experiences elevated triglycerides and cholesterol. The client appears discouraged and says, "Well, I guess I'd better cut out all the fat and cholesterol in my diet." Which is the nurse's most appropriate response? a) Well, yes, that will certainly lower the amount of your blood fats." b) "That's good, but be sure to compensate by adding more carbohydrates." c) "You need some fat to supply the necessary fatty acids, so it's mainly just a need to cut down on the amount of fat you consume." d) "You need some cholesterol in your diet because your body cannot manufacture it, so just avoid excessive amounts."

c) "You need some fat to supply the necessary fatty acids, so it's mainly just a need to cut down on the amount of fat you consume." The essential fatty acids, linoleic acid and linolenic acid, are necessary for muscle tissue integrity, especially of the myocardium. All fats cannot and should not be eliminated from the diet. Carbohydrates do not contain the essential fatty acids, linoleic acid and linolenic acid. The body does manufacture cholesterol. Test-Taking Tip: Key words or phrases in the stem of the question such as first, primary, early, or best are important. Similarly, words such as only, always, never, and all in the alternatives are frequently evidence of a wrong response. As in life, no real absolutes exist in nursing; however, every rule has its exceptions, SO answer with care.

A client who just returned from a cardiac catheterization reports to the nurse that the pressure bandage on the right groin is tight. What action should the nurse take? a) Loosen the dressing slightly. b) Notify the health care provider. c) Assess the pulses distal to the dressing. d) Have the client flex the joints of the right leg.

c) Assess the pulses distal to the dressing. Assessing the circulatory status of the extremity will determine whether the dressing is too tight. Loosening the dressing slightly may result in bleeding from the catheter insertion site and is contraindicated. Notifying the health care provider is premature; the health care provider should be notified if circulation to the leg is compromised. Having the client flex the joints of the right leg may result in bleeding from the catheter insertion site and is contraindicated. The leg should remain extended for several hours.

A depressed client has feelings of failure and a low self-esteem. In what activity should the client initially be encouraged to become involved? a) Joining other clients in playing a board game b) Singing in a karaoke contest to be held at the end of the week c) Assisting a staff member in working on the monthly bulletin board d) Selecting the movie to be played during the evening recreation period

c) Assisting a staff member in working on the monthly bulletin board Working on the bulletin board with staff members involves minimal energy and decision-making and is the least threatening activity. Playing a board game is too stressful at this time; it will be a better intervention when self-esteem improves and depression lessens. Singing karaoke is too stressful an activity because it requires energy and good self-esteem, which the client does not have at this time. Selecting a movie is too stressful at this time; it will be a better intervention when self-esteem improves and depression lessens. Test-Taking Tip: Do not select answers that contain exceptions to the general rule, controversial material, or degrading responses.

A nurse is presenting information on sexual health to teens. The nurse teaches that which three-dose vaccines are given to prevent infection from human papillomavirus? a) HPV and varicella b) MMR and Vero cell c) Cervarix and Gardasil d) Vero cell and varicella

c) Cervarix and Gardasil Cervarix and Gardasil are three-dose vaccines used to prevent infection with human papillomavirus (HPV), the main cause of genital warts and cervical cancer. MMR is a vaccine for measles, mumps, and rubella. Vero cell vaccine is a rabies vaccine. HPV vaccine is another name for Gardasil, and varicella vaccine is a chickenpox vaccine.

The nurse is caring for a patient with epistaxis and finds that the patient has risk for aspiration. What immediate intervention should the nurse provide? a) Provide ventilator support by placing the patient in supine position. b) Place a mild hot pack on the nostrils of the patient to prevent aspiration. c) Close the patient's nostrils and ask the patient to breathe from the mouth. d) Instruct the patient to continuously swallow the expectorant to clear the airways.

c) Close the patient's nostrils and ask the patient to breathe from the mouth. Epistaxis is nose bleeding caused due to congestion of the nasal membrane. The patient may aspirate the blood while bleeding from the nose. Therefore, the nurse should close the nostrils of the patient and ask the patient to breath from the mouth to prevent aspiration. Ventilator support is provided when the patient has respiratory depression. Patients are placed in the recliner position when there is an episode of epistaxis to let the blood drain out. Placing the patient in supine position may cause aspiration of the blood. Cold packs are used to compress the nose of the epistaxis patient, unlike hot packs which may cause vasodilation and increase the bleeding. The nurse instructs the patient to expectorate the blood or clots to prevent nausea.

A client is diagnosed with gastroenteritis. What does the nurse determine is the basic intention underlying the unique dietary management for this client? a) Provide optimal amounts of all important nutrients. b) Increase the amount of bulk and roughage in the diet. c) Eliminate chemical, mechanical, and thermal irritation. d) Promote psychological support by offering a wide variety of foods.

c) Eliminate chemical, mechanical, and thermal irritation. Irritation of the mucosa may cause increased bleeding or perforation and therefore should be avoided. All clients' diets should be nutritionally balanced; this is not specific to this client's problem. Bulk and roughage may irritate the mucosa and should be decreased. Psychological support is not the primary goal; efforts should be made to include foods that are psychologically beneficial but eliminate foods that are irritating to the mucosa.

An older client is transferred to a nursing home from a hospital with a diagnosis of dementia. One morning, after being in the nursing home for several days, the client is going to join a group of residents in recreational therapy. The nurse sees that the client has laid out several outfits on the bed but is still wearing nightclothes. What should the nurse do? a) Help the client dress and explain when residents are expected at the activity b) Prompt the client to dress more quickly to avoid delaying the other residents c) Help the client select appropriate attire and offer to help the client get dressed d) Allow the client time to dress but explain that client has missed the opportunity to attend the activity

c) Help the client select appropriate attire and offer to help the client get dressed Helping the client select appropriate attire and offering help in getting dressed aid the client in decision-making; new situations may be stressful and may lead to ambivalent feelings. Helping the client dress and explaining when residents are expected at the activity are not sharing decision-making; the client may not remember this explanation in the future. Reminding the client to dress more quickly to avoid delaying the other residents may make the client feel guilty and may increase anxiety. The client may perceive being told that the opportunity to attend the activity has been missed as punishment. Test-Taking Tip: The most reliable way to ensure that you select the correct response to a multiple-choice question is to recall it. Depend on your learning and memory to furnish the answer to the question. To do this, read the stem, and then stop! Do not look at the response options yet. Try to recall what you know and, based on this, what you would give as the answer. After you have taken a few seconds to do this, then look at all of the choices and select the one that most nearly matches the answer you recalled. It is important that you consider all the choices and not just choose the first option that seems to fit the answer you recall. Remember the distractors. The second choice may look okay, but the fourth choice may be worded in a way that makes it a slightly better choice. If you do not weigh all the choices, you are not maximizing your chances of correctly answering each question.

An infant is being admitted with bacterial meningitis. The nurse knows the priority nursing action is: a) Assessing the infant's neurological status b) Beginning intravenous fluids and antibiotics c) Implementing respiratory isolation precautions d) Teaching the parents the importance of maintaining a quiet environment

c) Implementing respiratory isolation precautions The infant's illness is contagious, and the nurse, as well as other clients, must first be protected with the implementation of respiratory isolation precautions. Assessment of neurological status, implementation of prescribed fluids and antibiotics, and parental teaching may be done after assessment. Also, antibiotics are usually not administered until after all cultures have been obtained.

A 6-week-old infant and his mother arrive in the emergency department in an ambulance. The father arrives several minutes later with two children, 7 and 9 years old. The infant is not breathing, and the eventual diagnosis is sudden infant death syndrome (SIDS). The parents take turns holding the infant in another room. The nurse remains present and provides emotional support to the parents. What is an important short-term goal for this family? a) Identifying the problems that they will be facing as a result of the loss of the infant b) Accepting that there was nothing that they could have done to prevent the infant's death. c) Including the infant's siblings in the events and grieving in the wake of the infant's death d) Seeking out other families who have lost infants to SIDS and obtaining support from them.

c) Including the infant's siblings in the events and grieving in the wake of the infant's death The other children need to be involved with the grieving process and to work through their own feelings. Identifying the problems that the family will be facing in regard to the loss of the infant is a long-term goal. It is too early to seek out other families who have lost infants to SIDS and receive support from them. It is premature to accept that there was nothing that the family could have done to prevent the infant's death; in fact, they may never achieve this goal.

When discussing the therapeutic regimen of vitamin B12 for pernicious anemia with a client, the nurse explains that: a) Weekly Z-track injections provide needed control b) Daily intramuscular injections are required for control c) Intramuscular injections once a month will maintain control d) Oral tablets of vitamin B12 taken daily will provide symptom control

c) Intramuscular injections once a month will maintain control Intramuscular injections bypass the vitamin B12 absorption defect (lack of intrinsic factor, the transport carrier component of gastric juices). A monthly dose usually is sufficient because it is stored in active body tissues, such as the liver, kidney, heart, muscles, blood, and bone marrow. The Z-track method need not be used as it is for iron dextran injections. Because it is stored and only slowly depleted, injections once a month usually are sufficient. Vitamin B12 cannot be taken by mouth because of the lack of intrinsic factor.

A nurse is interacting with a depressed, suicidal client. What themes in the client's conversation are of most concern to the nurse? a) Power b) Betrayal c) Loneliness d) Hopelessness e) Indecisiveness

c) Loneliness d) Hopelessness Loneliness and a sense of isolation may play a role in the intent to commit suicide. A real or perceived lack of support increases the risk for suicide because there is no "lifeline of caring." The main factor leading to acting-out on suicidal impulses is the feeling of hopelessness; there are no longer reasons to live. The struggle for power and dominance is more commonly encountered in the verbalizations of clients with paranoid schizophrenia. Betrayal is a feeling more often verbalized by clients with a diagnosis of a borderline personality disorder. An indecisive individual usually will not make the decision to commit suicide.

When a client is experiencing hypovolemic shock with decreased tissue perfusion, the nurse expects that the body initially attempts to compensate by: a) Producing less antidiuretic hormone (ADH) b) Producing more red blood cells c) Maintaining peripheral vasoconstriction d) Decreasing mineralocorticoid production

c) Maintaining peripheral vasoconstriction With shock, arteriolar vasoconstriction occurs, raising the total peripheral vascular resistance and shifting blood to the major organs. With shock, more ADH is produced to promote fluid retention, which will elevate the blood pressure. Although producing more red blood cells is a response to hypoxia, peripheral vasoconstriction is a more effective compensatory mechanism. With shock the mineralocorticoids increase to promote fluid retention, which elevates the blood pressure. Test-Taking Tip: Many times the correct answer is the longest alternative given, but do not count on it. NCLEX item writers (those who write the questions) are also aware of this and attempt to avoid offering you such "helpful hints."

The left foot of a client with a history of intermittent claudication becomes increasingly cyanotic and numb. Gangrene of the left foot is diagnosed, and because of the high level of arterial insufficiency, an above-the-knee amputation (AKA) is scheduled. The response that demonstrates emotional readiness for the surgery is when the client: a) Explains the goals of the procedure b) Displays few signs of anticipatory grief c) Participates in learning perioperative care d) Verbalizes acceptance of future dependency needs

c) Participates in learning perioperative care Active participation in self-care indicates a readiness to learn; it demonstrates that the client is interested in future expectations. Explaining the goals of the procedure may indicate intellectual readiness but not necessarily emotional readiness. An expected change in body image precipitates the grieving process; a client may be in denial if no concerns are expressed. The client need not be dependent; verbalizing acceptance of future dependency needs indicates the need for more teaching and emotional support.

The nurse is caring for a patient who is experiencing incontinence. Which nursing intervention will the nurse include in the patient's plan of care? a) Deep breathing exercises b) Keeping the perineum clean c) Pelvic muscle (Kegel) exercises d) Awakening the patient at night to void

c) Pelvic muscle (Kegel) exercises Kegel exercises are effective in correcting incontinence as they help to tighten the pelvic muscles. Deep breathing exercises will calm the patient and improve voiding, but they may not prevent incontinence. The genitalia and perineum are kept clean in all patients with urinary problems to prevent infection. Asking the patient to void before bedtime is more effective to prevent night-time incontinence than awakening the patient at night.

A client with a distal femoral shaft fracture is at risk for developing a fat embolus. The nurse considers that a distinguishing sign that is unique to a fat embolus is: a) Oliguria b) Dyspnea c) Petechiae d) Confusion

c) Petechiae At the time of a fracture or orthopedic surgery, fat globules may move from the bone marrow into the bloodstream. Also, elevated catecholamines cause mobilization of fatty acids and the development of fat globules. In addition to obstructing vessels in the lung, brain, and kidneys with systemic embolization of small vessels from fat globules, petechiae are noted in the buccal membranes, conjunctival sacs, hard palate, chest, and anterior axillary folds; these adaptations only occur with a fat embolism. Oliguria, dyspnea, and confusion are signs of an embolus, but are not specific to a fat embolus.

A client develops a nosebleed (epistaxis) and seeks treatment at a first-aid station. The nurse can help control the bleeding by: a) Tilting the head backward b) Packing the nose with tissue c) Pinching the nostrils together d) Instructing the client to blow the nose gently

c) Pinching the nostrils together Pinching the nostrils together places pressure on the bleeding vessel, which can help control the bleeding. Tilting the head back will cause the blood to be swallowed, which can result in vomiting. Packing the bleeding nostril with tissue may cause further damage if done too firmly; some of the tissue may be left in the nose, causing an additional problem. Blowing the nose can prevent clotting, which can result in prolonged bleeding.

A nurse is caring for a patient with a risk for increased intracranial pressure (ICP). This patient should remain in which position? a) Prone b) Supine c) Semi-Fowler d) Whichever position is most comfortable for the patient

c) Semi-Fowler The patient should be placed in the semi-Fowler position to elevate the head and allow gravity to prevent increases in ICP. Prone and supine positions allow gravity to increase the ICP. The patient should not change position often.

An infant who weighed 7.5 lb at birth now weighs 15 lb at 1 year. The nurse concludes that this infant's weight gain: a) Suggests possible maternal neglect b) Reflects the expected growth curve c) Signifies an inadequate weight gain d) Indicates insufficient dietary protein

c) Signifies an inadequate weight gain The infant's weight signifies inadequate weight gain, according to the weight charts of the National Center for Health Statistics. An infant's total weight at the end of the first year should be three times the birth weight. Suggesting maternal neglect is a judgmental reaction; more evidence is needed to come to this conclusion. There are not enough data to determine whether the infant has an insufficient intake of dietary protein.

The nurse is caring for a patient who is prescribed oxymetazoline (Afrin) for nasal congestion. What instruction should the nurse give to ensure patient safety? a) avoid exposure to sunlight. b) Administer medication for 10 days. c) Stop the medication after 4 days. d) Mix the medication with orange juice.

c) Stop the medication after 4 days. Oxymetazoline (Afrin) is used to clear nasal congestion in patients with respiratory tract infections. The nurse should ask the patient to stop the drug after 4 days. The drug may cause rebound effect in the patient causing severe congestion if administered more than four doses. Patients on diuretics are informed to avoid sun exposure because they cause phototoxicity as a side effect. Oxymetazoline (Afrin) does not cause phototoxicity. If the medication is used for ten days consecutively, it may cause rebound congestion in the patient. Medications such as potassium iodide are mixed with fruit juice for administration because of their strong odor, unlike oxymetazoline (Afrin).

A health care provider in the emergency department identifies that a client is in mild hypovolemic shock. Which type of drug should the nurse anticipate will be prescribed? a) Loop diuretic b) Cardiac glycoside c) Sympathomimetic d) Alpha-adrenergic blocker

c) Sympathomimetic Sympathomimetics are vasopressors that induce arterial constriction, which increases venous return and cardiac output. Diuretics promote excretion of fluid, which will exacerbate hypovolemia associated with hypovolemic shock. Cardiac glycosides slow and strengthen the heartbeat; they do not increase the blood pressure and may decrease it. Alpha-adrenergic blockers decrease peripheral resistance, resulting in a decreased blood pressure.

The primary health care provider prescribes prednisone (Deltasone) to a child with nephritic syndrome. Which information for child care does the nurse repeat to the parents? Select all that apply. a) Teach the parents about foods that have low protein content. b) Teach the parents to insist the child eat three large meals per day. c) Teach the parents to minimize the child's exposure to communicable diseases. d) Teach the parents about food that are rich in sodium to be included in the child's diet.

c) Teach the parents to minimize the child's exposure to communicable diseases. Prednisone suppresses the immune system and may increase the risk of infection. Therefore, the parents are instructed to keep the child free from exposure to communicable diseases. Nephritic syndrome results in proteinuria. Loss of protein in the urine causes a decrease in the protein level of the blood (hypoproteinemia). So, the nurse should tell the parents to provide a rich diet to the child.The child should be offered small, frequent meals that are served in an attractive manner.The child should not be given a diet with high levels of salt and fluids because they further increase edema in the child.

The nurse notices that a patient's urine specimen is cloudy. What does the nurse interpret from this finding? a) The patient is diabetic. b) The urine contains bilirubin. c) The urine contains bacteria. d) The patient's fluid intake is inadequate.

c) The urine contains bacteria. If the urine specimen is cloudy it indicates that the urine contains bacteria or large amounts of protein. Cloudy urine does not indicate that the patient is diabetic as diabetes cannot be confirmed before testing the urine. If the urine contains bilirubin, its color is very dark amber. If a patient's fluid intake is inadequate, the urine will be more concentrated. In that case, the urine will be dark instead of cloudy.

A nurse providing care to a client who had major abdominal surgery monitors the client for postoperative complications. Which clinical findings are indicators of impending hypovolemic shock? a) Diuresis, irritability, and fever b) Lethargy, cold skin, and hypertension c) Thirst, cool skin, and orthostatic hypotension d) Bounding pulse, restlessness, and slurred speech

c) Thirst, cool skin, and orthostatic hypotension With hypovolemic shock extravascular fluid depletion leads to thirst, peripheral vasoconstriction produces cool skin, and inadequate venous return leads to orthostatic hypotension. Although irritability may occur with hypovolemic shock, decreased blood flow to the kidney leads to oliguria; the temperature usually decreases with hypovolemic shock. Restlessness, not lethargy, occurs with hypovolemic shock; hypotension and cool skin are signs of hypovolemic shock. Although restlessness may occur with hypovolemic shock, the pulse is thready, not bounding; subtle changes in sensorium will not result in slurred speech.

A client with cancer of the thyroid is scheduled for a thyroidectomy. What should the nurse teach the client? a) The dietary intake of carbohydrates must be restricted. b) Chemotherapy may be used in conjunction with the surgery. c) Thyroxine replacement therapy will be required indefinitely. d) A tracheostomy requires an alternate means of communication.

c) Thyroxine replacement therapy will be required indefinitely. Thyroxine is given postoperatively to suppress thyroid-stimulating hormone (TSH) and prevent hypothyroidism. Increased intake of carbohydrates and proteins is needed because of the increased metabolic activity associated with hyperthyroidism. Chemotherapy is uncommon; radiation may be used to eradicate remaining tissue. A tracheostomy is not planned; it is needed only in an emergency related to respiratory distress.

A patient reports to the nurse that urination is painful. What clinical findings does the nurse expect to see in the patient's report? a) Kidney failure b) Diabetes insipidus c) Trauma to urethra d) Decreased muscle tone

c) Trauma to urethra Painful urination is called dysuria and is caused due to an infection or trauma to the urethra and the bladder. Kidney failure causes oliguria, in which the urine output decreases. Diabetes insipidus is a decrease in the production of antidiuretic hormone; it causes polyuria, or excessive urination. Decreased muscle tone causes incontinence.

The nurse observes that a patient has difficulty expectorating sputum while collecting a sputum specimen. Which intervention would help to collect a good specimen? a) Ask the patient to rinse the mouth with water. b) Collect the specimen after the patient has meals. c) Use hypertonic saline aerosol mist for inhalation. d) Keep the patient on nothing by mouth (NPO) status.

c) Use hypertonic saline aerosol mist for inhalation. Applying a hypertonic saline aerosol mist helps to produce good sputum specimen when the patient has a difficulty to raise sputum during specimen collection. The patient is asked to rinse the mouth with water before expectorating in order to decrease the contamination. Sputum specimens are collected before meals to avoid any possible emesis from coughing. Patients who have difficulty in producing sputum may be dehydrated; hence they are given adequate fluids and NPO status is avoided.

A client with a long history of cardiovascular problems, including angina and hypertension, is scheduled to have a cardiac catheterization. During preprocedure teaching, the nurse explains to the client that the major purpose for catheterization is to: a) Obtain the pressures in the heart chambers b) Determine the existence of congenital heart disease c) Visualize the disease process in the coronary arteries d) Measure the oxygen content of various heart chambers

c) Visualize the disease process in the coronary arteries Angina usually is caused by narrowing of the coronary arteries; the lumen of the arteries can be assessed by cardiac catheterization. Although pressures can be obtained, they are not the priority for this client; this assessment is appropriate for those with valvular disease. Determining the existence of congenital heart disease is appropriate for infants and young adults with cardiac birth defects. Measuring the oxygen content of various heart chambers is appropriate for infants and young children with suspected septal defects.

Each year, a client takes many trips to other countries. The client reports leg swelling during the long flights. The nurse should advise the client to, during the flights: a) Relax in a reclining position b) Sit upright with legs extended c) Walk around at least every hour d) Sit in any position that relieves pressure on the legs Eugene off target

c) Walk around at least every hour Muscle contraction associated with walking prevents pooling of blood in the extremities and dependent edema. Movement is required, not inactivity. Sitting in any position that relieves pressure on the legs does not include movement, which is essential to prevent thrombus formation.

A client comes to the emergency department because of minimal urinary output despite drinking adequate fluid. The client's blood pressure is 190/94. For what additional clinical manifestation associated with this data, should the nurse assess the client? a) Thirst b) Urinary retention c) Weight gain d) Urinary hesitancy

c) Weight gain Oliguria is the inability to produce more than 400 to 500 mL of urine daily. Expected daily urinary output is 1000 to 3000 mL daily, depending on the volume of fluid intake. If urine is not being produced in the presence of an average daily intake, fluid will be retained and reflected in weight gain. One liter of fluid weighs 2.2 pounds. Excess fluid contributes to an increase in circulating blood volume, causing hypertension. Thirst is associated with dehydration, not hypertension and oliguria. Urinary retention is unrelated to hypertension and oliguria. Urinary retention is the inability to empty the bladder caused by urethral obstruction, lesions involving the nerve pathways to and from the bladder or involving reflex centers in the brain or spinal cord, and medications. Urine is retained in the bladder until increased abdominal pressure causes urine to be lost involuntarily. Urinary hesitancy is an involuntary delay in initiating urination and is unrelated to hypertension and oliguria.

When obtaining the history of a client recently diagnosed with type 1 diabetes, the nurse expects to identify the presence of: a) Edema b) Anorexia c) Weight loss d) Hypoglycemic episodes

c) Weight loss Protein and lipid catabolism occur because carbohydrates cannot be used by the cells; this results in weight loss and muscle wasting. Dehydration, not edema, is more likely to occur because of the polyuria associated with hyperglycemia. Polyphagia, not anorexia, occurs with diabetes as the client attempts to meet metabolic needs. Hyperglycemia, not hypoglycemia, is present in both type 1 and type 2 diabetes. STUDY TIP: Begin studying by setting goals. Make sure they are realistic. A goal of scoring 100% on all exams is not realistic, but scoring an 85% may be a better goal.

Which pediatric patient may require a chelation therapy? a) a child with skin nodules b) a child with subluxated hip c) a child with lead poisoning d) a child with periorbital edema

c) a child with lead poisoning Chelation therapy is a pharmacologic treatment provided for the child with lead poisoning. The therapy tends to bind the metal and helps in its excretion from the body. Skin nodules and periorbital edema are seen in a patient who has neuroblastoma. Subluxated hip is a clinical manifestation of spina bifida.

You are working in the ED when a woman in active labor comes in, stating that she "has to push". You quickly determine that the infant is crowning. The mother had been experiencing moderate contractions for 2 hours when the contractions suddenly accelerated. This is the mother's third baby. The physician is not present. You call the physician, who tells you that he is 15 minutes away. What should you do? a) remain calm and provide clear and concise directions to the mother and the person accompanying her b) check for the presence of an intact amniotic sac c) all of these d) encourage the mother to blow or pant through contractions to slow the rate of expulsion

c) all of these All these interventions should be performed during a precipitous delivery. If the amniotic sac is intact it will need to be ruptured prior to delivery of the head. Delivery of the head should be achieved in a controlled fashion. Once the infant has been delivered, place the infant on the mother's abdomen.

A young child is brought to the clinic to have test done for an endocrine disorder. What is the most common endocrine disorder in children? a) hypothyroidism b)hyperthyroidism c) diabetes mellitus 1 d) diabetes mellitus 2

c) diabetes mellitus 1 Diabetes mellitus type 1 results from autoimmune destruction of the beta cells. It typically starts in children or young adults who are slim. Hypothyroidism in children may result from a congenital defect. Hyperthyroidism is rare in children. Diabetes mellitus type 2 arises from insulin resistance. In the past it occurred in those who were older than 45, overweight, and sedentary.

Your patient has metabolic alkalosis. You would expect to see which of the following ABG results? a) a low pH, low bicarbonate and low carbon dioxide b) a high pH, low bicarbonate level and low carbon dioxide c) elevated pH, bicarbonate and carbon dioxide d) a low pH, high bicarbonate and high carbon dioxide

c) elevated pH, bicarbonate and carbon dioxide Elevated pH, bicarbonate and carbon dioxide typically indicate severe vomiting has altered chemistry of the blood. Alkalotic agents, such as bicarbonates, and hyperaldosteronism are other potential causes of metabolic alkalosis.

A patient has metabolic acidosis. The nurse anticipates the patient will demonstrate which symptom? a) tachycardia b) bradycardia c) increased respiratory rate d) decreased respiratory rate

c) increased respiratory rate Metabolic acidosis causes the lungs to increase respirations to blow off extra carbon dioxide and increase blood pH. Decreased respiratory rate occurs in metabolic alkalosis, in which the body must conserve carbon dioxide to decrease blood pH.

Your female patient experiences incontinence when coughing or sneezing and wears an incontinence pad. She states that she is not incontinent at other times. She asks you if there is anything she can do to "strengthen her bladder." You know that strengthening of the pelvic floor muscles can be achieved by performance of: a) aerobic exercises b) breath holding exercises c) Kegel exercises d) isometric exercises

c) kegel exercises

A nurse is caring for a patient with meningitis and implements which transmission-based precautions for the client? a) standard precautions b) isolation precautions c) private room or cohort client d) no precautions are needed, meningitis is not a communicable disease

c) private room or cohort client Meningitis is transmitted by droplet infection. Precautions for this disease include private room or cohort client and use of a standard mask. When appropriate, the client must wear a mask when leaving the room, not the staff.

The nurse performs auscultation of the lungs in a patient and observes unilateral, high-pitched, musical, and whistle-like sound during inspiration. What does the nurse document about the adventitious breath sounds of the patient? a) rhonci b) coarse crackles c) sibilant wheezes d) pleural friction rub

c) sibilant wheezes Sibilant wheezes are abnormal sounds superimposed on breath sounds. These are unilateral or bilateral, high-pitched, musical, and whistle-like sounds during inspiration or expiration. Rhonchi or sonorous wheezes are loud, low, coarse sounds like a snore that are heard at any point of inspiration or expiration. Coarse crackles are loud bubbly sounds heard in early inspiration. Pleural friction rub refers to dry, creaking, grating and low-pitched sounds with a machinelike quality heard during both inspiration and expiration.

Your patient has diarrhea from an infectious cause and has been prescribed antibiotics. He asks you what he should be eating until the diarrhea had resolved. Choose the best response. a. "caffeine will decrease bouts of diarrhea due to its dehydrating effects" b. "eating spicy foods will kill the bacteria causing diarrhea" c. "eat bland foods such as toast, apples, rice, and bananas" d. "eating a high fiber diet will decrease diarrhea"

c. "eat bland foods such as toast, apples, rice, and bananas"

The most common sexually transmitted disease in the United States is: a. HIV b. Gonorrhea c. Chlamydia

c. Chlamydia The most common STD in the United States is chlamydia. Around 5 million people in the US contract the disease annually. In addition, it is estimated that around one billion health care dollars are spent each year on the diagnosis and treatment of chlamydia. Providing education to men and women about symptoms, transmissions, and prevention is essential.

Your patient with heart failure has been responding well to treatments that include medications such as ACE inhibitors and a loop diuretic. Today, the client is complaining about leg weakness and is refusing to ambulate. What is most likely occurring with this client? a. Hyperkalemia b. Hyponatremia c. Hypokalemia d. Hypernatremia

c. Hypokalemia Hypokalemia, or low potassium, often occurs as the result of treatments with loop diuretics like furosemide (Lasix). The signs and symptoms of hypokalemia include muscular weakness, pain and cramping, as well as serious cardiac dysrhythmias. Clients taking loop diuretics should be closely monitored for hypokalemia and also given potassium supplementation when indicated.

A 21-year-old woman comes into the emergency room, stating that she sustained injuries in a fall. She has a black eye and two fractured ribs. She has two children at home and is pregnant with her third child. She has no insurance and states she is unemployed. Her boyfriend is not accompany her to the emergency room. You suspect her injuries resulting from abuse. You ask the patient if she has been abused and she denies it. What is your best course of action? a. Do nothing it's none of your business b. Document your suspicions but do nothing c. Report the suspected abuse d. Tell the physician that you suspect the patient has been abused

c. Report the suspected abuse

A patient is taking rivastigmine (Exelon). The nurse should teach the patient and family which information about rivastigmine? a. That hepatotoxicity may occur b. That the initial dose is 6 mg t.i.d. c. That GI distress is a common side effect d. That weight gain may be a side effect

c. That GI distress is a common side effect

There has been a large fire in your facility, causing damage from water and smoke. You have been notified to prepare a list of all patients than can be safely discharged home. Which of the following patients could be safely discharged? a. a 58 year old male palliative care patient with lung cancer or continuous morphine for pain control and oxygen by mask at 8 liters per minute b. a 2 year old female with epiglottis and inspiratory stridor c. a 25 year old female patient receiving IV antibiotics for pyelonephritis (last dose due in 2 hours)

c. a 25 year old female patient receiving IV antibiotics for pyelonephritis (last dose due in 2 hours) The patient with pyelonephritis could be discharged home on oral antibiotics and counseled to return if her condition deteriorates. The other patients are unstable or require continuous nursing care

A patient undergoes laboratory testing with a basic metabolic panel. The nurse in the outpatient clinic notes a potassium level of 2.8 mEq/L. The nurse proceeds with the following teaching: a. encourage intake of foods such as apples, cabbage and corn b. encourage intake of foods such as grapes, eggplant, and peaches c. encourage intake of foods such as bananas, leafy green vegetables, and beans d. encourage intake of foods of foods such as cauliflower, applesauce, and blackberries

c. encourage intake of foods such as bananas, leafy green vegetables, and beans A normal potassium level is between 3,5 and 5 mEq/L. The patient's potassium level is low, and therefore the nurse should proceed with encouraging intake of foods high in potassium. Such foods include bananas, leafy green vegetables, and beans. The other foods mentioned are all low in potassium and would be more useful for a person with chronic kidney disease who may have high potassium levels.

The LPN is working as a home health nurse. She is visiting a new patient in their home and conducts a home safety assessment. Her assessment should include all of the following except: a. location of medications b. adequacy of lighting c. food preferences d. placement of furniture

c. food preferences A home safety assessment should be conducted by the home health nurse and include a number of areas for review. It is important to assess adequacy of lighting and inspect the areas in which the patient walks or moves around. Within this area, also assess the location and presence of furniture or large items that may provide barriers to a patient's mobility. The nurse should also obtain an understanding of the location of medications and cleaning supplies, to be certain they are within easy reach of the patient. Food preparation and storage practices should be assessed to ensure there is no risk of poisoning, but food preferences would not be important in conducting the home safety assessment.

Menopausal symptoms include all of the following except: a. depression b. headache c. narcolepsy d. hot flashes

c. narcolepsy Menopause is the cessation of menstruation. Generally speaking, the ovaries no longer respond to stimulation form the pituitary gland, resulting in failure to ovulate. Hormonal changes from menopause can trigger a number of unpleasant symptoms. The most common menopausal symptoms include headache, insomnia, fatigue, depression, and hot flashes. While hormonal replacement therapy was at one time considered standard care, there has been controversy regarding its use over the last several years due to studies revealing a potential association with cancer.

Tom is the licensed practical practical nurse caring for a busy group of 12 patients on a medical/surgical unit. Within his patient assignment , there are eight patients with DM who need to have their blood glucose checked prior to dinner. In a pinch for time, Tom only performs blood glucose checks on 6 of the eight patients. This is an example of: a. Beneficence b. Malpractice c. negligence d. prioritization

c. negligence

What preparation is required on the part of the female patient prior to having a Pap test (Pap smear)? a. fleet enema b. shaving c. no preparation is necessary d. douching

c. no preparation is necessary No preparation is needed for a Pap smear. Women may be advised to refrain from douching or using vaginal creams or suppositories within 24 hours of test. Refraining from sexual intercourse is unnecessary. A rectal exam is sometimes performed along with the Pap smear test, but there is no need to prepare the bowel

You are caring for a patient admitted with dehydration. Blood work has been drawn as ordered. You have started an IV of normal saline, which is infusing at 150 mL/hour. You are performing your admission assessment when you receive a call from the lab that your patient's BUN level is 102 mg/dL. What is your first action? a. decrease the rate of IV infusion b. do nothing-this level is normal for a patient who is dehydrated c. notify the physician-this is critical level d. increase the rate of the IV infusion

c. notify the physician-this is critical level BUN (blood urea nitrogen) is a reflection of renal function. Normal values are 7 to 20 mg/dL. The patient's BUN is at critical level-notify the physician. You should not change the IV rate without a physician's order.

Which of the following assistive techniques should the be used to transfer a patient who can bear weight from the bed to the chair? a. mechanical lift b. slide transfer c. pivot transfer d. assisted transfer

c. pivot transfer

Your client is to have an NG tube inserted. To mark the tube prior to insertion, you should: a. place the tip of the tube at the corner of the patient's eye and extend the tip to the earlobe, and then to the tip of the xiphoid process. b. place the tip of the tube at the corner of the mouth and extend the tip to the top of the patient's ear, and then to the umbilicus. c. place the tip of the tube at the patient's nostril and extend the tip of the earlobe, and then to the tip of the xiphoid process. d. place the tip of the tube at the patient's nostril , extend it to the tip of the earlobe, and then to the base of the ribcage.

c. place the tip of the tube at the patient's nostril and extend the tip of the earlobe, and then to the tip of the xiphoid process.

Your pregnant client has a high level of alpha fetoprotein. What does this high level suggest? a) congenital nephrosis b) placenta prebia c) uterine atony d) all of the above

congenital nephrosis Alpha fetoprotein levels (AFP) are analyzed at 15 to 20 weeks gestation. Elevated levels of AFP are associated with chromosomal abnormalities such as open neural tube defects, open abdominal defects, and congenital nephrosis. Decreased levels are associated with down syndrome and other chromosomal anomalities. Further testing is indicated when the AFP level is abnormal.

A client with heart disease has been reading on the Internet about the anatomy and physiology of the heart and tells the nurse, "I'm so confused." The nurse reinforces the pattern of circulation in the body. Which client statement indicates an understanding? a) "Blood enters the heart through the ductus arteriosus, flows into the left side of the heart, and exits via the aorta into the systemic circulation." b) "Blood enters the heart from the inferior vena cava; it then flows through the left atrium into the left ventricle, then into the lungs, and back into the aorta." c) "Blood enters the heart from the aorta, flows into the right atrium and right ventricle, through the lungs, then into the left atrium and left ventricle, and finally exits through the superior vena cava." d) "Blood enters the right atrium via the superior and inferior vena cava, flows to the right ventricle and then into the lungs, returns from the lungs to the left atrium and left ventricle, and exits out the aorta."

d) "Blood enters the right atrium via the superior and inferior vena cava, flows to the right ventricle and then into the lungs, returns from the lungs to the left atrium and left ventricle, and exits out the aorta." Stating that blood enters the right atrium via the superior and inferior vena cava, flows to the right ventricle and then into the lungs, returns from the lungs to the left atrium and left ventricle, and exits out the aorta correctly describes the flow of blood through the heart after birth. The ductus arteriosis is a fetal structure that is not present in the adult heart. Blood enters the right side of the heart via the inferior and superior vena cava; blood flows from the right atrium, to the right ventricle, to the lungs, and then to the left atrium. Blood exits, not enters, the heart from the aorta.

A client with a dysrhythmia is admitted to telemetry for observation. In the morning, the client asks for a cup of coffee. What is the nurse's best response? a) "Hot drinks such as coffee are not good for your heart." b) "Coffee is not permitted on the diet that was prescribed for you." c) "You cannot have coffee. I can bring you a cup of tea if you like." d) "Coffee has caffeine, which can affect your heart. It should be avoided."

d) "Coffee has caffeine, which can affect your heart. It should be avoided." Caffeine is a stimulant that causes vasoconstriction and is contraindicated for a client with a dysrhythmia. Although "Hot drinks such as coffee are not good for your heart" is a true statement, it does not provide information as to why it is not good for the heart. Adherence to a medical regimen increases when the client understands the rationale for recommendations. Tea contains caffeine and should be avoided by a client with a dysrhythmia.

A nurse is providing education to a patient with tuberculosis who requires drug therapy. The patient demonstrates an understanding of the principles of therapy with which statement? a) "Only one medication is necessary for a cure." b) "I should take all four medications until I feel better." c) "Regardless of medical treatment, tuberculosis cannot be cured." d) "I must take all four drugs for 6 to 9 months for prevention of resistant organisms."

d) "I must take all four drugs for 6 to 9 months for prevention of resistant organisms." Tuberculosis requires the administration of four different medications for 6 to 9 months to prevent resistant organisms and cure the disease.

The nurse is caring for a patient who has nephritis. What instructions should the nurse include while educating the patient about health promotion? a) "Space out your activities to lessen edema." b) "Increase the sodium content in your diet." c) "Increase the protein content in your diet." d) "Report any incidents of hematuria and edema."

d) "Report any incidents of hematuria and edema." Nephritis is characterized by inflammation of kidneys and manifests as edema over the face, especially eyes, anorexia, dysuria, and sanguineous urine. Increased serum creatinine, potassium, antistreptolysin-O titer, ESR, and red blood cells and protein in urine, is found in the patient. The nurse should ask the patient to report any incidence of hematuria and edema. Reporting the incidence of hematuria and edema helps to evaluate the effectiveness of the medication and the status of disease condition. A patient with nephritis shows signs of edema and increased blood pressure. The patient should be instructed to rest until the edema and blood pressure reduces. The patient should be instructed to perform some light activities to lessen fatigue, only after the edema and blood pressure subsides.The patient should be instructed to restrict the protein and sodium content in the diet, as the diet plan may increase the edema and blood pressure.

A client with a tentative diagnosis of pernicious anemia is scheduled for a Schilling test. Which body process associated with vitamin B 12 is assessed with the Shilling test? a) Storage b) Digestion c) Production d) Absorption

d) Absorption With the Schilling test, radioactive vitamin B 12 is administered, and its absorption and excretion are ascertained. Storage is not measured by this test. Digestion is not measured by this test. Vitamin B 12 is not produced in the body. Pernicious anemia is caused by an inability to absorb vitamin B 12 as a result of a lack of intrinsic factor in gastric juices.

A patient arrives at the clinic with edema of the face, anorexia, nocturia, and general malaise. The nurse notes crackles on auscultation of the lungs, jugular vein distention, and cola-colored urine. The nurse learns the patient had a beta-hemolytic streptococcal infection 2 weeks ago. The nurse anticipates which diagnosis? a) Impetigo b) Rheumatic fever c) Infective endocarditis d) Acute glomerulonephritis

d) Acute glomerulonephritis Acute glomerulonephritis is characterized by general malaise, fluid overload, and decreased urine output. It is caused by inflammation of the glomeruli as a result of infection by the beta-hemolytic streptococci. Impetigo, rheumatic fever, and infective endocarditis do not explain the patient's symptoms.

A health care provider prescribes enoxaprarin (Lovenox) 30 mg subcutaneously daily. To ensure client safety, which measure would the nurse take when administering this medication? a) Remove air pocket from prepackaged syringe before administration b) Rub site after administration c) Push over two minutes d) Administer in the abdomen

d) Administer in the abdomen Enoxaprarin specifically targets blood clots throughout the body and carries a lower risk of hemorrhage than that associated with the drugs heparin and warfarin. Enoxaprarin is administered once a day through a subcutaneous injection site around the naval. Enoxaprarin should be injected into the fatty tissue only, which is why the abdomen is the recommended injection site. Avoid administering in a muscle. Manufacturer recommendations indicate the air pocket from prepackaged syringes not be removed before administration. Rubbing the site is contraindicated as it can cause bruising. There are no recommendations to push medication over two minutes.

A health care provider prescribes ophthalmic drops for a client. What should a nurse include in the instructions for a client learning to self-administer eye drops? a) Lie on the unaffected side for administration. b) Instill drops onto the pupil to promote absorption. c) Close eyes tightly after administering the eye drops. d) Apply pressure to the nasolacrimal duct after instillation.

d) Apply pressure to the nasolacrimal duct after instillation. Applying pressure prevents absorption into the duct, which may lead to systemic effects. Lying on the unaffected side is indicated for ear drops. Tilting the head back and looking up facilitate the instillation of eye drops. Eye drops should be instilled into the conjuctival sac, not on the pupil. Closing the eyes tightly will force drops out of the eye.

A client with impaired peripheral pulses and signs of chronic hypoxia in a lower extremity has a femoral angiogram. What is the priority nursing action after the angiogram? a) Elevate the foot of the bed. b) Encourage the client to void. c) Maintain the high-Fowler position. d) Assess the client's affected extremity.

d) Assess the client's affected extremity. Because of the trauma associated with insertion of the catheter during the procedure, the involved extremity should be assessed for sensation, motor ability, and arterial perfusion; hemorrhage or an arterial embolus can occur. The client has an arterial problem, and perfusion is promoted by keeping the legs at the level of the heart. A general anesthetic is not used; therefore, voiding usually is not a concern. Maintaining the high-Fowler position is unsafe because it increases pressure in the groin area, which may dislodge the clot at the catheter insertion site, resulting in bleeding; it also impedes arterial perfusion and venous return.

What client response indicates to the nurse that a vasodilator medication is effective? a) Pulse rate decreases from 110 to 75 b) Absence of adventitious breath sounds c) Increase in the daily amount of urine produced d) Blood pressure changes from 154/90 to 126/72

d) Blood pressure changes from 154/90 to 126/72 Vasodilation will lower the blood pressure. The pulse rate is not decreased and may increase. Breath sounds are not directly affected by vasodilation, although vasodilator medications can decrease preload and afterload, which could indirectly affect breath sounds in heart failure. The urine output is not affected immediately, although control of blood pressure can help preserve renal function over time.

A client was diagnosed with cancer of the head of the pancreas two months ago. The client is admitted to the hospital with weight loss, severe epigastric pain, and jaundice. When performing the client's assessment, the nurse expects the client's stool to be what color? a) Green b) Brown c) Red-tinged d) Clay-colored

d) Clay-colored Tumors of the head of the pancreas usually obstruct the common bile duct where it passes through the head of the pancreas to join the pancreatic duct and empty at the ampulla of Vater into the duodenum. The feces will be clay-colored when bile is prevented from entering the duodenum. Green stools may occur with prolonged diarrhea associated with gastrointestinal inflammation. The feces are brown when there is unobstructed bile flow into the duodenum. Inflammation or ulceration of the lower intestinal mucosa results in blood-tinged stools. Test-Taking Tip: Note the number of questions and the total time allotted for the test to calculate the times at which you should be halfway and three-quarters finished with the test. Look at the clock only every 10 minutes or so.

The nurse is planning nutritional education for a client with lower extremity arterial disease (LEAD). What diet modifications should the nurse include? a) Decreasing both fluid and sodium intake b) Increasing both calcium and potassium intake c) Increasing both vitamin E and refined grain intake d) Decreasing both cholesterol and saturated fat intake

d) Decreasing both cholesterol and saturated fat intake Lower extremity arterial disease frequently is accompanied by generalized atherosclerosis; decreasing both cholesterol and saturated fat intake will help decrease lipid buildup on artery walls. Decreasing both fluid and sodium intake are inappropriate dietary modifications; this client does not have edema. Increasing both calcium and potassium is not appropriate for the client's condition because it may alter the client's electrolyte balance. Recent research indicates that supplemental vitamin E can precipitate cardiac problems and only should be taken when prescribed by a health care provider who can monitor the client's ongoing status. Increasing grain intake will add calories and may contribute to unnecessary weight gain.

What breathing exercises should the nurse teach a client with the diagnosis of emphysema? a) An inhalation that is prolonged to promote gas exchange. b) Abdominal exercises to limit the use of accessory muscles. c) Sit-ups to help strengthen the accessory muscles of respiration. d) Diaphragmatic exercises to improve contraction of the diaphragm.

d) Diaphragmatic exercises to improve contraction of the diaphragm. With emphysema the diaphragm is flattened and weakened; strengthening the diaphragm is desirable to maximize exhalation. Prolonged exhalations are more desirable; clients with emphysema have an increased residual volume, which eventually causes a barrel chest. Abdominal exercises enhance, not limit, the accessory muscles of respiration which are needed as a compensatory mechanism for clients with emphysema. Sit-ups are too strenuous for clients with emphysema.

What is a grave disease process in which the patient experiences both bleeding and intravascular clotting at the same time? a) Lymphedema b) Multiple myeloma c) von Willebrand disease d) Disseminated intravascular coagulation (DIC)

d) Disseminated intravascular coagulation (DIC) DIC is a grave disease process in which the patient experiences both bleeding and intravascular clotting at the same time. It results from the overstimulation of clotting and anticlotting processes in response to disease or injury, including septicemia, obstetric complications, malignancies, tissue trauma, transfusion reactions, burns, shock, or snake bites. von Willebrand disease is an inherited bleeding disorder characterized by abnormally slow coagulation of blood and spontaneous episodes of gastrointestinal bleeding, epistaxis, and gingival bleeding caused by a mild deficiency of factor VIII. It is common during postpartum periods, as menorrhagia, and after surgery or trauma. It affects both women and men. Multiple myeloma is a malignant neoplastic immunodeficiency disease of the bone marrow. The tumor, composed of plasma cells, destroys osseous tissue, especially in flat bones, causing pain, fractures, and skeletal deformity. Lymphedema is a primary or secondary disorder characterized by the accumulation of lymph in soft tissue and edema. This can be caused by obstruction, increased amount of lymph, or removal of lymph channels and nodes; it may be hereditary.

The nurse is collecting the data of a patient who underwent a tracheostomy and hears crackles during auscultation of the lungs. What intervention may help this patient? a) Avoid providing pureed food to prevent choking. b) Support the patient to sleep in the supine position. c) Close the nostrils and ask to breathe from the mouth. d) Encourage coughing and deep breathing in the patient.

d) Encourage coughing and deep breathing in the patient. Tracheostomy is performed on patients with laryngeal cancer. Crackles in breathing sounds are observed when there is an obstruction in the airways due to excessive secretions. The nurse should ask the patient to take deep breaths and cough frequently in order to clear the obstructions. Pureed food does not cause choking and is preferable for a patient who has undergone a tracheostomy. Patients with obstructed airways are placed in the Fowler's position to clear the airway. The nurse would close the nose of the patient with epistaxis and ask the patient to breathe from the mouth. This intervention is not followed for the patients with tracheostomy.

A patient visits the clinic after a possible exposure to human immunodeficiency virus (HIV) and would like more information on how she will be tested for HIV. The nurse correctly responds that which test will be used first? a) Western blot test b) Rapid plasma reagin test c) Sexually transmitted infection screening d) Enzyme-linked immunosorbent assay (ELISA)

d) Enzyme-linked immunosorbent assay (ELISA) ELISA is performed first for HIV. If a positive result is obtained, the Western blot test is performed, which is a more specific test. The rapid plasma reagin test is used for syphilis. Sexually transmitted infection screening may be done in conjunction, but it is not used initially for assessing for contraction of HIV.

Which statement most accurately describes the disease tuberculosis (TB)? a) All strains of TB are resistant to antibiotic therapy. b) TB has the highest rates in the white U.S. population. c) TB is easily spread from person to person via respiratory secretions. d) Most people who become infected with the TB organism do not progress to the active disease stage.

d) Most people who become infected with the TB organism do not progress to the active disease stage. Most people who become infected with the TB organism do not progress to the active disease stage; they remain asymptomatic and noninfectious. These people will have a positive tuberculin skin test result, and chest radiograph results will be negative. These people retain a lifelong risk of developing reactivation TB if the immune system becomes compromised. A common misconception about TB is that it is easily spread—in fact, most people exposed to TB do not become infected. The body's first line of defense, the upper airway, prevents most inhaled TB organisms from ever reaching the lungs. TB rates in the white population are about half those in the nonwhite population. More than two thirds of reported cases occur in racial and ethnic minorities, particularly among Hispanic and African American populations. Not all strains of TB are resistant to antibiotic therapy. A growing percentage of new TB cases are resistant to the medications that are traditionally used to fight the disease.

During a prenatal checkup, the nurse learns that the patient has severe nausea and vomiting associated with weight loss. What intervention would the nurse provide for the patient after the primary health care provider's orders? a) Oral antibiotics b) Multivitamin tablets c) Oral rehydration fluid d) Normal saline infusion

d) Normal saline infusion Due to nausea and vomiting, the patient may have a fluid and electrolyte imbalance. The patient should be infused with normal saline to restore the fluid imbalance. Oral antibiotics should be prescribed if the patient has a Helicobacter pylori infection. Multivitamin tablets and oral rehydration fluids should be administered when the patient does not have symptoms of vomiting. The oral route is not preferable for the patients having severe nausea and vomiting, as this reduces the absorption of medications.

While conducting a cardiac assessment for a patient who is 78 years old, the nurse notes that the patient is suffering from sinus dysrhythmias. Which cells are reduced in this condition? a) Conduction cells in the bundle of His b) Conduction cells in the internodal tracts c) Conduction cells in the bundle branches d) Pacemaker cells in the sinoatrial (SA) node

d) Pacemaker cells in the sinoatrial (SA) node A reduction in the number of pacemaker cells in the SA node may account for sinus dysrhythmias in the older adult patient. Reductions in the number of conduction cells in the internodal tracts, bundle of His, and bundle branches contribute to the development of atrial dysrhythmias and heart blocks.

A client who is having a difficult labor is found to have cephalopelvic disproportion. Which medical order should the nurse question? a) Maintain NPO status. b) Start peripheral IV of ¼ NS. c) Record fetal heart tones every 15 minutes. d) Piggyback another 10-unit bag of oxytocin (Pitocin).

d) Piggyback another 10-unit bag of oxytocin (Pitocin). When there is cephalopelvic disproportion, a is indicated; infusing oxytocin (Pitocin) at this time could result in fetal compromise and uterine rupture. The nothing-by-mouth (NPO) status is appropriate in anticipation of a cesarean birth. A peripheral IV is needed not only for hydration but also for venous access if IV medications become necessary. The client probably has an electronic monitor recording the fetal heart rate and uterine contractions; the findings of these assessments should be documented regularly in accordance with hospital protocol. Test-Taking Tip: Do not panic while taking an exam! Panic will only increase your anxiety. Stop for a moment, close your eyes, take a few deep breaths, and resume review of the question.

A health care provider prescribes an antihypertensive medication. Which over-the-counter medication should the nurse teach the client to avoid because it has the potential to counteract the effect of the antihypertensive? a) Omeprazole (Prilosec) b) Acetaminophen (Tylenol) c) Docusate sodium (Colace) d) Pseudoephedrine (Sudafed)

d) Pseudoephedrine (Sudafed) Pseudoephedrine has a pressor effect that may counteract antihypertensive medications, causing an increase in blood pressure. Omeprazole does not interact with antihypertensives. However, it can increase the action of phenytoin, digoxin, clopidrogrel (Plavix), and cyclosporine. Acetaminophen does not have to be avoided when receiving an antihypertensive. Docusate sodium does not have to be avoided when receiving an antihypertensive.

A skier skied off the marked trail into the woods and collided with a tree. After several hours, the skier was found by the ski patrol and brought to the emergency department of the hospital. Moderate hypothermia (temperature range 87° to 90° F) is diagnosed. What clinical findings specific to moderate hypothermia should the nurse expect the client to exhibit? a) Confusion and lethargy b) Shivering and irrational behavior c) Absent reflexes and dilated pupils d) Rigidity and slowed respiratory rate

d) Rigidity and slowed respiratory rate Moderate hypothermia (temperature of 87° to 90° F) is characterized by rigidity, bradycardia, slowed respirations, metabolic and respiratory acidosis, and hypovolemia. Confusion and lethargy are seen with mild hypothermia (temperature of 90° to 95° F). Shivering occurs with mild hypothermia but disappears at temperatures lower than 92° F. Profound hypothermia, with temperatures lower than 87° F, is characterized by absent reflexes, profound bradycardia, and fixed, dilated pupils.

A patient is prescribed prochlorperazine (Compazine) to prevent chemotherapy-induced vomiting. During the follow-up visit, the nurse finds that the patient continues to have vomiting despite taking prochlorperazine (Compazine). Which medication does the primary health care provider prescribe in order reduce nausea and vomiting in the patient? a) Granisetron (Kytril) b) Ondansetron (Zofran) c) Metoclopramide (Reglan) d) Tetrahydrocannabinol (Marinol)

d) Tetrahydrocannabinol (Marinol) Vomiting is a common side effect of chemotherapy. Vomiting occurs due to excessive breakdown of the normal gastrointestinal cells. Prochlorperazine (Compazine) is an antiemetic agent. If prochlorperazine (Compazine) is ineffective for the patient, then tetrahydrocannabinol (Marinol) may be administered to reduce nausea and vomiting. Granisetron (Kytril), ondansetron (Zofran), and metoclopramide (Reglan) are not helpful to increase a patient's appetite.

A child is scheduled to receive a dose of digoxin (Lanoxin) for treatment of a ventricular septal defect. Upon reviewing the child's chart, the nurse discovers a low potassium level. What would the nurse infer? a) The child has hospital acquired infection. b) The child has developed brain deformities. c) The child requires increased digoxin (Lanoxin) dosage. d) The child is at increased risk of digoxin (Lanoxin) toxicity.

d) The child is at increased risk of digoxin (Lanoxin) toxicity Children with congenital heart disorders (ventricular septal defect) may be prescribed digoxin (Lanoxin). The nurse should regularly monitor potassium levels in the child who is on digoxin (Lanoxin), because low potassium levels enhance digoxin levels, consequently increasing the risk of digoxin toxicity.The child's white blood cell count would increase if the child developed hospital acquired infection. Reduced potassium levels are unrelated to infection. Diagnostic techniques, such as magnetic resonance imaging (MRI) would help in detecting brain deformities in a child. The digoxin (Lanoxin) dosage is likely to be reduced in the child with lowered potassium levels.

A nurse is teaching a class of parents with autistic children about the signs and symptoms of autism. The nurse should include that one of the hallmark autism characteristics is which? a) Spinning in circles b) Rocking back and forth for hours c) The inability to sit still for an hour d) The inability to maintain eye contact with another person

d) The inability to maintain eye contact with another person

A nurse is caring for a patient with benign prostatic hypertrophy (BPH) that has been refractory to management with medicines. The nurse anticipates the patient will undergo which procedure? a) Transurethral needle ablation b) Transurethral microwave thermotherapy c) Photoselective vaporization of the prostate d) Transurethral resection of the prostate (TURP)

d) Transurethral resection of the prostate (TURP) Transurethral resection of the prostate is considered the gold standard for BPH treatment. Although transurethral needle ablation, transurethral microwave thermotherapy, and photoselective vaporization of the prostate are all used to treat BPH, TURP is the preferred method.

A client is admitted with the diagnosis of possible myocardial infarction, and a series of diagnostic tests are prescribed. Which blood level should the nurse expect will increase first if this client has had a myocardial infarction? a) Alanine aminotransferase (ALT) b) Serum aspartate aminotransferase (AST) c) Total lactate dehydrogenase (LDH) d) Troponin T (cTnT)

d) Troponin T (cTnT) Troponin T has an extraordinarily high specificity for myocardial cell injury. Cardiac troponins elevate sooner and remain elevated longer than many of the other enzymes that reflect myocardial injury. ALT is found predominantly in the liver; it is found in lesser quantities in the kidneys, heart, and skeletal muscles, and is primarily used to diagnose and monitor liver, not heart, disease. AST, also known as (serum glutamic-oxaloacetic transaminase (SGOT), is elevated 8 hours after a myocardial infarction. Total LDH () levels elevate 24 to 48 hours after a myocardial infarction. Test-Taking Tip: Do not spend too much time on one question, because it can compromise your overall performance. There is no deduction for incorrect answers, so you are not penalized for guessing. You cannot leave an answer blank; therefore, guess. Go for it! Remember: You do not have to get all the questions correct to pass.

What behavior by a client with a long history of alcohol abuse is an indication that the client may be ready for treatment? a) Drinking only socially b) Not drinking for a week c) Hospitalization for detoxification d) Verbalizing an honest desire for help

d) Verbalizing an honest desire for help When clients with alcohol problems voice a desire for help, it usually signifies that they are ready for treatment, because they are admitting they have a problem. Adherence to an alcohol treatment program requires abstinence. A week is too short a time to signal readiness for treatment. Hospitalization alone is not an indication that the client is really ready for treatment, because many factors can influence admission.

When performing a physical assessment, the nurse identifies bilateral varicose veins. What does the nurse expect the client to report about the legs? a) Burning sensations in the legs. b) Calf pain when the feet are dorsiflexed. c) Increased sensitivity of the legs to cold. d) Worsening ankle edema as the day progresses.

d) Worsening ankle edema as the day progresses. When the legs are dependent, gravity and incompetent valves promote increased hydrostatic pressure in leg veins; as a result, fluid moves into the interstitial spaces. Clients report feeling an ache or heaviness in the legs, not burning sensations. Calf pain when the feet are dorsiflexed, which is referred to as Homan sign, most often is associated with thrombophlebitis. Increasing sensitivity of the legs to cold reflects inadequate arterial blood supply; arterial circulation is not affected by varicose veins.

You are walking down the hallway in the hospital on your way to the cafeteria for lunch. An older man walking towards you suddenly collapses. Your first step is to: a) provide 15 chest compressions b) give 2 breaths c) call for help d) assess responsivensess

d) assess responsiveness Assess responsiveness first. If the patient is unresponsive, call for help (call a code).

When caring for the patient who is dying, the nurse anticipates which of the following types of respirations close to death? a) tachypnea b) kussmaul respirations c) orthopnea d) cheyne-stokes respirations

d) cheyne-stokes repirations cheyne-stokes repirations occur most often when patients are approaching death. They are characterized by breaths that greatly vary in depth and volume, but tend to become progressively deeper and faster, then slow to an apneic phase. Orthopnea, tachycardia, and kussmaul respirations are not typically associated with impending death.

Which of the following is considered to be a late and ominous sign in pediatric clients with increased intracranial pressure? a) emesis b) pupil dysfunction c) decreased LOC d) cushing's triad

d) cushing's triad all of the given are signs of early increased intracranial pressure, except cushing's triad, which may indicate impeding brain stem herniation. Cushing's triad includes increased blood pressure, decreased heart rate, and irregular/abnormal breathing patterns.

A patient comes into the emergency room after being stung by a wasp. The patient is known to be allergic to wasps. You administer oxygen and epinephrine, obtain vital signs, initiate an IV and administer IV fluids. The patient is hypotensive. How should you position this patient? a) in the supine position laying flat in bed b) in the fowler's position making it easier for the patient to breathe c) in a lateral recovery positon in case the patient were to start feeling nauseous d) in the recumbent position with the legs elevated

d) in the recumbent position with the legs elevated Placing the patient in the recumbent position with the legs elevated above heart level will improve venous return and help increase blood pressure.

Your patient dies. The family and a few close friends are gathered in the waiting room. Regarding the body, you should: a) insist that they all view the body to assist with closure b) tell the family that they may view the body at the funeral home c) only allow the immediate family members to view the body d) provide the opportunity for those who wish to do so to view the body

d) provide the opportunity for those who wish to do so to view the body The family should be offered the opportunity to view the body and say goodbye if they so desire. They should not feel pressured to do so. Whether close friends view the body is typically the family's decision.

You male patient is complaining of dysuria, frequency, hematuria and mild swelling of the penis. His symptoms are MOST compatible with: a) pyelonephritis b) cystitis c) prostatitis d) urethritis

d) urethritis Pain and burning with urination are common symptoms of urethritis. Hematuria or blood in the semen, swelling of the penis and itching and tenderness of the penis may also occur. Cystitis, pyelonephritis and prostatitis do not typically result in swelling of the penis.

Your patient has just had a PICC (peripherally inserted central catheter) inserted. Which of the following statements indicates that your patient requires further teaching? a. "I will not lift more than 10 pounds with my affected arm." b. "I will inspect my catheter site daily for signs of redness, leaking, or swelling." c. "I will ensure that no one takes a blood pressure reading in my affected arm." d. "If my catheter breaks, I will apply tape and call my physician/nurse within 24 hours."

d. "If my catheter breaks, I will apply tape and call my physician/nurse within 24 hours." The patient should be counseled that should the catheter break, whether bleeding does or does not occur, they should apply sufficient pressure at the site with sterile gauze so that it is tightly and well covered and immediately report this to the physician. Maintain pressure on the site at all times. All other answers are correct.

A patient is receiving carbidopa-levodopa for parkinsonism. What should the nurse know about this drug? a. Carbidopa-levodopa may lead to hypertension. b. Carbidopa-levodopa may lead to excessive salivation. c. Dopaminergic and anticholinergic therapy may lead to drowsiness and sedation. d. Dopaminergics and anticholinergics are contraindicated in patients with glaucoma.

d. Dopaminergics and anticholinergics are contraindicated in patients with glaucoma.

The nurse has initiated teaching for a family member of a patient with Alzheimer's disease. The nurse realizes more teaching is needed if the family member makes which statement? a. As the disease gets worse, the memory loss will get worse. b. There are several theories about the cause of the disease. c. Personality changes and hostility may occur. d. It may take several medications to cure the disease.

d. It may take several medications to cure the disease.

Your client has an abdominal incision. Which of the following beverages will best facilitate wound healing? a. Milk b. Diet soda c. Apple juice d. Orange juice

d. Orange juice Orange juice contains high amounts of vitamin C. Vitamin C helps in the formation of collagen, which is important for wound healing.

A 52-year old patient experienced cardiac arrest from a myocardial infarction. During his acute care stay in the hospital, the patient flirts with all female nurses. When he is asked to stop, he withdraws and later complains of chest heaviness. What is a possible explanation for the patient's behavior? a. Boredom from restricted activity b. Lack of motivation to recover c. Frustration from illness d. Threatened self-concept

d. Threatened self-concept

You are caring for an 8 month old infant. Which of the following is most appropriate to ensure your patient's safety? a. ensure that the patient's call bell is within reach b. provide 1:1 nursing care c. Inform the parents that they are not allowed to leave the patient's room at any time d. ensure that the bars on the crib are raised to their highest level whenever the patient is unattended

d. ensure that the bars on the crib are raised to their highest level whenever the patient is unattended Ensuring that the bars on the crib are raised to their highest level will prevent the patient from falling. The infant will be unable to use a call bell. Providing 1:1 nursing care is not always feasible. Parents will need to leave the room for brief periods and should be reminded to ensure the bars on the crib are at the highest level. They should also be reminded to inform nursing staff when the patient is unattended

You are caring for a patient who has been diagnosed with AIDS following treatment for PCP (pneumocystis pneumonia). The patient's CD4 count is 300. You know that the guidelines for antiretroviral therapy state that patients should be started on therapy when the CD4 count: a. is > 500 cells/mm3 b. is ≥ 400 ≤ 500 cells/mm3 c. is < 500 cells/mm3 d. is < 350 cells/mm3

d. is < 350 cells/mm3 Antiretroviral therapy should be initiated in all patients with a history of an AIDS-defining illness or with a CD4 count < 350 cells/mm3. The presence of an AIDS-defininf illness, such as PCP, also indicates that antiretroviral therapy should be started.

What is the rationale for not using the gluteal muscle for injections in children until they are approximately 2 years of age? a. this area is too painful for children to receive injections in b. a painful muscle from an injection into this site may cause the child to stop walking and regress in behavior c. injecting into this area may cause bruising which may be mistaken for child abuse d. it takes time for the gluteal muscles in children to develop after they have begun to walk

d. it takes time for the gluteal muscles in children to develop after they have begun to walk In children, you do not inject into the gluteal muscle until they have reached approximately the age of 2 and have been walking for a time sufficient to develop the muscle

Your client is in active labor at 41 weeks. The membranes rupture and you note that the fluid is thick and green in color. This is indicative of: a. blood b. infected amniotic fluid c. normal amniotic fluid d. meconium

d. meconium Amniotic fluid that is stained a greenish color and is thickened contains meconium, the newborn's first stool. This is a warning sign of potential fetal distress. The physician should be notified, and the newborn may require tracheal suctioning immediately following delivery.

Your client has legally appointed health care surrogate who decides to withhold all food and fluids from the client. The continuation or discontinuation of food and fluids was not documented in the client's advance directives. You feel that withholding food and fluids is cruel and inhumane. What is your best action? a. withhold the food and fluids even if you believe it is cruel, inhumane, and illegal b. report this ethical dilemma to the client's family and doctor so that it can be resolved c. feed the client and provide fluids to ensure humane care even when you feel it violates ethical practice d. request an ethical conflict resolution with the ethics committee or the nursing supervisor

d. request an ethical conflict resolution with the ethics committee or the nursing supervisor Many healthcare facilities have multidisciplinary ethics committee that convene to resolve ethical dilemmas and conflicts. Some of the commonly occurring ethical dilemmas that impact clients revolve around euthanasia, physician assisted suicide, the continued administration of pain medications to relieve pain even when it hastens death, advance directives, and withholding food and fluids

Your client is receiving a continuous gastric tube feeding. The rate of the tube feeding is 75 mL per hour. You aspirate 45 mL of gastric contents. What should you do? a. discard the residual and discontinue the tube feeding b. continue the feeding and discard all of the residual c. return the measured residual and continue feeding d. return the measured residual and discontinue the feeding

d. return the measured residual and discontinue the feeding You should return the residual gastric contents back into the tube because it is less than 150 mL. Residual contents over 150 mL are not returned. You should also discontinue the tube feeding because the residual is more than 50% of the hourly rate. Half of 75 mLs is 34,5 mLs, and you have aspirated 45 mLs.

hyperemesis gravidarum

severe nausea and vomiting in pregnancy that can cause severe dehydration in the mother and fetus. 1. Related to HCG hormone increase 2. Commonly seen in first pregnancy, multi-pregnancies 3. Hx: H. Pylori 4. Tx: Small frequent meals, drink between, IV fluids and electrolyte replacement. 5. TPN & PPN through central line (worst case scenerio)


संबंधित स्टडी सेट्स

Excel Chapter 7: End-of-Chapter Quiz

View Set

Entrepreneurial Small Business Chapter 9 - Small Business Marketing: Customers and Products

View Set

CH 13 - Abstract Classes and Interfaces

View Set

7 Antibacterial Drugs That Disrupt the Bacterial Cell Wall

View Set

Flashcards from HTML and CSS lesson

View Set

Dilations ( assignment&quiz ) 100% and 70%

View Set

Lifespan G&D Final Review Unit 4

View Set